You are on page 1of 131

AJC Mid-Year 07

PAPER
1

1. 2. 3. 4. 5. 6. 7. L 9. 10. 11. 12.

To what extent should parents be held responsible for their children's actions?

Art has no place in today's modem wodd. Comment.


Should the State interfere in what a person wants to do with his body? "The road to hell is';ften paved with good intentions". Discuss.

Examine the view that when fighting a war, we should not be concerned about principles.
Freedom of choice

is this always good?

Shorrld young people in your country be fearful about the future?

The need for ethical standard among scientists is greater today. Comment. Shopping is_a national pastjme for Singapore. How far should this trend be encouraged?
Small businesses cannot survive in toclay,s world ot big players. Do you agree? Break the rules. How much would you advocate this? To what extent should the problems of a country be the world,s concern?

AJC Mid-Year 07
INSERT

ctTrEs
Passage

l:

Peter Hallwrites ..-.

Freud famously said that after 30 years researching the feminine soul, he'd never answered his real question: what does a woman want? We urbanists could likewise confess that we've failed to answer our conundrum: what makes cities tick? Why do cities have brief golden ages, but then languish? Why are Athens or Florence or Vienna no longer qeative powerhouses? Why have Manchester and Berlin, once workshops of the 5 world, given way to Guangzhou and Shanghai? Why do some citjes seem to retain, or regain, their power?
Some of us think the answer ljes in number crunching-as with economist Richard Florida, author of ''fhe Flight of the Creative Class,,'who iound that concentrations of gay people correlated with urban creativity. The problem js that even jf this approach works io;one city 10 at one lime, it may not work for others. The other method is to use history, to ask how precisely it was that great cities came to be great. Here, the danger js that you may end up with a series.of unique one-off explanations. The challenge ts to-RnO if tneri,s anyihing thb stories have in common_ And they do. Look at creative cities at their zenith: plato's Athens, Michelangelo,s Florence, Shakespeare's London, Mozart,s Vienna. All were economic lea;ers, cities";t the heart of vast trading empires, places in frenzied transition, magnets for talented people seeking fame and fortune. Outsiders made these places what thjy were: ntnens;i vJrstn greenot card holders, the noncitizen Metics; the Jews in .190d Vienna; fo.ergn inists in paris tge. They were patrons because many had made-monet from rrade, 20 ::":.19 as anrsts., tney occupied a speciat marginat position: not at the hean of courfly or ::_ye anstocratrc estabtishments. yet not entirely shut out either And thus they absorbed and reflected the huge tensions between conaervahve and radrcar rorces thZt ihreateneo to divide these societies.

tl"-TT

It was the same, but with sub e differences, in the great manufacturjng cities.. Consider lvlanchester in 1780, ctasgow in 1850. Detroit in ig.to-, Silicon Vafley in f-S6d bassage; egalitarian places open to tate'nt, Jt-improving ano 1L:"sen-eoucattng, :']:t""ilJi9 engaged in learning and innovation through networks that were at once competitive and cooperative. There are astonishing pa..Gls between L;ncashire in the 1780s and t790s.and Siticon Va ey in the 1960s anO tne tgZOs. tn Uottr, one innovation 30 Droughr ronh another in great chains of creativity. places like these flourished not because of physical circumstance, but because their peopre demonstrated innou"tive energy. ""iejtionat

ylP:t

n;

;"i"

What are-the 2'tslcentury equivalenls? They are the great global megacity regions mr ion people around. cities ritJLondJn. ruew i"ork ind -Hong as li:"1"9^r"9 llli,o-"."r 19 Kong. here too ts huge innovative power,_charging through the cores of the great centraj cities but also diffusing out into neighboring places ihrough elaborat; networks of information exchange. This information, generaGd and exchangid and reprocessed, forms the raw material of the new urban economic drivers. This nelpei to establish lhe advanced services: finance and business services; command and contror tunctions- uotr-li +o governfient and private business; creative and cultural industries like the media, higher education and health care. And these, in turn, generate a vast array of consuimer services----entertainment, personal, and hospitality:which simuttaneously cater to the

disposable incomes of affluent residents and to business and leisure tourists.

Ihus, great cities thrive if they do a good job of information processing and knowledge 45
creation. lncreasingly, there is a winner-takes-all rule: top places and their regions grow at the expense of smalle. minnows in the pond. Success breeds success, aided by builFin self-reinforcing advantages like a major jnternational airport. One additional factor asserts itself in a world where information is a homogeneous traded commodity: the English language. The great English-speaking metropoles, London and New York, increasingly dominate the global information economy, attracting firms, investment, capital and talent. But cities and their people have surprised us before. The 21st century, experts agree, will be the Asian century. China, for sure-lndia, most probably-will win back the foremost positions they occupied in the pasl history of civilization. Thear past record and present achievement both suggest that they are powerfully placed in the great race to marry artistic creativity and technological innovation, despite China's language disadvantage. Their return will come through the creative power of their great cjties*Shanghai, Beijing, perhaps l\,4umbai-rapidly rising in the global hierarchy to challenge today's teaders. Adapted from Peter Hall's'How Cities become Great,'. (Newsweek International)

50

55

Passage 2: George Yeo wntes


Economically, the world is breaking up from empires and big natjen- siates to small states, provinces and city-regions. Small states, each with a population of less than i0 rnillion, make up two- thirds of the members of the United Nations. lncreasingly, it is at the level ol city-regions that competition for human talent and ,nvestments takes place. All over the world, institutions that evolved in response to the needs of an earlier period of industrialization are no longer adequate. Smaller, more responsive units of organization are required. A pattern of competition and cooperation among city-regions will appear, not unlike the pattern in Europe before the age of nation-statea, with international organizations Iike the old Hanseatic League providjng loose coordination. lndeed, we are somewhat surprised by the jnterest in Singapore by big nations such as China and lndia and by distant places like South Africa, Kazakhsian and the new Palestinian state. Their interest reflects the increasing fragmentation of the world into cityregions, each of a size and scale comparable to that of Singapore and its 3 million people. China, for example, is now divided administratively into aity- regions, each of about 2 milljon to 10 million people. These city-regions have considerable autonomy. Each must sojve probiems ol urban planning, housing, tratsportation, road congestion, education and policing while attracting investments and creating jobs.
ln this age of city-regions, Singapore,s experience as a city-state becomes usefulto others.
10

15

Singapore, as an independent city-state, has advantages over city-regions that are parts of nalion-states. The greatest advantage is our ability to control the movement of people into 20 Singapore. lnstead of indiscriminate urban drift, we select migrants based on talent, income and other criteria. Without this, Singapore would be like many other fast_growing cities in the Third World, with high crime rates, traffic congestion, slums, prostitution, drug addiction and severe pollution.

ln this new world, a new balance between rights and duties, independence and 25 interdependence, competition and cooperation wjll have to be found. The ideas of democracy and socialism will have to be reinterpreted East Asia wjll make a major contribution to this reinterpretation, not because East Asians are wjser. Almost 1S0 yeirs of war and revolution have brought untold suffeting to the region. yet precisely beiause the destruction has been so complete, reconstruction has been made much easier. 30 Singapore, like most of the countries of East Asia, is in a relatively youthful phase of
2

development. lnstitutions are still flexible.


foolhardiness. fills the air.

can-do spirit, sometimes bordering

on

Westeln liberals oflen sneer at Asian forms of democracy which are still relatively young. Westem democracies prioritize individual rights, unlike the Asian model of group solidarity However, Western liberals often forget that a democracy without group solidarity can become a game where wealth is redistributed from the rich to the poor in the form of aid and from the disorganized citlzens to the organized bureaucracy. Wjthout strong moral underpinnings supported by the entire community, resentment from the rich and poor alike will inevitably result. Democracies which see only rights without obligations eventually destroy themselves. 40 the precise reason for Singapore's version of socialism- ln many ways, Singapore is socialist, especially in its enormous subsidy of housing, health and education. Socialism works when jt strengthens group responsibility. lt is dysfunctional when it leads to individual iresponsibility. lnstead of tl^re Westem social security systems similar to that of 45 the unbreakable communal iron rice bowls in Maoast China, Singapote deliberately works our welfare policies through the family. The objective is to strengthen the family net, not weaken it.
Thas is

Treatment of minorities is another aspect of an evolving Eagt Asian democracy that bears watching. ln a winner-take-all, one-man-one-vote situation, minorities will revolt against dominant majority. Other ways must be found to ensure fair minorjty representation. Singapore created Group Representation Constituencies, which forced all major political parties to field a multiracial slate of candidates in parliamentary elections. ln lndonesia, pancasila democracy conscjously plays down Javanese dominance. ln the Association of Southeast Asian Nations, considerable importance is given to consensus-building.

50

socialism are being conducted. Some will succeed, others will fail. lf Western influence had not affected every facet of life in Asia, this enormous transformation would not have been possible. ln the same way, the rise of industrial Asia will eventually have far- reaching effects on the rest of the world, inctuding the West.

ln East Asia today, institutions are still plastic. lvlajor experiments in democracy and

55

Adapted from ceorge yeob "ln Asia and Eisewhere, Smaller Will Be the Better Way to Govern,l ( I nt ern ati o n al H e ratd Tri b u ne )

Queslions on Possoge I 1. According to paragraph 1, what is the key question that urbanists have failed to answer? Use your own words as far as possible. trl

2.

ln your own words as far as possible,

a) b)

identify two possible methods from paragraph 2 used to find out why cities thrive. explain the problem with these

methods.

t3l

3.

From paragraph 3, state in your own words as cities have in common.

fal as possible three characteristics that creative


I3l

4.
5.
6.

Explain in your own words as far as possible what the author means by the .special marginal position" (line 2'l) that outsiders

occupy.

tlj

Why does the author use the word 'astonishing, (line 29) to describe the parallets between Lancashire and the Silicon

Valley?

t1l

Explain what the author means by 'a winner-takes-alt' (line 46).

I2l

According to paragraph 7, what are the factors that led the author to believe that the 21st century will be the Asian century? Use youl own words as far as I2l

possible.

S 9

Queslions on Possoge 2 According to the writer, why should singapore be surprised by the sudden interest from big nations (tine

11-12|?

Iil

summarize the factors which conhibute to singapore's success. using material from paragraphs 3-7, write your summary in no more than 120 words. Use your own word; as far as possible.

singapore's success depends

on...

t8l

answer a word or short phrase. From Passage 1: zenith (line 15) baggage (line 27) drivers (line 39).....

10 Give the meaning of each of the io|owing words as ihey are used inthe passage. you may write the jn

From Passage 2: bordering (line 32) plastic (line 55)


t5l

11. Both

althols discuss factors contributing to a cityls success. How far do you agree with their views? How far do you think Singapore is a great city? lllustrate your arguments by referring both to what articles experiences. I8l

you have read in the

and to your own

AJC Mid-Year 07 Paper 2 Answer Scheme Questions on Passage


1. According to

paragraph 1, what is the key question that urbanists have failed to answer? Use your own words as far as possible. [1]
Lifted L3-7 Why do cities have brief golden ages (1/2), but then languish('1l2)? OR Why do some cities seem to retain (1/2), or regain (1/2), thei. power? Paraohrased They failed to identify the reasons tor a city's rise (1/2) and fall (1/2). OR They failed to identify the reasons for a city's ability to maintain (112J ot teestablish theirdominance/success ('112).

2. ln your own words as far as possible, a) ldentify two possible methods from paragraph 2 used to find out why cities thrive

l2l
Lifted ..number crunching.. .. use of history....
Stalisiics ol populalion Refer back io lhe past

ParaDhrased Analysis of statistic$ or data/ trend ('l) Study/ examination of past events/ history (1)

>

1m m

b)

Explain the problem with these methods [1]

Lifted ...even if this approach works for one city at one time. it may nol work for others ..

Finding pasi records orn Simiarities between cities would be imposslble lo find 0m Allhough it is suiiabe for a cily al lhat period, !!j?) may noi be suitable for olhe6 (0) (0) Too many dislinci reasons/ d ffcjlies would have difi reasons

Paraohrased Because it is difficult to find similadties between cities OR The results cannot be applied to all citjes OR The methods cannot be used to draw conclusions.

3. From paragraph 3, state in your own words a9 far as possible three characteristics that creative dties have in common [3] Lifted (L16- '18)All were economic leaders, cities at the heart of vast trading empires, places in frenzied transition, magnets for talented people seeking fame and fortune.
Excellent economic stalus (12) Crucial stalus forlrading (O)

ParaDhrased {Anv 3\ Each is an economic power ('l), a centre

forlrading activities (1). These cities


experience franiic ('l12) development (1/2) and draw skilled personnel (1) in search of reoutation and wealth.

Crcalive people
Arts people (0)

11)

4. Explain in your own words what the author means by the 'special marginal position" (line 21) that outsiders occupy ['1]
Lifted

L21-23 : not at the heart of courtly or aristocratic establishments, Yet not entirely shut out either.

Paraphrased Outsiders seem to occupy a unique peripheral role among the nobility - socializing within their midst but not being entirely accepted by them. OR Outsiders are not part of the aristocrats (1/2) but were welcomed bV them anyway (1i2)

X noi aboul physical location

5 Whv
I ifted

parallels does the author use the word 'astonishing' (line 29) to describe the Valley? [1] beMeln Lancashire and the Silicon

lrnferredl

-CGxbt

ParaDhrased

ahiost two centuries apart (1/2), so they should be vastly different. Yet they share many srmilarities. (1/2)

6. Explain what the author means by'a winner-takes-all" (line 46)i2m)


Cities who have managed to get ahead in the race (1/2) \,!ill expand or develop(1i2) , leaving no chance to losing crties to calch up/ hinder the gro$4h of others/ monopolise the

(L46-47) lncreasinglY, there is a winner-takes-all rulei too olaces and their regions glg!{q!]he expense of smaller minnows in the pond.

author to believe that the 7. Accordrng to paragraph 7, whai are lhe factoB that led the
21e century wtll be the Aslan century? (2m) Lifted

(L54-55)Their PAg! record and present achievement both suggest that they are PgllglbllY plA!9d in the great race to tr4ry adistic creativitv and technoloaical innovation

What they have done prevlously (1/zJ ano now r112) demonstrates that they have the capablldy irlzj to orawl comb,ne the merits / origrnal ideas rn the arts and science (1/2)

Questions on Passage 2 should Singapore be surprised by the sudden interest why writer, 8. According to the ,.^nari^nc /li^a 11-12 '.i^ Paraphrased Lifted as such nations (L11-12) ... bY !!g Because even significantly larger nations China and lndia and by distant places (1/2) and those who arctaraway \112\ ate like South Africa, Kazakhstan and the observinq us. new Palestinian state thew ter

@rhesurprlseof

9. Summarize the facto.s which contribute lo Singapore's success usinl materiattrom paragraphs 3-7, write your summary in no more than 120 words
I8l

Lifted
1

Paraphrase
The sove.reign nature of Singapore as opposed to other city-states who are subjected to the sovereiontv of the country. strict immtgration rules

..independent city-state..

t
3

our ability to control movement of people into Singapore. (120-21)


...

w-ave

4 5
6 7

lnstead of indiscriminate urban drift, we select migrants based on talent, income and other criteria... ... relatively youthful Phase of development (L31) ..flexible (132) ..can-do spirit (132)

And we grant entry only to eligible applicants/ based on criteria of ability/ merit ln terms of progress, Singapore is still in its budding period. ,

And hence, Singapore is willing to accept changes And remains adventurous/ gung-ho/ willing to ky Singapore's success is also attributed to their people working towards a common goal
bond (0)

....a democracy without grouP solidarity... (136)

strong moral underpinnings (L38)

while being guided by the same set (1/2) of values(1/2) that are firm/ unwavering/ unchanging/ firm (1/2) Singapore has a peculiar brand of socialism (1t2) which strengthens group responsibility ('112) through strengthening the family net (1/2)

...Singapore's version of socialism... (L41) ...works our welfare policies through +h- {.mih, /l 1a\

10

...minorities will revolt (L50) ......ensure fair minoraty reoresentation. (L51)

Singapore ensures stability (1/2) through ensuring that all minority groups participate in the oolitical svstem. ('112)

Total of 11 marks

10. Give the meaning of the following words as ihey are used in Passage 1 and Passage

You mav write vour answer in one word or a short phrase. (5m 1t2 0 peaU high poinu furthesu (P'1, maru line15) highest zenith prime/ most top successful period of time/ acme/ apex/ oinnacle weight (line27) something that burden / baggage encumbrances holds you back/ obstacle force catalysu impetus/ driver (line39) propeller/ propellant (idea of force must be there) resembling extreme proximity/ bordering (P2, line elmost like/..close 32) to/ edoino plastic (line 55) malleable/ subject to change/ plianv flexible

Both authors discuss factors contributing to a city's success How far do you agree with their views? How far do you think Singapore is a great city? lllustrate your arguments by referring both to what you have read in the articles and to your own experiences.
'1

1)

Question requirements: Make a stand for both questions Cite both

fl

Economic leadcrs

(Ll6)

Singapore, as an indpendent ciry-state, has advantages over city-regions thal arc parts ofnalion _ states (Ll920) Our ability to controlthe movement ofpeople into Singapore (L20-21) lnstead of ind iscriminate urban drift, we select migrants based on talert, income and ollrer criteria

2.

Citics

at the heart

ofvast rradin8 empires (1,16-17)

(L2t-22\

3.

Places in frenzied transition

(Ll7)

lmplicd: Iow crime

rates, little traffic congestion, no slums, prostitulion and drug addiction arc not serious

problems, no pollulion (23-24) Magnets for talented people seeking fame and fortune In a relatively youthful phase ofdevelopment (1,3l -12

Ll7-18
Outsiders made thcse places what they were

(Ll

lnstitutions are still flexible (L32) ), institutions are

. .

ioutsiders] occupied a special marginal position: not at tho heart of courtly or arislocratic establishments, yet not entirely shut out either' They absorbed and reflected huge tnsions belween conservative and radical forces that threalencd to divide these societies

stillplastic (L55)

6.

Manufacturing cities Places without arisiocratic baggage (L27)


Egalitarian places open to talent (L27)

A can-do-spirit, somelimes bordering on foolhardiness, fills the air (32-31)


Singapore's version ofsocialism enormous subsidy ofhousing, health and

7.

. . .

education. (L42)
Strengthens goup responsibilify (L47) Works wclfare policies through the family strengthen the family net (47-48)

8.

Sell improving and self-educating (L28)

Treatment of minorities:

engaged in leaming and innovation through networks that were at once conpetitive and

Ensute fair minority representation through

GRCs (52)

.
9.

coopcrative l lroir peoplc demonslratcd exceptional innovative energy

Huge innovative power, charging through the corcs ol the great central cities but also diffusing out into the neighbouring places through elaborate networks of
exchangc (L36-38)

'nibrmation ]0 Advanced services: finance and business scrvice, command and control lunctions both in government and privatc business;creative and cultoral industries like the media, higher education and health care. (1,39-

42\

tl

Vasl array of consumer srvices cntertaiDmenl, personal and hospitality - which cater to the disposable incomes ofaffluent residenls and to business and leisure tourists. (L,l2-44)

t2 Creat cities thrive ifthey do a goodjob olinformation processing and knowledge creation. (L45-46)

ll

'fhe Englisb lang.rage. The great English speaking


metropoles increasingly dominate the global information economy, attracting fi rms, investment, oapital and talnt ((L49-51)

14 Marry artistic creativily and technological innovation

(L55-56)

Passage 1: RE: 'magnets for talented people" (Line 17) EV/ EX: Agrees to this being an important factor for success. Student can discuss the importance of foreign talent in cosmopolitan cities in the world today. But any 'attractive' city may very well attract the'wrong people' and hence lead to the problems raised by Yeo (L23-24 - high crime rates....). Singapore has done well in preventing such problems from occurring through their stringent immigration policies Foreiqn t;lent plays a major role in ensuring Singapore's success Through tilling up of jobs that the educated Singaporeans do not want (eg. Nursirg or labourers) to plugging ihe gap left behind by the aging population, Singapore has managed to maintain ecoiomic development at a sufficient level and till today remains a "magnet for talented people". Our attractiveness, thus, is testament of our success Other points discussed: l\reritocracy Multiculturism Hall (impt of creativity) - lnnovation / eg. Spring Singapore; creative community Singapore's emphasis on lifelong learning in relation to Hall's view in para 4 ("engage in learning")

cJc

Mid-Year 07 Paper

1.

How far do you agree that terrorists should be pitied, not hated? An effective government requires a free press.' Do you agree? "Human life and dignity are sacrosanct." Do you agree thatthis principle is overemphasised today? Environmental conservation is more important than economic developmeni today. Discuss.
"

2.
3.

4.
5.

Women do not need equality today. Men do." What are your views?

6.
7.

'Globalisation privileges the rich more than the poor.' How far is this true?
To what extent should the decision to get marrie'd be an individual choice?
Do people rely too much on medical science for the ills of life?

L L

Should extreme sports be banned? stumbling block for young people is their sense of hopelessness.' To what extent is this true with regard to Singaporean youth today? museums still important in modern society?

10. "The

11. Are

12. Why write?

END OF PAPER

,/tr

CJC Mid-Year 07 Paper I (Version 2) "Singapore's education system has failed to address the needs people." How far do you agree with the statement?

l)

of our young

. . r
2)

Essays should evaluate the extent to which policies, curriculum, teaching strategies etc havg catered to what young people require or want out of life or find necessary or useful in life (these wants must bejustified). Weak students who merely list the failings of the education system without linking it to the needs oflhe yonng people will not pass. For balance, essays should examine both the successes and failings of the Singapore education system in relation to what the young people require.

"lndividuals must do more to reduce environmental danage." Discuss. Keywords: MUST DO MORE. Answers should analyse the necessity of individual efforts in conserr'ing the environment. Answers should evaluate and assess what is cunently being done before deciding if more shouid be done or what more should be done in the near

. . .

future. Students need to evalllale the extent to which the individual can effect any

change in the conservation of the environment. Students may consider individuals linking up with olher $oups to increase their influence and ability to cut down on environmental damage. A balanced essay should have a comparative analysis ofthe efforts ofthe individual in relation to the govemment and other environmenlal groups. For example, through govenment lobbies and not voting for political parties that do not support or cncourage envircnmental conservation.

r)

How much influence do parents still have on the lives oftheir children? Students need to compare and conhast the degree ofcontrol/ sway parents have on thc lives of their children in ateas such as studies, work, leisure and character in relation to parents ofthe previous generatiot Better essays should provide justification lor their stand by citing reasons such as changing societal values, disintegralion of families, changing nalule of work, mass media influences, inlbrmation technology, alcohol irnd drugs, educational changes etc. A weak essay will simply list thc positive,r ncgative things parents do and avoid the discussion ofparenlal issues in relation 10 changes ovcr time

. .

4) "Advefiisements aro manipulative and misleading." Discuss.

Students should evaluate the degree

. lt

advertisements exploit consumers by playing on their minds and providing inacourate information is necessary to identiry and evaluate the techniques of persuasive advertjsing and give specific examples as theirjustification.

to which

IJ

For balance, students should show how advertisements can be honest as well as inlbrmative of new products and services available or relay impoflant information and policies to the masses. Weak essays will merely list advertisements that are deemed to be misleading without justificalion or analysis or make a simplistic comparison between persuasive and informative advertising.

5) Are youths loday complacent about the future?

. . .

Students should evaluate if the atl.itudes of youths today reflect overconfidence, smugness and a lack of worry about the challenges of the

. . . . .

fuhlre. They should compare and contrast youths oftoday with previous generations of young people in lerms of their attitude / behaviour. Thc 'Future' should encompass political, environmental, economic and social scenarios. Specific examples oI youth complacency include poor voter turnout, politicai apathy, lack of community/environmental awareness and social indifference. Examples slrould not only come from Singapore. A wearl essay will limit itselfto personal anccdotes.

6) Assess the impact ofmodeln methods olltransport on societies. Students should examine thc ways modern methods of transport like Mass Rapid Transit, cars, aeroplanes, buscs, even energy-saving modes oftransport

havc changcd societies and weigh both the positive and negative effects of
these changes.

Sludents should consider the economic, social, political, cultural and


environmcntal impact and also its effects on the lifestyles ofindividuals. Weak students wiil be purely descriptive of the different modes of transport and their advantages and disadvantages.

7) With globalization, talent is becoming more mobilc. ls this a positive trend? Students must show an unde$tanding of global compctition for the best and the brightest. They should evaluate if such a phenomenon is positive for the individual and the nations that gain and lose the 'lalent'. Examples of negative elfects would be loss of cultue, palriotism and family

r .

values. Positive eflects could include the acquisition of new knowledge, expertisc and better opportunities, leaming of new cultures and countrics becoming m.rre cosmopolilan. Good arswe$ would consider the effects ofa reverse bmill drain and assess i1. Other arguments can be that of the rise of a common intellectual pool, a common language, a common culture that may result fiom the mobility. They should also give a global perspective and a range ofexamples. Weak answers may just iist some consequences of the movement of talent without much evaluation

8) Can television promote healthy lifestyles? Students have to discuss the potential of television to actively or indirectly encourage a healthy way ofliving. Students should look at programmes that actively promote healthy living and representations of healthy or unhealthy living (smoking, drinking, bad eating habits) in the content of the programmes Healthy lifestyles would largely refer to a way of life that leads to physical, fitness, mental alertness the idea of a sound mind in a sound body through active participation in areas such as spofis, travel, yoga and dance. Weak ossays may just list the vaious types ofhealthy programmes to justif

. .

the stand.

e) Account for the phcnomenon ofviolence in schools today. Students must give an cxplanation/ reasons for the occurrence of violence in many schools in the world today (E.g America gtur cultuc and easy accessibility to guns. Japan, Singapore, Korea unhealthy pressure on young people to stay ahead of the pack which manifests itsclf in lu d ways). Good essays will evaluate and explore the underlying causes behind rccent tends of violence in schools causes by looking at changes in the l'amily' society and school. lor example they should look at the underlying reasons behind thc increase in bullying incidents in schools today Weak Essays will merely list examples of violencc that maybe narow, isolated and anecdotal.

10)

Will

newspapers become obsolele in the near future?

Students should evaluate

if the newspapers will remain viable and suNivc

in

coming Yeals. Studenls should show an unde$tanding that thc newspaper as an industry may not be economically viablc due to the ioss of advertising rcvenue as they are curcntly tlueatened by newer forms ol mass media like the internet ncws websites, sms news updates, l-tc and an emerging younger population thal want their inlomation fast, accessible and in bite sizes. Weak scripts may list the pros and cons of newspapers without linking i1 to whether newspapers would be phased out in the coming years'

ll)"Singaporc has not done enough to foster


agree with this statement?

entrepreneurship

"

How far do you

Answers must evaluate the extent to which schemes and policies by the govemment, govemment-linked bodies, national organizations and corporations have effectively encouraged or curtailed the growth of private enterprise (i.e. individuals setting up their own business ventures and taking risks) in Singapore.

. . .

of national e g' Singapore initiatives to develop the entepreneu al spirit in A*Star, SME funding, MOM policies, educational policies, role of EDB. Answers must assess whether or not such initiatives have been successful in developing entrepreneuship. Students may evaluate the examples oflocal successful entrepreneurs such as Oli,"ia Lum, Sim Wong Hoo. Adam Khoo' Ceorge Quek in rheir justification. Weak Essays will merely provide a list of local initiatives without assessing its effectiveness in fostering entrepreneurship'
influence ofpop music on society today A requirement would be the evaluation of the exlent and nature of the effects ofpop music on modem society by considering its impact in any of the following rclevart aleas: social, moral, psychological, political' economic and cultural Studnts should justify the social, political effects of pop music by citing specific eiampies of the impact of pop starc such as Michael Jackson, Elton John, Bon Jovi, Stephanie Sun' Better answers should have a wide range of examples and cover different countries and cultues in relation to effects on the different segments of society such as children and youth. Weak answers will merely list the positive and negative effecfs of pop music without evaluating the degree ofinfluence'

. It is necessary for students to give specific examples

12) Assess the

. . o

'

/(

l.

CJC Mid-Yeqr 07 Poper 2 Lewis Thomas writes...


Everyone must have had at least one personal experience with a computer error by this time Bank balances are suddenly reported to have jumped from $379 into the millions, appeals for charitable contributions are mailed over and over to people with crazy-sounding names at your address, department storcs send the wrong bills, utility companies write that they're tuming everything off, that sort of thing. lf you manage to get in touch with someone and complain, you then get instantaneously typed, guilty letters from the same computer, saying, 'Our computer was in error, and an adjustment is being made in your account.'

These are supposed to be the sheerest, blindest accidents. l\,4istakes are not believed to be part of the normal behaviour of a good machine. lf things go wrong, it must be a personal, human error, the result of fingering, tampering, a button getting stuck, someone hitting the wrong key. The computer, at its normal best, is infallible.

'10

I wonder wheiher this can be true. After all, the whole point of computers is that they

represent an extension of the human brain, vastly improved upon but nonetheless human, superhuman maybe A good computer can think clearly and quickly enough to beat you at chess, and some of them have even been programmed to write obscure verse They can do anything we can do, and more besides.

15

It is not yet known whether a computer has ils own consciousness, and it would be hard to
find out about this. When you walk into a computer laboratory and stand listening, it is easy to imagine that the faint, distant noises are the sound oi thinking But real thinking, and dreaming, are other matters.
On the other hand, the evidences of something like an oncorscious, equivalent to ours, are all around, in every mail. As extensions of the human brain, they have been constructed with the same property of error, spontaneous, uncontrolled, and rich in possibilities. 20

lvlistakes are at the very base of human thought, embedded there, feeding the structure like root nodules. lf we were not provided with the knack of being wrong, we could never get 25 anything useful done. We think our way along by choosing between right and wrong alternatives, and the wrong choices have to be made as frequently as the right ones We get along rn life thrs way. We are built to make mistakes, coded for error'

We learn, as we say, by 'trial and error'. Why do we always say that? Why noi 'trial and rightness' or'trial and triumph'? The old phrase puts it that way because that is' in real life, 30
thg way it is done.

A good laboratory, like a good bank or a corporation or govemment' has to run like

a add up to the numbers book, and all by the is done flawlessly, everything computer. Almost the predicted sums. The days go by. And then, if it is a lucky day, and a lucky laboratory, somebody makes a mistake; the wrong buffer, something in one of the blanks, a decimal misplaced in reading counts, the warm room off by a degree and a half' a mouse out of his box, or just a misreading of the day's protocol. Whatever, when the results come in, something is obviously screwed up, and then the action can begin.

35

The misreading is not the important error; it opens the way. The next step is the crucial one lf

/7

at thall',then the new finding' 40 the investioator can bring himsell to say, 'But even so, look i";"Jt io"r snatctring what is needed' for prosress to be made' is the move ;;il;i;,
based on the error.
10

or new varieties of music' Whenever new kinds of thinking are about to be accomplished' sldes debaling in the same mind ir]!i" r'r"i u" an argument-beforehand With hto other wrong sooner 45 ia an imiable understandrng thal one is.nghl and the ;i"r.a-nn;;;s, not the two sides and lifrt""|. tiJir'ing is settled, but there can be n; action at all if there are towatd error The tendency the ;;.-;;;;""1. ir'e hope is in the facultv oi wrongness side represents wrong of info';ation to l;nd lightlv on the r""p

i" in;;

"",""i?ounrtin" the highest of human endowments

i"o*,i,ii"

11

our genetic 50 human grft p--"'hap" eu"n "jl?'31:9-li as a mistakes making for instru&ions Other creatures do not seem to hlve DNA sequences guide action for iJin" p"rt oic"irv ri"ing, certainly not for programmed error as a

It may be that this is a uniqueiy

12

therc are more choices than two We are at our human finest, dancing with our minds, when go, all but.one, bound to be wrong' iorn"t,r*"tn"* are ten, even twe;ty different ways to ground This 55 of selection in such situations can lid us onto totally new a single center only had fa ibility. tf we ,ri"""" i" i"i[J "-ptoiaiion anc is based on human rnstead of be made of responding onll, wl"en a correct decision was to provlde lor being ir," i,,-nf" ot aiterent. credulous, easrly conned clusters of neuro'les thal rp ir"", down dead ends. out rnto blue sky. a'ong wrong turnings. irr"iiriii"t"' oi"J 60 "] "v". tn" *ul, *e are todav stuck

;;i i;; ;il;"t" ;;;;r;;;;;;;t

#,Hffi; ;; ililonl lw

rast

13

are limited' most of them to The lower animals do not have this splendid freedom They never seen a Cats, for all their good side never make,mrstakes ltrave "il"oiui"-.f"ffiOiriiy dogs are sometimes fallrble' occasron-ally,able to make r"jli-t, "r""i"1ll "i blundering catget their masiers Fish are ;;"ii;itt"kes, out iney ihis way bv trying to mrmic perfect in 65 "n"i.i"g' tney oo tnoiv.iuai cells in a tissue-are mindless machines' n"rr""""i" """,vtni"g as absoluteiy inhuman as bees their performance,
complex computers for the We should have this in mind as we become dependent on more let them go thelr way lf we arfanoement of oul affalrs Cive the comouters iheir heads l say: proceeds' the ;;:, ,ulning ;ui h""a" td one side and wincins while the.work good 70 .nsqih,lrlies for the fulure of manklnd' ano computerkind are limitless Your average ;;r."r"iion" in ;nt,"nt which would take a liretime of slide ru'es for anv "n gain from the near infinity of precise' mechine-made J- *n'rt """r0 would begin the solving of some rn,slomoutat,on wnicn is now so easrly withrn our grasp. we for r*-iti"t-"e should we so about orsanizins ourselves plain fact of life a single 75 fNins on a planetary scale, now that we have beco-me' as a do;ng thrs are "*i"f ussume. as a working hypothesis. that allthe right ways of ""-i.,,.n"'" w. much alternatrves of wrong "ln in"" tor mivrng ahead is a sel w-h"i up think can of us ""-"""4, than the short list;t mistaken courses that any il'^o. :nrt more interestinq to computer when rt is printed out we need the in iuli in inrin,t" r,"t w; ";;;, tn" "nd to go lf it is a brs enoush mistake' we could 80 "6*r "av fhd ourselves on a new level' stunned, out in the clear' ready to move agaln

14

il;i5;;i;'a; ::;"";J;;.;; -"" ; "::t;;i ;i'":'i"";;"i ;;;i;ti iiot ;ilr[#L ;;;i;;

ffi'#ii;i;il;il,lli""oot,

Adapted from To

Et is

Human by Lewis Thomas

/&

From paragraphs 1 and 2, why would one be surpdsed at the examples of computer error? Use your own words as far as [2m]

possible.

2. 3. 4. 5.
a) b)

From paragraphs 3 and 4, how are computers and humans different in the way they 'think'? Use your own words as far as possible. [4m] From paragraph 7, Why do we always say'trial and error'and not'trial and rightness'? Use your own words as far as possible. [2m] Explain the paradox (apparent contradiction) in the sentence, 'What is needed, for progress to be made, is the move based on the error.' (lines 41-42) [2m] Explain the meaning ofthe following phrases in your own words as far as possible.

'We are built to make mistakes, coded for errof (line 28) ll ml 'give the computers their heads' (line [1m]

68)

6.

Why is making mistakes beneficial to society? Summarise in no more than 130 words, using material from paragraphs 6 to12. Use your own words as far as possible. [6m]

7. 8.

From paragraphs 12 and 13, why does the writer draw attention to the 'absolute
infallibility' of 'lower animals' (line 6'1-62)? Use your own words as far as possible. [2m] From paragraph 14, what is needed for mankind to move ahead (line 77)? Use your own words as far as possible.[3m]

e. ervc
[5m]

w rc vr u rv ruluwl r9 wvr us ds rEy drv ulcu nr urc PdJJdgE r vu 'ldy your answer in one word or a short phrase.
u

re

'

Icd|r

19

(a) (b) (c) (d) (e)

blindest (line 8) embedded (line 24) tendency (line 47) endowments (line 49) stipulated(line50)

10. The writer argues in favour of the benefits of error-making. To what extent should your

society be more tolerant of people making mistakes? Justify your answer with reference to the ideas in the text and to your own ideas and experience. iTml

tq

CJG Mid-Year 07 Paper 2 Answer Scheme

'L From paragraphs I and 2, why, according to the writer, would one be surprised at the examples of computer eiroi? UsG ygua own words as tai as possible. [21

Lifted
lvlistakes are not believed to be part of the normal behaviour of a qood machine. If things go wrong, it must be a personal, human error.

Possible reDhrase Computers are not expected/ lt is typical of comDuters to make anv blunders / slips. I1l lf errors were made, it would be assumed that it is a result of human incompetence,/ miscalculation/ oversioht. Ill Bonus mark The computer errors described are of an extreme nature. / Computers should be efficient but enors are almost absurd, bizarre in nature. Ill

they'think'? Use your own words as far as possible.

2. From paragraphs 3 and 4, how are humans and computers different in the ways
t41

A good computer can think clearly and quickly enough to beat you at chess,

Possible reohlase a) Computers can think more logically / supercede humans while still maintaining the essence oi human thought. ['l]
b) Computers can also work faster process ideas faster. [1]

Computers have improved functions. 0m


NB: Answers for a) and b) must be pniasec! in inc COmF,aiai;ve ioiin, ic [' s: logically, fastg!, etc in order to get the full mark. Answers without the comparative (eg computers are logical and think fast) 0m. Computers are programmed to respond in a mechanical manner and lack awareness.

It is not yet known whether a computer has its own consciousness, and it would be hard to find out... (it is easy to imagine that the faint distant noises are the sound ofthinking.)

t1l
Human beings on the other hand have genuine cognitive and imaginative capacities. ['1]

But realthinking and dreaming are other matters

why do we, according to the writer in paragraph 7, always say'trial and erro/ and not'trial and rightness'? Use your own words as tar as t2l
3.

possible.

J6

Possible reohrase Lifted it ls We use that expression as ii captures most life, the way in real that is, -.because accurately what we experience when we done.
try to accomplish or achieve something. [1]

Unacceptable lift: real life Reality [1/2m] Part of life Ilml We don't often get it correcu succeed on our first attempt. OR We become more proficienU competenV skilled through experimentation and unsuccessful attempts. I1l

4. Explain the paradox (apparent contradiction) in the sentence' '\lvhat is needed; 121 for progress to be made, is the move based on the error'' (lines

4142)

Lifted for progress to be made

the move based on the error

Possible rephrase It would be expected that to advance/improve, no mistakes would have been made / or one would only move forward. not back. l1l lnstead what is implied here is that advancements are made only after a fault or blunder has been committed. [1]

5, Explain in your own words as far as possible what the

writer means by the

phrases:
a) "We are built to make mistakes, coded for

erlor" (line

28)

tl]

programmed to Human beings are engineered/ created and predisposed/ biologically

commit blunders.
Not perfect / imperfect by nature [0m] b) give the computers their heads (line

68)

ttl

-Computers should be given autonomy/ allowed independent functioning/ operations

I,

Vocabulary:
1m -Purely by chance/ luck. 0.5m

t51

0m

Blrndest (line 8)

-Entirely unintentional

-Totally lacking in perception / judgement / reason/foresight - unexpected / unforeseen / unpredictable

accidental

Embedded (line 24)

Deeply entrenched

hidden 2

rooted

Tendency (line 47)

lnclination Propensity Naturalleaning SubconsciouslY favouf

Endowments (line 49)

cifts
Natural capacities Natural abilities

Stipulated (line 50)

Dictated/ sPecified/ laid down/ set down as an essential part Set down explicitly

S'ummarise 6. Why can making mistakes be beneficial to society? your own words as far as Use 6 to paragraphs 130 words, using materialfrom

in no more than

l2

possible.
l\rlGtirkes are at the very base of human

t6l

ihought... (line 24)


were not proviOed with the knack of being wrong, we could never get anything useful done. (lines 25-26)
rt we

Possible rePhrase errors are the foundation of human cognitive activity/ critical reflection [1]
We could nol achieve anything or engage in any productive activity. [1]

We tearn, as we saY, bY'trial and error' . in real life, the way it is done. (lines 29-31) Almost everything is done flawlessly. something screwed up, and then the action can begin (lines 33-38) Tfre miweading is not the important the new error; it opens the (lines 39-40) finding... OR The capacity to leap actoss mountains of rnformatron to land ....highest of human endowments (lines 49-50)

We neconre wiserl more informed/ make better judgements by making errors ['1]
unacceDtable lift: learn

tvtGtaGiire

catatvsv impetus for

chanqe, lest we become too complacenv mech-anical/ seemingly perfect [1]


istakes allow us to break new conceptual ground/ have new perspectives/ dlscovenes / lnnovation and lll

way

f,at is neeoed, for Progress to be made, ;s the move based on the error' (lines 41-42)

io

mike advancements based on these

new perspectives/ discoveries. ['1] Unacceptable lift: progress' error one Dersoective against the

ihere has to be an arqument

l1

beforehand ({ine 44)... there can be no action at all if there are not the two sides, and the argument.. (lines 45-46)
8.

other, it leads paradoxically to a creative tension. [1]

9.

the richness of selection in such Mistakes give us diverse and manifold situations can lift us onto totally new options ['l] ground. (lines 55) lf we had only a single center in our without which we become stagnanv brains, capable of responding only intellectually static.[1] when a correct decision was to be made, instead of the jumble of different, credulous, easily conned clusters of neurones that provide for being flung off into blind alleys, up trees, down dead ends, out inio blue sky, along wrong turnings, around bends, we could only stay the way we are today, stuck fast. (lines 57-60)

7. From paragraphs 12 and 13, why does the writer draw attention to the ,absolute infallibility' of 'lower animals' in line 62? Use your own words as far as possible.
t21

Lifted
lndividual cells in a tissue are mindless machines, perfect jn their performance, as absolutely inhuman as bees Sometimes there are ten, even tlventy difFerent ways to go, all but one bound to be wrong, and the richness of selection i; such situations can lift us onto totally new ground. This process is called exploration and is based on human fallibiliiv

Possible rephrase -Animals do not make errors because -their actions are biologically/ genetica y predetermined / diciated bv instinct. I1l -Human beings on the other hand have the capacity for reflective choice which could resLrlt in making errors.[1]
Acceptable lift: lower animals

move ahead (line 77)? Use your own words as far as

8. F.om paragraph 14, according to the writer, what is needed for mankind

possible.

to
t31

lvlankind would need to accept the unlimited array of faulty [1] but stimulating/ engaging options {llthat the computer can generate and pay less attention to the restricted number of errors that humans can anticipate. lf it is a big enough mistake, we could find lf the relevant computer error is ourselves on a new level, stunned, out in substantial/ massive enough, [1/2mj it the clear, ready to move again. would elevate us to the next paradigm/ plane/ intellectual horizon from which mankind can progress. I1/2ml Unacceptable lift: level 10. The writer argues in favour of the benefits of error-rnakingr To what extent should your society be more tolerant of people making mistakes? Justify youl

Lifted A set of wrong aiternatives much longer and more intercsting than the short list of mistaken courses that any of us can think up right now

Possible reohrase

answer with reference to the ideas in the text and to your own ideas and exPerience. [4
More tolerant: 'pts in summary are relevant but must be contextualized. *pt of discrim bet good and bad students diflerentiate bet mere blunders and productive ertors. *compadson with other more tolerant societies is in order. Singapore has not tapped the potential of making productive errors - essentially intolerant of genuine errors which we either condemn or draw into mainstream culture. -Would encourage entrepreneurship and risk-taking. -Creativity vs conformity (following a template) -Gracious and accepting society - allowing people who may not succeed initially to blossom. -stress levels would be reduced.

Egs Education - unforgiving and rigid. Employers - intolerant of efforc - employees become risk-averse and timid. Conversely, people who commit less errors become arrogant and complacent. Politics - repressive and uninviting.
- give us a right understanding oi being human

both limitations and potential

Lers talqrq!! -Security and order - there is little margin for error in these times (terroism, crime, political choices etc) - consequencesl -Excellence and efficiency are required in a city that wishes to go global. -Mistakes could be simply due to bad judgment / negligence - for eg, risktaking vs making wild decisions.

J+

lJc Mid-Yeor 07 Pqper I

'!.

To what extent does the Singapore education system meet the needs of society? Examine the validity of the statement that poverty is the parent of crime (Aristotle)

2.
3.

"Technological progress has caused society to regress in other ways " Discuss' Consider the implications of dishonesty behaviour among young people." Do you agree?

4.

5."Theworkingmotherphenomenoninmodernsocietyistherootcauseofanti-social

6.

"Sport does not build character. lt reveals character'" How far do you agree with this statement?

7. Would you consider Singapore successful in instilling in her people a sense of


belonging?

8. g.
'10.

Should animaltesting be banned?


"The weak believe in luck. The strong believe in cause and effect " Discuss Can the mass media be blamed for all the problems of the modern world?

1.

Does religion still have a place in our scientiflc world today?


"We continue to only harm the environment " ls this a fair assessment?

'12.

IJC Mid-Year 07 Paper

1. To

what extent does the Singapore education system meet the needs of society?

D-E Essav

. . . . .

Will detine the term "reeds of socie,y". For e9, Singapore needs to nurture good leaders, forward thinkers and entrepreneurs and she needs to establish an open and inclusive society and a civil society. Will define "education system" as formal education system and what it comprises (eg cufficulum, school structure, teachers, assessment etc). lvlay be ote"sided discussion highlighting that Singapore education system does meet or does not meet the needs of society. Will have limited exarnpres to suppod one's view.

c OR BETTER Essav
Will discuss how the Singapore education system is ablp to meet the needs of society but also give a baranced discussio, on how the Singapore education system fails to meet the needs of society. For example, Singapore's education system seems to adopt a more didactic approach to National Education which does not encourage the young to speak their mind and actively participate in forging a civil society. Will give varied and cunent examples to support the views. For eg, the Social-Emotional Learning Framework guides the establishment of school programmes for the holistic development of the child. Will hiqhlight current trends. For e9, attempts to diversify the education landscape to address and recognise the varying abilities of differcnt members of society and establish "peaks of excellence". Will evatuate whether all needs of society can be met by just an education system (in relation to Singapore education system) or are there other institutions (eg government and charity organisations) and processes that need to work hand in hand with the education system. For example, the family, just as much as the education system, plays a signiflcant part in the upbringing ofthe young to ensure that we have a clvil society

. . .

2. Examine the validity of the statement that poverty is the parent of c.ime (Aristotle). D-E Essav

. . . .

Ctatily whal "pouerty is the parent ot crime" means Recognise that the question presents an efretne view, that is to say that povedy is the root cause for crime and, hence, questions the accuracy of such a statemeni Discusses whether poverty is the cause ol crime, without analysing whether poverty is the

Superticiat examples will be given, mainly assuming and glossing over the causal relationship betvveen poverty and crime, without addressing the complex factors at wo* For eg, a student might explain that a poor person will be forced to steal to feed himself, without acknowledging that many developed societies have put in a welfare system that
provides a safety net for ihe impoverished and underprivileged.

root cause (parent) of crime.

C OR BETTER E59av

J6

Will understand and illustrate through examples lhal povedy can resull in different types of crime, ranging from those which are in response to impoverished conditions such as theft, prostitution and dealjng in drugs and those which are the indirect result of such condilions such as family and gang violence resulting from anger and frustration at being
deprived or marginalised. eyaruale whether poverty is the root cause (parent) of crime. There will be a comparison with other reasons for crimes such as greed, revenge, desire for power, in the name of religion and even for thrills, highlighting examples wfen crimes have taken place due to other reasons apatT from poverty, such as the NKF Saga corruption (due to greed). Will have a good mix ol global examptes, comparing developed and underdeveloped countries, to illustrate the arguments crafted. For eg, student might highlight that rn poorer countries like Laos, Vietnam and Thailand, there is a high rate of child and female prostitution due to poverty. At the same time, in developed countries like Japan, some teenage girls prostitute themselves not because of pove.ty, but so as to support their extravagant lfestyles Witl pay attention to the keyword "validity" and "parent" and analyse whether lhe stafemena is a realistic description of present society at large orjusl a generalisation.

Wll

3. "Technological progress has caused society D-E Essav

to regress in other ways." Discuss.

. . . . .

Will clariry the term "regress" and "technological progress'. Will recognise that "regress" presents a negative view. Will rest/ict the discussion to only effects of technological progress on society and not examrne whether the eflects have led to regression or not W;ll provide a one.sided point of viewwhere studenl willexamine onty how technological progress has caused society to regress. Will give ,imiled and out-dated examples.

C OR BETTER Essav

Will recogrise that the question already assumes that there is progress in society due to tec h nol ogical p ro g ress Will explain and evaluale trow technological developments may lead to deterioration or progress and examine the effects of "technological progress" on society from diferer( perspectives (eg sgcial, political, economic, environmental, medical) For eg, communications technology like the lnternet expands markets for businesses but, because there is no central control of the lnternet and limited means of censoring its materials, it is often exploited by paedophiles who post more than 20000 images of child pornography online daily. Will provide a Mde range of examples lo supporl one's views. Will provide a balanced discussio, on how technological progress has led to progress and regression.

4. Consider the implications of dishonesty. D-E Essav

Will define "dishonesty".

el

. .

Will describe the implications of dishonesiy, for eg, how it can destroy ones' relationship, working life and personal life. Will provide limited examples, that is, restricting the discussion to a personal perspective or providing only examples from Singapore.

C OR BETTER Essav

. .

. .

Will disc{rss the implicalions of dishonesty on a micro (individual) and macrc (global) perspective. On a micro level, the student would discuss dishonesty, for e.9., in tems of slealing, in relationships, in workjng life and consider the implications ofthis dishonesty. On a macro level, the student would discuss dishonesty, for eg, in foreign relations, global politics, global business, international sports arena and the implications of it. Will evaluate the ex,ent (scale and duration) ofthe repercussions of dishonesty. Will ,llusfrate through current examples the implications of dishonesty, for eg, NKF T.T. Durai's dishonesty has ruined his career, bankrupted him and caused him to be a pariah in society and, for eg, the doping scandal involving French cyclist Floyd Landis who tested positive for synthetic testosterone and recent admissions by Lance Armstrong's former team-mates that they had taken the used the banned endurance-boosting drug EPO in preparation for the 1999lour have undermined the credibility of the sport. Will explain the impodance of honesty Will be able to point out that sometimes the implications of dishonesty may be posftiye.

5. "The working mother phenomenon in modern society is the root cause of anti-social behaviour among young people." Do you agree?
D-E Essav

. . . . . . . . . .

Will exprai, the phrase "working mother phenomenon". Will discuss types of anti-social behaviour ranging frorn the deviant to the criminal (For eg, promiscujty, prostitution, school bullying, vandalism, being inebriated). Will have a few outdated examples, ignoring the time frame of the question, i.e. "modern society". Will recognise that the question presents ar extrcme view since the question reads as "working mother.....root cause of anti-social behaviour... " Will discuss how the working mother phenomenon adversely affects the young, wilhout anatysing whether it is the root cause of anli-social behaviour among the young. Will p/ovide limited examples of anti-sociai behaviour among young people.

C OR BETTER Essav Will pay atlention to the keyword "root cause" and analyse wfefrrer the sta,emert is an

ovef-genefalisation. Will evaluale and have a good variety of rccent examples evaluating whether working
mother phenomenon is the root cause of antisoaial behaviour among young people. Will offer a balanced discussio, by comparing olher causes of anti-social behaviour among the young with the working mother phenomenon ta delemine the root cause or whether it is a combination of several faclors. For eg, peer pressure, stress trom school and society and exposure to the other sources of influence through the mass mediaWill recognise that there are alternaive caregivers who may be just as effective in fulfilling the roles and responsibilities of the mother such that the blame cannot be pinned on the absence of lhe mother.

J'

May attempt to highlight that though there are other causes for anti-social behaviour among the young, the working molher phenomenon mighl have aggtavated the situation. Fot eg, the mother's absence can result in a child looking for guidance and company elsewhere.

6. "Sport does not build character. lt reveals charactel." How far do you agree with this statement?
D-E Essav

. . . . . . . . . .

Will attempf to defrne the key phrase "charactel' in terms of qualities that one has such as resilience, determination and diligence. Will restricl the discussion and examples Io behaviour and, hence, taits displayed during the course of engaging in the spor7. Will explain the causal relationship between sport and the development and revelation of character. Will fimit discussion to sport not building character but rather revealing character. ln other words, the student would not be discussing the other perspectives that sports builds as well as reveals character or that sport builds character but does not reveal character. Will have a limifed range of examples from the same spoti (For eg soccer).

C OR BETTER Essav Will point out that "character" in the first sense refers to poiitive traits while "character" in the second sense may refer to both positive and negative araits. Will attempl to give a balanced discussion, illustrating how sport also builds character. WiIt discrss how it is both a nature and nurture issue. Wlll ustate argurrents using a good variety of rccent examples in the international spofting afena. l,4ay highlight a lotally different perspective that sport, in some cases, does nol necessariiy build nor reveal character such as in entertainrnent sporis, as in ihe case of the popular wrestling show, World Wrestling Entertainment, where some of the action is staged and wrestlers have to take on the role of a hero or villain at difletent times.

7. Would you consider Singapore successful in instilling in her people a sense ot


belonging?
D-E Essav Will laryely describe what Singapore has done to instil in her people a sense of belonging. Will have miaimat drscussion on the keyrvord Sr.rccessful" Will give limiled and mundane examples such as singing the national anthem and saying the pledge in schools, or giossing over National Education in schools. Will have a narrcw scope of discussion, focussing mainly on schools, paying little or no a{tention to other groups

. . . .

C or BETTER Essav

. .

Will discuss what


20061

means to have a sense of belonging, especially to a small cosmopolitan city like Singapore for instance, (a) Feel passionately about some aspect of our country (b) Get together, do something for the community lNational Day Rally Speech

it

Will evaluate the extenf ofsuccess in instilling in her people a sense of belonging
1

A wide range ol recerl examples to subslantiate their arguments is necessary for them to do weli in this quesfion. (For eg, the overseas Singaporean Unit established in 2006 to reach out to Singaporeans livjng abroad, the National Youth Forum in 2004 and Youth Workgroups set up by the Ministry of Community Development, Youth and Sports to involve young people in issues relating to community involvement, national engagement,
opportunities for all and youth entrepreneulship)

Will go beyond discussing the successes/ llaws of the National Education programme in schools and look at other ways in which the government instils a sense of belonging to Singapore: recogrr:se that heattware is not just something we do l', sctools; it is also about getting all Singaporeans to engage and padicipate in shaping the character and life
of our society.

8. Should animal

testing be banned?

. . . . . . . . .

D-E Essav

Will take an ernotiora, stand, denouncing animal testing as cruel and totally unnecessary, condemn supporters of animaltesling - rack barance Will have rimited discussior onwhy animaltesting should be banned - reasons may not be always sound or welhargued. Will only put forth reasons against animal testing bul wrl not be able to suggest atternalives and just insist plainly on banning animal testing.

C or BETTER Essav

Will be objective in their argument, putting forth both the benefils and drawbacks ol
anrmaltesting. Will recognise that the word "should" reflects lhat a moral dimension ought to be addressed by evaluating the morality of lhe motive, the ac{ itself and the consequences that arise. Will d,btr'guish lhe different purposes of testirg (for eg, medicine, cosmetics) and Will also be able to give a wide range ol examples of types of animal tesfing (for egi LD50, Draize eye test, skin irritancy test) and evaluafe whether these forms of testing are absolutely necessary. Will discuss some araernatives to animal lesarrg (for eg, in-vitro te6ts, computer software, dalabases of tests already done to avoid duplicalion)and cite some examples of some companies which have stafted to adopt these alternative measures (for eg, Body Shop, L'Oreal). May divide the animals lnto groups - they may perceive that it is more inhumane to conduct tests some animals like chimpanzees and dogs compared to rodents.

9. "The weak believe in luck. The strong believe in cause and eflect." Discuss.

. . .

D-E Essav

Will attempt to explore the truth of the statement though will not always be able to pin down what they consider "weak" and "strong". Ihe key phrase "cause and effect" may not be properly understood or explained. Will attempt to bring up a couple of exceptions for balance.

jD

c or BETTER Essav

linked .- w,ri-r oe ,ol" to po int out what the key phrases mean and exptain how they are ;iiirg'l and self-belief of independence' idea ind "cause and etfeci" rcfet lo lhe i.". 'o"ing;

of one's own destiny by imposing a rational approach to ?nalysing and one's circumstances, while "weak" and "luck" rcfer to the idea of helplessness' maniging";ntrof uncerLiniy and allowing oneself to be subjected to the vagaries of life' of Will exptain why suci a claim is made in lhe statement by giving a wide range ;strong" moves strategic or wellplanned who have put in efforl and made exampies-the will hardly afttibute their success to pure luck (For eg' if success form sot" to attlin a winning an election, economic success, having meaningful relationships and leading ofthose iurririini tit")-uno oy aiscussing the lrane otmind(ie whether they are weak) Deal" or No game shows.like."Deal participate in ;# ffit ;:D, Totoind Big Sw-eep and there whether evaluating present) and which aie based on luck with no cause and effect ii i "",r""t rctationship between one's frame of mind and one's attitude towards one's ability to control one's circumslances. Wtiquestio, tne a.sumptions reflected in the slatement lor the countetargumentby nigf'lijt'ii"g occasions where the slrorg seize the oPpolultilie: aflorded bv succeed in iJniaipiiou" incidents and determine causal factors that would help them he deduced a penicillin when of (For eg, Alexander Fleming's discovery in"ii petri dish a "nl""uort" relationship between the rnould that accidentally contaminated lausal ionLining , St+nylo"iccus bacteria culture and the subsequent elimination of the bacteria ) ir," .rtirni , emselves demonstate a belief in luck (For eg, highly skilled and oi

"n"n soccer p-layers who believe in wearing their lucky jersey number') successful

10. Can the mass media be blamed for all the problems of the modern world? D-E Essav Witt attempt to

. . . . .

tist some of the problems of the modem world presents Will pay attention lo the keyword "atl" and recognise that the question

an

extreme view . media Will address the qu eslion tty comparing a variety of factots with the mass question i'e "modern of the frame [,4ay nave a few outdated'examples, ignoring the time wo.ld" superticial exampres will be given, mainlf glossing over the disadvantages the mass media bring about without rnuch analysis

C or BETTER Essav

. . .

Witt

giue- Oro"a sp eclrum of Problems llte modem world faces tyPes of mass Will iave a good va riety of recenl examples that refer to lhe different
media.

Will offer Oi""rii"O

r"Oi"

problems a balanced discussio, by comparing other causes of these or if the mass t" evaluate whether it is a combination of sevenl factors can or should be

. .

""n blamed.

ttrfv be blamed for the problems and to what extenl they

the be able to provide balance by having examples of problems not caused by Witt problems "lro media and show how the mass media help alleviate these mass case of whether or Will be able to rec ognise that sometimes lhere may not be a cleal

(for eg' while we are now more not the mass mediishould be blamed for the problems face of terrorist threats' the terrorists aware about precautionary measures to take in the perpetuate fear') have also indiiectly made use ofthe mass media to

3l

May recogrise lh at the mass media may be a convenient scapegoaffor societies to pin all ihe problems on so that the source of the problems can be kept concealed or they can absolve themselves from blame.

'11. Does

religion still have a place in our scientific world today?

D-E Essav

. .

\ /ill be able to recognise that the science and religion may potentially be conflicting because of their fundamental assumptions about otigins and reality and the values
that they subscribe to. Will adopt lhe narrow view that dichotomises science and religion' that is' the scientific persp""iive is based on empirical evidence and is' hence, objective, whereas religion is

that data are where word scientific subjectivism/relativism is a typicat leature of the

Lased on faith and

is thus more subjective, without acknowledging

. .

always subject to manipulation and interpretation Will confine argument lo mainly a discussion about terron3m Wilt give rinited exatnpres (For eg, focus mainly on 911 bombings' London bombing on 7 July, draw a simplistic conclusion about the importance of religion from the fact that places of worship still exist without being able to elaborate the example )

. .

C or BETTER Essav will recogni"u that the statement assumes thai science has come to dominate the wcid and there is an imptied assumplion lhal rcligion does not have a place in our world today because scierce ard religion arc mutually exclusive. Will be able to point out that religion is able to meet certain needs in our lives that science may not be abte to (for eg, psychological function; dealinq with death, suffering, fear, anxiety; makes lhe world comprehensiblel assigning meaning to events and providing a framework for interpreting events that seem randomly disconnected; provides meaning and purpose in life; trans;ending the mundane to look beyond everyday events and attempting to understand them in a metaphysical framework, ior eg, interpreting life cycle events such as birth, adulthood, marriage, death; helps individuals adjust to changes in iheir livas; prcvides c framework for the behavioural norms, that is, how one should act and behave both as an individual and as part of society ) Will be able to.ecognisethat retigion sfirl is a sersdrve issue/ held in high rcgard in our world today. Willdefinitely be ab,e to point out that religion is a sanctuary and a source of identity for people living in a world that is undergoing rapid change and, in situations when that sanctuary and identity are challenged, it leads to conflict that, in extrerne cases' manifesis itself in acts of terrorism. Will give a wide range of globat examples lo Provide balance (fot eg, ihe influence religi6n has over legll systems (e.g. Sharia Law) in l\4uslim counlries, the increasing nuriber of people embracing Buddhist philosophy as a means of coping with the slress of modern living and to counler materialistic values, the growth of evangelical Christianity as people seek-meaning in life and the atternpts to marry science and religion through the iheory of lntelligent besign. All these have taken place despite the advancements in
science).

12. "We continue to only halm the environmenl'" ls this a fair assessment? D-E Essav Will be able to point out that "otly" is an exteme word. Will clarry and explain in some detail lhe ham we have done io the environment as well as what we are stit doing to ham the environment ("continue")

. .

. .

Will provide balance by pointing out what we have done to protect lhe environment andlor lhe efforts to minimise the harm Will provide outdated or a limited range of examples lo illustrate the argument crafted (For eg, may give mundane examples such as the 3Rs Reduce, Recycle and Reuse, saving eleckicity/ water at the individual level or discuss deforestation and other forms of pollution without being able to give details.)

C or BETTER Essav

. .

in great detail using recent and the consequences which we to the envjronment harm we have done exarnples of the (Refer in The Straits Times dated to examples are already facing and will face in the future. 27 Juoe -29 June 2007.) Will have a good mix ol recent global examples (For eg, global warming and its impact on the local climate e.g extreme weather patterns such as flooding in Australia and heat waves in lndia, destruction of habitats due to pollution or the logging and farming industries which lead to dwindling animal populations and marine life.) Will atso be able to list some ol lhe rccenl efforfs (For eg, Live Earth concert) to save the Eafth and evaluate the success ofthese measures vis-d-vis the harm done.)
Will be able lo explain and illuslf,al9 their argument

:3

UC Mld-Yeor 07 Pqper 2

Secular Humanism is Hamtul

John Gray writes.....

Of all the myths spawned by the Enlightenmen{, the idea that we live in a secular age is

the most absurd. Throughout much of the wodd, religion is thriving with undiminished

vitality. Where believers are in the minority, as they are in Britain today, traditional faiths have been replaced by iiberal humanism2, which is now established as the unthinking creed of conventional people. Yet liberal humanism is itself very obviously a religion-a 5 shoddy derivative of Christian faith. lf this is not recognised, it is because religion has been repressed from consciousness in the way that sexuality was repressed in Victorian tames. Now as then, the result is not that the need disappears, but rather that it returns in bizaffe and perverse forms. Secular societies may imagine tlFy are post-religious' but 10 actually they are ruled by repressed

religion.

Liberal humanism inherits several key Christian beliefs-above all, the belief that humans are categorically different from all other animals. According to humanists' humans are unique in that, using the power over nature given them by science' they can
create a world better than any that has existed before. ln this view, the earth as simply a mass of resources for human use, and the other animals with which we share it have no 15 value in themselves. Those who hold to this view of things see themselves as toughminded scientific realists, but in fact they are in the grip of one of the worst legacies of Christianity. The humanist view of the earth as an instrument of human purpose as a securar rendnron of the biblical myth of Genesis.

The role of hollowed-out versions of Christian myth in humanist thought is particularly 20 clear in the case of lvlarxism. Marx's absurd idea of'lhe end of history", in which communism triumphs and destructive conflict then vanishes from the world, is transparently a secular mutation of Christian beliefs. The same is true of Francis Fukuyama's equally preposterous belief in universal salvation through "global democratic 25 capitalism". ln both cases, what we have is myth masquerading as

science.

Ihe trouble with secular myths is that they are frequently more harmful than the

real thing. ln traditional Chrislianity, the apocalyptic impulse was restrained by the insight that human beings are ineradicably flawed. ln the secular religions that flowed from Christianity, this insight was lost. The result has been a form of tyranny, new in history, that commits vast crimes in the pursuit of heaven on

ebrth.

30

The Enlightenment refers to the historical intellectual movenenl

it

the 18d Century which advocatd rc!!94 or

rationality as the primary basis of authorit.


god. '?Liberal or secular humanism is the philosophy tbat rejecrs th existence ofa

3+

The role of humanist thought in shaping the past century's worst regimes is easily demonstrable, but it is passed over, or denied, by those who harp on about the crimes of religion. Yet the mass murders of the 20th century were not perpetrated by some latterday version of the Spanish lnquisition3. They were done by atheist regimes in the service of Enlightenment ideals of progress. Stalin and l\4ao were not believers in original sin. 35 Even Hitler, who despised Enlightenment values of equality and freedom, shared the Enlightenment faith thal a new world could be created by human will. Each of these tyrants imagined that the human condition could be transformed through the use of
science.

The irony of secular culiures is that ihey are ruled by myths. lt is a commonplace that science has dlsplaced religion. What is less often noted is that science has become a vehicle for needs that are indisputably re{igious. Like religion in the past, though less efiectively, science offers meaning and hope. ln politics, improvement is fragmentary and reversible. In science, the growth of knowledge is cumulative and now seemingly unstoppable. Science gives a sensation of progress that politics cannot deliver. lt is an iliusion, but that in no way diminishes its power. We may live in a post-Christian culture, but the idea of providence has not disappeared. People still need to believe that a benign pattern can be glimpsed in the chaos of human evenis.

40

45

The need for religion appears to be hard-wired in the human animal. Certainly the behaviour of secular humanists supports this hypothesis. Atheists are usually just as 50 emotionaliy engaged as believers. Quite commonly, they are more intellectually rigid. No
doubt there are many reasons for this state of affairs, but I suspect it is the repression of the religious irnpulse that explains the obsessive rigidity of secular thouqht.

Liberal humanisis repress religious experience-in themselves and others-in much the way that sexuality was repressed in the straiFlaced societies of the past. ln secular cultures, religion is buried in the unconscious, only to reappear-as sex did among the Victorians-in grotesque and illicit forms. lf, as some claim, the Victorians covered piano legs in a vain effort to exorcise sex from their lives, secular humanists behave similarly when they condemn religion as irrational. lt seems not to have occurred to them to ask where it comes from. History and anthropology show it to be a species-wide phenomenon. There is no morc reason to think that we will cease to be religious animals
than there is to think we will some day be asexual.

bU

liberal humanist thinkers were adamant that religion would die out with the advance That has not come aboul, and there is not the remotest prospect of it happening in the foreseeable future. Yet the idea that religion can be eradicated from human life remains an anxiously defended article of faith among secular humanists. As secular ideology is dumped throughout the world, they are left disoriented and gaM/ping
l\,4any

of, science.

65

The Spanish Inquisitjon was a religious tribunal esiablishcd in orthodoxy jn Spain. It was notorious for ils use oflorture.

lll?!

by the Spanish monarchyto maintain Calholic

3/

10 lt is this painful inability to reconcile

their expectation with the reality around them, I believe, that accounts for the peculiar rancor and intolerance of many secular thinkers. Unable to account for the irrepressible vitality of religioh, they can react only with puratanical horror and stigmatise it as irrational. Yet the truth is that if religion is irrational, so is the human animal. As is shown by the behaviour of humanists, this is never more so than when it imagines itself to be ruled by reason.
Here we have the paradox of secularism. Secular societies believe they have left religion

70

1,1

behind, when all they have done is substitute one set of myths for another- ln effect, liberal humanism has taken Christianity's unhappiest myth*the separation of humans from the rest of the natural world-and stripped it of the transcendental content that gave it meaning. ln so doing, it has left secular cultures such as Britain stuck between a humanist view of mankind that actually comes from religjon and a more genuinely scientific view in which it is just one animal species, no more capable of taking charge of its ciesiiny ihan any other....
12

75

80

Humanism is not an alternative to religious belief, but rather a degenerate and unwitting vercion of it. Among the many varieties of religious life that are thriving among us-Hindu and Buddhist, Jewish and Muslim, along with many new and hybrid traditions-this pale shadow of Christianity is surely an anomaly. Weighed down with fears and anxieties that the rest of us have never known or have long since left behind, it survives only as a remnant of a time when .eligion suppressed natural human impulses. We rnay not be far from a time when atheism will be seen as a relic of repression, like the frills that may once have been draped over piano legs.

85

13

)[

'1. Accordingtotheauthor,whyisitabsurdtoclaimthat'weliveinasecularage'(linel)?

1l

2. 3.

'the unthinking creed of conventional people' (lines 4-5)

What does this reveal about the autho/s attitude towards liberal humanjsm? [2]
What does the author intend you to understand by ending the second paragraph wfth three dots (...)?[1]

4.

rom paragraph 3, describe the two mutated versions of Christianity and exPlain how they are similar. Use your ow, words as taras possible[3]
5.

dentify the paradox in paragraph 4 and explain it.[2]


People still need to believe that a borigt pallem can be grimpsed in the chaos of human events.' (lines 4748). Explain what the author is saying by bringing out the meaning of the rtaLrcrsed words. [2] xplain the authols criticism of the secular humanists rn paragraih 1A. Use your as far as possrb/e. [2] 8. hat is the author implying by using the word 'animal' in the penultimate sentence of paragraPh
10?

own worcls

l2l

9.

ive the meaning of the following words as they are used in the passage. You may write your answer in one word or a short phrase.

. . (c) categorically (line 12) ........................


. (d) legacies (line
(e) vehicle (line 42)
17)
.

1) (b) bizarre (line 9)....


(a) spawned (line

................................

Ill

... .. .... ................ nl


.......................................111

............................ ..... t1l


t1l

tO. Using material from paragraphs 7 to I of the passage, summalise the author's reasons fol asserting that religion is an inherent need of human beings and his argumenl against the repression of religion. Wriie your summary in no more than 120 words not counting the opening wods which are printed below. Use yo ur awn wards as fat as possible. According to the author, one reason for human beings' inherent need for religion is... [7]

li

11

ohn Gray is confident lhat rellgion is a natural and healthy expression of Man s deepest needs How convincing are his arguments? ls your generation becoming more or less religious and do you regard this as broadiy beneficial or harmful?l8l

,r,

lJC2 Mid.Year Paoer 2 2OO7 Suqqested Answe.s


According to the author, why is it absurd to claim that'we live in a secular age'(line 1)? (1m +% m bonus)
From the oassaoe Throughout much of the world. religion is

Question 1:

Suooested answers ln mosl couniries, religion is glg.lgllg

thrivinq with

nd im in

ishedJilqliq

viqorously/ flourishinq/Drosoering (%
m) Note: grow ONLY (0m)

and continues to exisgstill exists purposef ullv/meaninqfullv OR has !b9Jig9!Ilg[9!g!!lb9:49!to survive.

(%n)

Bonus:
undiminished unabated (% m)

Question

2 'the unthinking creed of conventional people' (lines 4-5) What does this reveal about the author's attitude towards liberal humanism? (2m)

the unthinkinq creed of conventional people

The unretlective

ideoloqv/doctrine/svstem ot beliefs of conformists/individuals lacking in oriqinalitv. ('lm)


(Note: without thought -0m)

Nole: candidates must get ALL 3 poinG to obtain 1 mark. For I or 2 points, award % mark only.
The writer's attitude towards liberal humanism is one of

disdain/ contempt (1m) disapproval (% m)


Note: mockerv/sarcasm (% m)

Question

3 does the author intend you to understand by ending the second paragraph with What three dots (...)? (1m+ % m bonus) From the Dassage

The humanist view of the earth as an instrument of hunan purpose is a secular rendition of the biblical myth of Genesis....(line 19)

Suooested answers Since Genesis is ihe fi6t chapter of the E!!!g (% m bonus), the author wants us to understand there are other instances in !!qqj!b which illusirate man's manipulation of the earth. (1m)
OR

Any sensible answers to the effect of 'more to follow' (% m)


OR

The writer intended the reader to deduce the implications/draw his own conclusion for his comparison between liberal hLrmanisrn and chistianitv. i'1m)

From paragraph 3, describe the two mutated versions of Christianity and explain how they are similar. (2m) IJse your own wotds as far as possrbre.
From the

Question 4

Marx's absurd idea of "the end of history' , in which communism-t umPhs and destructive conflict then ygqisbglfrom the world, is transparently a secular mutation of Christian beliefs. (line 20-23)

The success of Marxism/collectivtsm/soclallsm wll bnng about the di9epp93l3!99".1b!d (% m) of devastating/damaginq/harmf ul

discord/struqqle/clash/tension/difference

(%n)
Note: for'destructive confliet', award (% m) only if the two words are simultaneously paraphrased

The same is true of Francis Fukuyama's eouallv Dreoosterous belief in universal galyelig! through "global democratic capitalism". (line 23-25)

Fukuyama's theory that the equal opportunity and freedom inherent in an international market-based /LaissezE3!r9-es9!94y (% m) will 93yq!!9Ih9!9 wotld. (% m)
Notei lor'universal salvation', if only 'salvation' is present, award 0m.

4a

ln both cases, what we have is

masque.adinq as science. (line

Eylb 25)

I ln both situations they are fiction/untruths I (% m) disquised/masked as obiective facls. (% mJ

Question 5
ldentify the paradox in paragraph 4 and explain it. (2m + %m bonus)
From fhe Dassaoe (a) % m bonus for explaining technique. A

paradox

an apparentlv self-contradictory

statement /idea which is nonetheless true/logical on closer examination


(b) % m for identifying the correct paradox from the text

'. . secular myths .. are frequently more harmfulthan the Jeal thins. (lines 26-27)

(c) Being essentially stories, myths are not taken seriously by those who consider them mere fictional creations- However, the inaccuracies and distortions inherent in myths can be more damaging than the truth. (2m)

oELurdr rEr9rerr \lrE zol

9! "'d' d,c wulu,y or unrelated to spiritual and religious affairs and it therefore seems contradictory to combine lhe word with "religion" in describing liberal humanism. However, liberal humanism - though a secular school of thought - can arguably be considered a 'religion' because its adherents resemble religious followers in

unqLrestioning faith, blind

obedience, dogmatism, intellectual belief in imposing man's will on nature (para 2) assertion of man's right to exploit natu.e (para 2) expectations of paradise and redemption, albeit in this world.

Any 1 of the above


(2m)

The word "heaven' with its implications of a torm of tyranny... that commits vast happiness and moral perfection seems crimes in lhe pursuit of heaven on eadh antitheticalto "tyranny" which is associated
...

(line 30)

with misery, coercaon and cruelty. However, it is indeed a historical paradox that diclalors have committed great atrocities in an attempt to impose their utopian vision on the world/to create their vision of an ideaupertect world (2 m) Note:

. .

Answers should be assessed holistibally lf marks are awarded for part {c), dpjgl!award marks for part {b)

Question 6
'People still need to believe that a h9 g!_831re!! can be gtt4pgggl in the 9tA9! of human events.'(lines 47-48). (2n) Explain what the author is saying by bringing out the meaning of the italicjzed words.
From the oassaoe Suooested answers 'People still need to believe that I Humans stillwish to believe that they can L see/disceln/perceive (% m) a qlimpsed be b9!ig!]j!9!!can in the gl!e99 of human I benevolenUkindlyffavourable/constructive/hope (lines I (% m) design/purpose/trend (% m) behind the confusion/utter disorder (% m) of human

47-48)

events.'

incidents/circumstances.

Question 7
Explain the author's criticism of the secular humanists in paragraph 10. lJse your own urords as lar as possible. (2m\ From the oassaoe It is this painful inability to reconcile theiexpectalion with the reality around them, I believe, that accounts for the peculiar rancour and intolerance of many secular

thinkels

Suooested answers They are just preiudiced against religion because they cannot explain/accept the fact that (understand why) religious fervour cannot be held bacldremains unabated. (1m)

4,

lJnable to account for the irrepressible vitality of religion, they can react only with puritanical horror and stigmatise it irrational. (lines

] They attack the supposed absurdity

of

68-71)

as

I religion and delude themselves most ] especially when they think that they are I governed by/directed by/ grounded in logic/good sense while (1m)
i
I

... this is never more so than when it rmagrnes {self to be ruled by


reason.(lines 72-73) (include this so that we can explain/justify to our students the paft on deluding themselves)

Question

I
By using the word 'animal'to describe the human, the author iS sayrng that humans are similar io animals (% m): that thev are ruled bv instinct (% m). He is implyins that it is_bs!!gli!g!!!gt_!el!el than reason ('lm) that explains mankind's need for religion.

What is the author implying by using the word'animal'in the penultimate sentence of paragraph '10? (2m) Yet the truth is thai if religion is irrational, so is the human animal...(line 72)

Ouestion

Word in contexl myths spa@ed by the Enlightenment (line 1)

+ Y2 m spread multiplied
Nole: generated (1m)

1 mark

Y, metk

LJJI
started began brought about

Any one of the % m answers suggested

caused spurred propelled ignited triggered

it returns in

biare

and perverse lorms ...


(l;ne 9)

very extremely

/'m

Lm
odd peculiar strange unusual

Any one of the 7, m answers suggested

extraordinary/ special

Note:

1r

weird (1m)
humans are

categoically
different from ... (line 12\ One of the worst legacies of Christianity (line 17)

undoubtedly indisputably undeniably irrefutably

certainlY

surely definitely absolutely

clearly obviously

A situation that exists as a result of things that happened at an earlier time generation/belief/ movement Consequences left behind by a previous generation/beliefl Movement
heritaqes medium means channel

Science has become a vehicle fot needs ... (line 42)

transporter

carier
tool platform vector mode lransmitter

Question 10

Using materialfrom paragraphs 7 io I of the passage, summarise the author's reasons that religion is an inherent need of human beings, and his argument against repressing religion.

Wdte your summary in no rnore than 120 words not counting the opening words which are printed below. Use your own words as tar as possible. [7]
According to the author, one reason for human beings' inherent need for religion is...

Pt no.
1

From passage

paraphrase that it is inqrained in our nature. [1]

The need for religion appears to be IAlglgilgd in the human animal. (line 49)

...the behaviout ol seculat humanists supoorts this hvpothesis. (line 50)

This theory/thesis/notion/concepgidea I%ml is dgEelell1blggEeelaDle l%ml by the actlons of the secu/ar h u m anists/at h e i st s...

4+

:th&re
(line 5'1)

morelllelllplgelvJglt!

who are inflexible in their thinkinqrwhose flame ol l-ejerence about the world is tixed

lll

tne glsglglyeiqiCjq -etpla-as secular thought. (line 53)

ot

nsi

h. this actually describes/makes clear/accentuates lhe

IY,ml

unc;mpromisinq/unvieldinq/i4fl exible dlY [7,m] nature of the secular/wo


thoughLtu/orldview.

cuttures, rerglon is hgligd in the unconscious, (lines 55-56)

r:lecutir

h{ietiesthat are non-rcligious, lhe


ieiit's uuc66-scio,rslpsvc tre lzml

need for belief is submerqed/concealed I%ml in the inner

of religion has ,,ilu to reappear.. in qrotesque and FoweverJhis suppression the ironic effect ot revivinq/.esurrectinq ''rriJit tor i'(ti-*s so-sz)

NB: 3 points here


the Victorians piano in a y3!-q-gfigd to legs covered 9&!9!g9j9! from their lives, (line

t%ml it in various unnatu ratta bnoroa!lb4E!9Cld!9!9l9C r/,mPnd 1ig1P1 1%m1 manners

lflas some claim,

Lut

as tne

pqlsin-ggfsg!

[%m] from our

daily experience is fq!!!g, l%ml

behave similarlv "eiulaihtimantsfs leliqion as condemn when they

iecunliunaniststameisls act in the


they [%rn] manner when

!IIe!!g!4i.

(iines 58-59)

criticise/denounce faith/belief/reliqion J%ml

Eigory and antnolology 9b9!

it to

be a sPecies-wide Phenomenon (lines 60-6'1)

Moreover, hisforY and anthropology/academic siudies have

provenlesiiblishedrdemonslrated [7'm] th=t thJ nee-dior rilgton rs a thorouqhlv/ fullv/human excerience OR somelhinq

@rpsr!9!999

t'l'tl

ihere s

no rnore reason to thlnk that we will cease to be religious enimals than there is to think we wlll 6mE diy be asexual. (lines 61-62)

itrerefore, the need for religlon ls as inteqral to our beinq human as our sexualitv. [1]
w-hilemany/,bera,humsnisf s/atheistswere resolved/unswervinqlunwavillnq tzmt iilfr6ii betief tnat religion would 'k^-;-- -vlin.r l%ml with the rise of the

Manv libera! humanist th,hkers were ada;ant lhat reliqion would llgiq! h the advance of science lines 63-64

4t'

scientific wotldview.

-ha@493p991,

and th.ere

is not the remotest ProsPe9! ot

lt

their exoectation has not been materialised/fulfilled [%m] and it may, time to come [%m]

;;fi;;6-b;;tin

point) Maximum 7 marks ('l ma* for each

Question

oJ Man s religion is a natural and healthy expressron -l*o'n'n-ii"v i" "onria"nt that becomlng generation his arguments? (R1) ls your 0""""i "i"i..-n"* "onvincing are (R3) 62fl-nful? o' us o'ouoti o"nlti";"1 more or less reliqious (nzt ano oo you req'aiJinis

11

BAND DESCRIPTOR9
1-3

fri
writer Those scriots with only a restatemenl ot the itt"*pt to tel;te to the students gene@lion

maln ideas

lMut

"nvl"o."

passage to begin the discussion ldentify at least one point from the

Tendstomerelvbeareferenceoftheviewsofthewriterbywayofrestahngthe
nuestion or summattzing or restating of the texl question or ivlisinierpreiaiion of ihe idea3 taised in lhs ln addition to the above, Tends to be superficial' with Evaluation is attempted but not always convincing. limited develoPment

of the issues, though not Exolanallon shows adequate level of understanding thorough in suppod and illustration
ln addiiion,

u".Jii;uin",no "Jt"i"
J,,itri

;il;:I;"
A verv good
illustration.

;;;;;;aluation

"no "lear irJljttion

conclusrons evatuahon. by devetoping ideas to logical

personal rnsght

anJ apt rllustratron

Greater

or poinls an-d treatment or content

support and apt understanding of the issues' with thorough

T{OTE:
'1

iiualnis muit
experiences

his arguments?' (R1), On the Question: 'How convincing are their own with evidence from the passage and suostantiate their stand

2.

On the Question: 'do you regard this as broadly beneficial or harmful?' (R3),

Students need to provide a balanced discussion of why being more or less religious has both benefrcial and harmful consequences.

3.

lvlarkers should not be expected to infer on behalf of the students RcsponGes should be backed by good ond relevant exampleg

4
5.

cood scripts should discuss if there is a need for religion beyq!!llhc-99!l!9!L.q]l


Sinqaoore

4+

JJC Mid-Year 07 Paper

1.

"The world is addicted to violence." Discuss the truth of this statement' Consider the effects of your country's move to establish a more cosmopolitan environment. History is a tool for politicians to achieve their ends. Do you agree? "The purpose of education is to make civilised men of us all" ls this view still relevant today?

2. 3.

4.
5.

The benefits of environmental conservation do not justify its cost Discuss "Good governance is the only reason why Singapore has succeeded " Discuss "Technological advancement has worsened the problem of poverty " Do you agree?
in our increasingly connected world " How far do you

6. 7.

B. "competition is inevitable
agree with this view?

9.

"Access to new media has empowered individuais, but weakened establishments." ls this a fair statement?

ro. Freedom and responsibility are inseparable. Discuss.


'!1. Ac'tion packed movies have undenieble appeal but

qu-'stioneb!e'.'slue Discuss'

12. Can discrimination ever have desirable consequences?

4t

P5 Q

--

Pb
E

gn -

P9
:E
&=

E.E

:;_F 3 E 0: : 3o
= el=o4=O{ >; qb-3 -; oDGi= I F

>i
i

ee
F

E q*E

H =

!e E -P3] irD:

3 9-3

K-

q= ;

r:-;Sg
-- E g o E 5 RE

3h sElE $r E9E o;6:;64: t=zn..oa rE.q


s

9;E

3e
_E

Eh

E E

3p
i6
a fr

o-

6: =s3 :9
F

.0

6.: b

6.e
EEt

; o;g - o>
-9
d_j

c= - {t;i a5 6 E E; > :d";"


-ro o6; 9o;

E, P-q -s

!;lE
F

E+IE

b-EP
gEq>
s

UF2

e 3 3 E* c

d !-n. 6 d=

g= ';F
n-9

-;8. E E ",9 FE-9::.9

3",9;
F

dPq ok6
9='rJ

izu

e4e z<1

; Jrt o s
F

Fdoii
tn .q

od<

p 6^
o5

)!o4S
q

=! ; ols

q
F

P9s

-Ef

5r:E EI6 ;eaa


o,
CE

ijp

,-

F{
g!:
^
e

:Ef
i9
dP
d-9

P l?; i ;i qe! 9o F F b

-s:6Y '= 6-,"


9bbb
oo9.

-:o ?.
.".9

qt

>F
PaPi,q :6 F " 96 5 o irl qro.eb e 6
E

n -6>E
a-U

'

.9

$; -: ed: d* 6 6,! i
ee

P!

E9i

qca6 =H bol q 0 >6 -

FegP

.9

.e9o^9o;r

6# i; e;
^i

4el

i!)

P o-=^=

3f;e -sE
*.E o9)

6 g: e3;'4 I -: E;t n6es$s-;Efi:!!fi5

>uq9ii

ob o,9 q E E q,5 -69 Eo'E

s!-9;R9;e-sfr=icE Ei:fr= j ei*,;t .gF;:5E fi'- i: v; s s!ssse $9tei;s;$


e93, E=F ii-^:Spr Eb"9'9= o?ic s9 ; 6E; P"-;P6P 9@ oF-qPJ?o =oad = E;=E:s'6 i= P:,HE> -j*5 ,:; E*-;r:g; fr:3:E i ei b:" o 9ra ; I -

--Fo69-1:;b"go.-

'

g39Eae P ePb q.,>EEIEipFe

PHHEg:ueHEEg F3!

e,ese;e;-:EE ;i3
:g*EeiE
.!oys.69,3*

l:eE;
.,E "

gn:;

at

ij9.90

+**ij;+{iF!!EIpp=
} 3 F;E 5 -: sg

s=-iiHrs EIc6l:b i5 s f P:::s 5 po af i 3 Ha i 0;:!'o-te>; t:;X:"

sl r;

Eir

?e *

H.#;9i b:J *g e
6r ^ "b;;-e;Fs' q t * LlE

o,
Po I qZ p

q ] \i F! 4 e F j:

{ pat P 9 E:e *'P b'i "a ;' . \-:- I Sdte E.q -s5:nA e 9 g=> eb-'.eF 3 ;,- E

i;

'9

., 9 >

.ii drr.f

oii

4*

RE> >

! O"= Egb s E; <Es


6: P-t

9 tc Hig''9i,r.*Etigt,:i : j o - o - I : i X :' o r 9 ' o,, - "^6c. 6i6 -djo --0 E="s, ; i I : I g ;':ir 9 i **=' I xs 3 Fri i rFi E!:E g g: ! e

4;o ;3=epu6

o::! g5:

i6bEi,,c;3

E!ur>o-"
9

ii!

H.9

r45 ii i;
E E

t
q-

.E
;!i

96

:,sP.E 2,.p 96 SE;3. d !

E dE

.90

c9

;9

.9

+; ' 5;: g{;c

: a't

ig

+E"sggs

to

'6

:^;st:i;!:4gEl n -! ;aP9.3E :tE

::+!ie:r";UbJHl.s i n'ii;c} :t i: f, n[;l ; : g{ es j E i ii; ";i i-p ri : o o' i Pii$ l!;;;Fs abt Fi: i;;
b5 o *

; j : i:

i ;:l: F fE l;fr F:;:;gi:e"

;- E;T

I -;
F*

I E d ;e -o = 3 63 q* 3E
".F 6ab
a :

r=i

S*

s;

*'r lE ?Ffi ii !:: g, :i,"8 *E EF;:;-i!: iH; 5.'= E;: e; [.] i e e--!* a I;:.6:1 pgfi ;i r:l![; ;:p;; e *ii5!; d g s{ l;'.:g; g 5;$E:51!: b::'Y $ !ii :r:tsis $O: ille g gEl 5i Pl i, ;tsi'BE i Jg tFSi lFiI:?
6 d ._ ?
H
", F

"

o,
;;o@

:;EE

ss1 q= d o

il;

qsE:91 31aF vq .::o;6: -: F s a 0E I i *Ee " 99! 6*

;;R

ibi. c Rb 6 I
d.s
F
E

e-*r,"eg e; *.; :i5 El .;E rd:;1" :;:el:;lE.I ;q;i ru"iq ErrEr FFA'co^*id!,ol ;!;3 ";;s;oEsr_i .; -oo=o I i : t : * ir t e : ',! i-: : I ; ; : g : ". r g 3 I i e I " ';6{-R:r !EiE",t gp;gu:gfrrgtiiiiil,r;"Ll #! o ipLp,i 4:; a? p g { r; S5r !f i?.; e 4: a p I95i3 ej ii E-e

gi.PA""6

r:;.
r bf

3,
;2
=

Ii::t

5,

C] !

aHF -i'c"; EgE


!, e; P E;5 5 3 3 a 5 E AE
.E

iP

i:-;o; qq3[g* pg'iE ud",o>o ;; tu +=,:iFP5;B:i=E -:9q9^"

1^

Ea
!;d

c <v, grU:ii, 5 :oqai

3.3s;:. ..9i#BFp";
o9-t=E-==

6>nr> ao esqF;Frqf-!o BU r... ?,[;E:f?$!i io,o-.o.=,

9s's.^ r3..o !'b rt:i : ;Ibq i-6 iE:a[:d

tb
P3

5::

: aoYoi

.gEi!!;g!F E;;r 69t


c>;:

5/

.e.
=

6F!g:
b

:hg" :
3p 3n

sl

6'

!;
o

'-q E; H: g r6 .,a399 g= '@ r E l 9Ea_ ; "i !l o- * A Pl" :=5"9;ga Eo' o.9e-t; E'-

g:

I $6

9r o Eo

E{'38
qq
?

;9E;

!!

.t 'a sESE E*:


#

se;:SE

-Pe. P;rC ;Ee *;; ;p s *; * s; e3E cE


o5
F
Y

;;cI

o"o-c

9^6:: o,
96
@:

!9=

ffiEi+aiiFiEgi;; }E
i=iilE !tbi!ii?E ;i;E 3iEi" g d,:,!e3sE5: 5,39+
ar:l-oYoojj oi uo
'o.>-

!b orfi " ".! as 9.b+P;ij,' HUi- F'l ,"Y:Pr-g

,":" .li^
;;6", ,,6E;

"4.i Fl6=

EiiEJ
3

"Ee
b
hi

.9

s,.icPh@o.

:'b

- ,,E
E

;iE:nrc;H;:A:i5rfiiFi
iff60cseFt3;i3g$;:ipF
-F;

? 6: !s90

o
a e EE: 9!>eotuE

9a sot s!h :: :-t ;pI


eE

t
! 9 o q5

l=
o

t;ils;Rds

tr ^t Pq 3i. e 9!iqPsIi= ,Pib'i i* tiEflt !3iii:'-E3g6LEI u: iE3e,a"E;EP EE;3=3:i b.'

.!

;i,EPP-oEft

3i 3 ter ; :::: "'[:

.9

oj

c.13ii

fl

BF qB
: Y c

a:

uFgI! ,3 g9 3:
.e
.9

qEiE

e EE o I' -E
q $ 6gs
E

os.pEP

eirc

;i; fesEl;,: s;i


ri! :EgIgiBI;IEEE g'Hn;3::3;: -E:;3s
iP;g;i sEggt#E!
,E.,i:e !q-e*t'.PE 6 9!:9? - F-:
dI: .,ooiroc2oo
3

; :E,PT

6.

.E -ts i

H! EEe; E Eil;F;H
.0,,Es5gE-E

41

;! r
t-

E;ialiup
9cob E P 3dt

Et

D-hnr@ !9d :F9, 6

6P EF

=o 6E
E

.:
J]>:-.

ll
n"

O P q

.9 ,. r 6tr o 6-

sadSPI
X:F;o.9 ,^!;! xt o)
F

q .. ts o - - t ao+ t:9o x o o I:;6 E EP

E
0l

Pe
o!

63
F
F

96
!E

!e#:9!
5' -' coo::
a

g:5

t?
l

+3
F

'6i
q
E"

46.;_3 $!
-9q! o; "

g
o

>

.9

i: b
1E

:i eE ".9
o 0 ":E

So
I
.9

!; cq 4 rc: Y c o! qeYP"4s @;. o-- o , : E o< d o

t
'6
q

o --

EIEi!

H;

>;6
F

3h5

q6o ;P q..e E e gE F g

!!6
l

d9
6 A C;:
5 E;
C1

i !e 3 s 0 c; $;
q

FE }

o6-E;^.

59

ab;

t3

b;3

-p E: l>co
4

p
A

614

ii

5:5:
d

o9

=g.'! $B c o o r;-

s:;
=

. P i. AE - o= 1.y '= ; oo
; ;i
-o; 9pb O:E

; r :; f,;; as: ie,c i ; :;#:;E=:5FEgC+t


q

'99-

tsi9P

"P ?:a

r')

(t;6 =

isi;e
X

ar
5

iaE;i:siit

35,
'

E{s!=sct5-EaP:5*:

$.e
g

tH =9q P s 1E{85.;--o: !:: F-9:ii: :66:,e609 r:'Fr 6 q 0 : ;".{3';!bE9 -? E a >E :: o oq c a Pg *E,'=".iS ! gEe;E aoioeii; 9 V;E F 3 PP F "BE .ii p8
9r

T:6=

m.

s3d:E

! gi

fi

!:

! 6
F

..

i; E;Ag;EEIFe i* t r*;*=EePt
F

="iE :, '-=i". s)!!

o h ! E h 5;

et;i;59Pi r:!peEsE

; ; e 9 3 X'5 ;; rp

3E,

g#$!*= HEE;Egr:lf

5e

'6> =6t9i:E > q: F 6 99E i:i I E.'-i ""


P-

.P

:
.9

o c h(1

!e

o *E

c!: =,

': -;

3!'

E()

;3;;9 EE_ii E6 ?tLt, i: 36P;.'


!:

F69 ido 9,:PoBP

o oa s* 9 E g .>s':'--E b

9; -:6 Y:ooo : ! 9.9 6:5oqF

o.91

Y.9.!.E; ]I;',s
o

PEf :f r 9: ;;=;3d =* FP:pi=s:

s4

] E F r ; ;ti,5 -,i.i- o: :Fi*i:[iriiPfi":[:in_ :cr

b er oI

;91
:o6
=

f sghEt :iE!,s; i fi 3*;s g;


F

i l. "

F< E e 3q d:, .: 9,6

E -A

:I

6E

b , t.. d E ! d
F

gsF

S: b ?9 E'
E

-o

i;
lc
l9

.gc

b:

.9

t;
JF

59 93 6o

sF

= .9!r"ElE

o-

*
$

:9
H

eR 0 9F;E

9-q
q

5o .a = E. !fr 9o -99
]F

6:r:: t-q-aiih
o,

a!

e gie Pi a i' ;!;;

I Eg!io
sE
F

iiN 3 E di-i ; I E FI

e9:

*S

Eb 3EE l6
F

9:;-9 fi--5i 9- t

:E cS e
!:69> I -o* e F 9,-i E de
F

o:: cA5!E
=

o,

gq
Pe.
6

s g=

E -6

I :
;+:

E:E o 6 ?: F
o

E;5 P!

EgEs*3 F .3PP:;=
9T E3#

I .. !EEgBrE
6.9

s:E; !!-3

F 5-e'g

q;'o. q o! 6!:q9;n,:"1

o " - o @ ni ^ i, ;9 e3be

3,rE!i
3P.

+E!3E I

;! P cE 50E3q
rE
Ug
q

q-!: i -:lo

'6

sTs 6i6

3 2e

!*

'6=

js
.)l

Fi!
b

zl

'

Prr nr & tl

Ql

i;=_

;
.9

99 :.;

E: P = e =9:
z

FPQPPQ

q-9:,
ur]

h g z'6aai,
L!,,,

oYl

Hi' *i !r*Fi:TTit
EE
o

". :;

-P.e

=3 EA! 9=q
o

:- Er o.:.o t o r ":E

.--<

i
6 t t

e 6 3 =,=* p!

9=

lE

! E;H;q; FEi !.9. ER eEd.:*Eq3EE33 igE"a:E{fEeFs

5i n:j99^ie-q;3;

F"^. 3q
9a
q

, L o!r ,4F u ttP. , j," l" {9!ee ^!4-

--"! ;.1

;i,d

=^

H::,:JE3;Pi'Lre34:8P 0:6o ijo o(o!oaq-(of o d,gisPS$ii:H:;:3F;i5

-6:-o , x6q'os5:"9g!,i; t;P 5i::[,E6r-E;o{

<-a,:

e! if E1 t=

::
=

a aEE:e ;e;eaE!

-,r

rPP9

;EPE Eg o E;
B

4 :3 E- !3s""i;. t E i; X$s s.f; .;e ;;Halil ebi; 5 .Es;6 gq,


t-9+ 6;l.q

qL

r ooLo c9o::Fr

l-

0?4

F:!F=gF

t;e:r
c

E*;:Ei;
I$

Es; 65: Ei;FfrE

.,-e :6 Ft

Et -p = o-P
^l

"E: i;Hr lf i ispg:r. eb!


E< o

c,5r :. q

50,06:cj. 9:qLdi-e

!S !

-:;_ 5o; d_L:9uo Sipi ea? Eio-.<P3 ;6 *s!: e;9 e:.3sq3, ; -rLsH=*r'i,IneiiHS 8"1H6*Xa E6t!iEri;Bg r<6<6r oT36t 6FUl 6:

t EE; P"EE
e

oq

EF;3 ::i E :ri:i*rhi


e : 3 ij " e Tzr^-

Eb,
6 >E a2E
-c 4d_c

aE Eg; f=-i i ! ttrEed i:38:;-". iEFtig r liE::i,uHfr ai*Hh:eH qlri.e -uQ;*;rs,

'o

b "oE.

E.ii i t.s* La: 9P Po,


og, ,

tsit s*;:*i

EoJ

'

-3p:iadiil ;ip\aEEp:i! 5 b! e,h 3[.t. ,3 S i 3:3 6< or':4' ti i


<tt
,

3 c ii':"

Et';::H-[E!
H

is '6= E{-995E
OE

;s gEpBE3
e dq;
p ! I 'io
B

g FeI f f E I E b:b Q e c 9:, p; E6 e 3 t,

o. b._;,;J .5r -:;;,o.i" gt ur:sri


; o

'i;

-o

;{6'F',: i'1 ?;:, - f {iol:i


ciFiiq-'i;q
if!

n....os: .s3*Bf
EL:JEq

BI.e B" :;_.9 ss",T 3;;';e acgS:6 eF9, I 69


E -:;

.,

E[;; 5-?.i3 - E9o. o

-t

.!

.9

!i E o B " ':::9* &3n r: r'ld:ocofz::.o

:6

qr o :e i to566 oY B g.r B E r6:9 oJo06@--. :(! oE'f -e 9 =geE',5sgi"- lx g, 5 h::" P;g eg-b9 -y EeltEHs':li;e=:eE t 2 si s q:6n q - o: 4 sz b h B;ec !! F E! 3Es,q Fqg*s =&R: ii PF;,lsPF*5E;;F39g ii9!6Y9cc EB 6I
dl

13

-9 oi!\o

!=:E ;E3?: Eco,q !:-or;,;4


:: E- 9:r:5
Ee

+n, e o qar = X'6!! : o

r P39 b
iib b; E: :dii!6:

e.P"

(s

;6s;:ieiiaE: 5 9 6c 6'
HE3;Es3g=3t
HEa

;3gPe*gl:"i6# 9od:os

* e;

'6=fsp
a ii*;

9*

q-9

o,

? g;C d
r
ai

hE
r" E

.9 .,

i2
F

fl;.'i; EEP:Et
n

= -=: ;Fi -> .e g E ",9 He:9

FEF

e;

9 9 e -s

ci1e:!9

'

rE -

6+atse. ;; Enii

3E
.9

3 3

'
,9

i!qi[".;:g i E 9 P'- - e P!
e .2

E. l? e 6;=;,4 ..;o;:,;tb5;
C

;E
9{9
doi -oc

orE

-9

.ooY! .:ort

E6qsEE*t; ;I:!6j::ou:!o

i;?E-

o,

.9

;.e o ii;

:bsll

.,:

Passage

Ruth Hubbard says...

This century has wilnessed holocausts of ethnic, racial, and religious exlerminalion in many parts of our planet, perpetuated by peoples of widely djfferent cultural and political affiliations and beliefs. lt is beyond comprehension that educaled persons can come forward to argue, as though in complele innocence and ignorance of our recent history, that nothing could be more interesting and worthwhile than to sort out lhe "racial" or "elhnic components of our species so as to ascedain our root idenlity. And where to look for that identity if not in our genes? Every decade or so, a new prophet promises lo decode the currently advertised Rosetta Stonel that will enable each of us to not only know who we tauly are, but to use that knowledge for our individual and collective benefit And, of course, no period has inspired more hope and determination than the present, now that large sums of money have gone into spelling out-"sequencinq"-the molecular composition of the "genes" which are part of that long string of'bases" curled up within the nucleus of each of the cells in our bodies. To make good on this, effort has been rendered all the more urgent by the fact that some of the scjentists pioneering the lask have suggested that the very volume and complexity of the new information make it virtually impossible to interpret- So, what to do? Why not use the intormation to tackle the meaning of'Jace," the question that has bedevilled Europeans and both European and African Americans for a very long time? And belter yet, why not use it to tackle the relationship of race to health, a question that haunts us in more recent times? Let us disregard the differences rn income, family slructure, education, rales of employmenl and incarceration, and such Let s gel down to the basics-our essential nature: our DNA (or "genes"). lt we could establish "racial" gene cluslers and if we could use these to predict disease, and so prevent il, wouldn't we all be better off?

t0

15

20

25

'4

Severalthings are wrong wilh this plan. For one thing, lhe manifestations of inheflteb conditions can vary considerably and unpredictably from one person to anolher and, indeed, in the same person at different times. This is so because many factors, both within and oulside ourselves, affecl lhe ways we develop and function This is true of our biological characteristics as well as our psychological and social ones. Even 30 conditions such as cystic fibrosis or sickle cell anemia, which foilow predictable patterns of inheritance, can exhibit a wide range of symptoms that differ in thetr severily in different people or, indeed, in the same person at different times When it comes to the more common and prevalent health conditions or diseases, such as the various cancers or the vascular conditions that can lead to hearl atlacks or strokes, genes do nol predict the facl or tjme of their occurrence or their severity with any degree of accuracy. For these sc{ts of conditions, a person's life circumstances, beginning al birth are betler predictors than their genes are This rs not to say thal "genes" aren t involved DNA is involved in everylhing that goes on rn our bodLes by virtue of the fact thal DNA specifies the composilion of proteins and differenl proteins are continuously being synthesized and participate in all our biologica{ functions. In fact, that's why DNA is important But the relationshrps belween our DNA and our proleins are neither simple nor one on one and they

35

40

rThe Rosella Slone ls an ancient stone wilh iexis engraved to honour lhe Egyplian pharaoh ti is commonty used as an idiomalic lerm lo suggesl a discovery ihal provdes knowledge ior lhe solvrng of a puzzle

36

change over lime. That is what makes genelic predictions problemalic and

rrnreliable

45

Furthermore, different degrees of disabiliiy are associated with different mutations and the degree of disability is different for different people as also for the same person al different times. Stranger yet, a mutation that elicits severe symploms in one person may elic'l no symptoms at all in others. No one yet understands why this is so except to assume that the relevant genes and proteins engage in multiple 50 interactions wlth other things going on within the organism as well as outside it.

The complexilies that emerge upon closer examination of such relatively well understood patterns of gene function make a mockery of the prelense thal we can use the worldwide distribution of the genes of human populations to shape our
understanding of our hislory or lo guide social

policy.

55

Adapled from Ruth Hubbard, "Race and genes'

Passage

Moore & wilson says...

Let's begin our slory with a pop quiz on genetic science. We have a group of follr men. lf you were able to look jusl at their genetic code, their DNA, which of these men have the most genetic differences? I have no question, given what we know about human genetics, that the tall person and the short person aIe gignificantly more different than lhe black man and the white man of a similar height The human qenome project is one in which scientists around the wodd are kying to map the location and function of every gene in the human body. The viewpoint of most geneticists today is that."race" has no real meaning in science. Trying to mix genetics with race is inappropiale, it cannol be done. Race is'something we do to each other: it has nothing to do with what our DNA does to us
lndeedp it's based genetically, but it's a very small part of us*How small? While each

10

cellin the human body has 100,000 genes, only about six genes control skin color _six out of 100,0001 What's more, everyone reading this article shares the same six genes, including lhe genes for dark skinl What that means may shock some people; that each of us has the potentialto produce skin as black as an African native. That is why geneticists say race, like beauty, is only skin deep. lf yoLl expect that there exlsts pure races, lhat is totally absurd Think of what people would look like without
skin. When you do that, you see we're all alike, we're prirnates.

15

So now that this information is out. peopie might accepl rt people mrght

not But you


20

can'i era6e what you've learned throughout your whole life. Like what about racial differences in sports? lsn't that genetic? Sorne people say black athleies have a genetic trait called 'fast-iwitch muscles " which allow thern to run, iump, and shool bel1er than whites. But science tells us that lots of athletes have fast-lwilch muscles, while nol all black people are good at sports. Why are Latinos good at soccer? Do we ever talk about their 'soccer" genes? ls there a "martial arts" gene for Asians? A

"golf'gene for whites? What about Tiger Woods? Did he gei his abilities from his
black falher or his Asian mother? And that's exactly the problem with trying to deilne people by race; there are no clear scientific categories that truly separate humans by the color of their skin. Bul that doesn't mean some people don't try

E '

So. f race rsn't based on scLence, what is it? The best evidence is lhat race is not in 30 our q"nes, rl's rn our heads, something we made r.lp a long lime ago ln the days when humans could see lhat the earth was flat, they could also see that people looked different. And though we now accepl that our eyes deceived us aboLlt the globe, the belief in racial differences has been tough to change

a -

Essentially, race rs like the dots on the screen. ln science, race means nothing The 35 dols mean nolhrng until we create the meaning, as we've done over human history' Race isn't science. Race isn'l biological So it is helpful to go back and use science to uncover the fact that there's no scientific basis for all the racial differeaces that we make such a big deal over. But we can't deny the fact that, even though a social construci, it has become a parl of our lives, our everyday lives And il stands in the 40 way of our being able to achieve all that we need to individually, collectively' or as a
nation

r '

The solutron to race relatlons may be to form relationships with one anolher. lt is absolulely recognized thal this partlcular friendship is diffictllt to create People have a whole pile of history that makes it difficult to create these kinds of relationships. Do 45 we throw up our hands and say it's dfficull, it won't happen? I think not
Adapted frcm Pam Moorc & Pete Wilson, "What is Race?"

6o

Questions on Passage

Fram Paragraph 1: Explain how it is ironic for "educaled persons lo wanl "lo sort oul the tacial' or'ethnic components of our species so as 1o ascertain our rool identily " (lines 3 - 6)
121

What does lhe author mean when she says "a new prophel promises 10 decode ihe currenlly adverlised Rosetta Stone..."? (lines I - 9) t2l
From Paragraphs 4 6: The wriier suggesis reasons why the knowledge of genetics cannot be relied on io predicl diseases. Summaflse the reasons provided from paragraphs 4 - 6 of Passage 1 in no more than 120 words, not counling ihe opening words given below Useyourown words as far as The ftrsl reason why genes shauld not be used ta predicl health isthal

From Paragraph 2:

[8]

Frcm Paragraph 7:

ldenlify the writeis attilude and explain how il is onveyed jn the phrase "make a mockery of the prelense..." (line 53) t2l Questions on Passage 2

From Paragraphs l &2: ihe writers' inlention in using ihe Pop Quiz. Explain .

I2l

'

From Paragraph 5: 'Why are Latinos good at soccer? ...A 'golf' gene for whiies?" (lines 24 - 27) Explain the writers'inlention in lhe use of rhelolicalquestions

I2l

Hom Paragl apn o ldentify and explain the figuralive language in this paragraph .

t3l
t1l

"

From Paragraph 7: Why does the author feel thal il is djfficuli to create relationships wiih one anoth_or of differing race? Use yo,r oW,? words as far as poss/b/e.

bt

Question on Passages 1 and 2 Give the meaning ofthe following words as lhey are used in Passage 1 and Passage 2 You may write your answer in one word or a shorl phrase.

exlerminalion (line
bedevrlled (line

1)

...............

19) ......

Faam Passage 2;

map (line 7)

absutd (l'ne

I7)

...........

essentially (line

35)

.........-................-.-..........t51

10

Ruih Hubbard argues thal it is unreliable to depend on genes to predict our illnesses. Moore and Wilson discuss how our race is not delermined by genes. ln your opinion, is there a value foa the study of genes? Using relevanl argumenls from lhe passages as well as your own experiences, justiry your I8l

view.

(L

JJc Mad-Year07 P2 answer scheme


Questions from Passage
1

r.

Explain how I is ronic for "educaled persons" to wanl "to sorl out ihe tacial or 'ethnic' components of our species so as to ascea6in our root idenlity " (lines 3 6) l2l

Lifted

as though rn

educated persons can come iorward lo argue

agnorance of our recent hrstory

"o.pr"t"

rnno""t

""

unj informed/most knowledgeable about histoical events and lraqedies arisinq fronr genetic/racial determinism/ discrimination to wanl to prcvenu.void tha qdnF IragFo,er lo n happening
again/makinq the same mistakes. (1)

it

.,s expeciea-

qars!!I!!9q4!&{9d

i,lit

eoucated

pe"o* -,ll

be

*lt

"

Onty

thal nothing could be more interesting and Howeve. these educated people are actually the wonhwhile rtJn ro son oul lhe ldcdl' or ones most obsessed/ fixatpd the sortinq of ''eihnic components ol our species so as lo difierences within the human species/carrying out asceriain our root idenlity the segregation which may lead to hislory repeaung

Whal does lhe author mean when she says a new prophel promises lo decode the currently
adverl'sed Roselta Slone.. "? (lines 8 - 9)

I2l*1 Paraphrased/lnfer.t
Jusl as a new prophei provides guiclance io make sense of a difficult probl8m, (1)

(scientlsts pioneering the task)

(spelingaut

who promises to clecode sequencing")

jnfornatian rnake it vitlualy impossible lo interyret)

(the vety valune and canplexty ot the new siudies (1) '

lhe curently advertised Rosetta Stone

lo decipher/make sense otlshare their understanding otsolve the puzzle oflinterpreUtranslate the gene poou generic 'ln
bonus for answets lhat points

.so new generations ot scientisrs hav emerged

hyped nature of the pronise

to the aver

FromParagraphs4-6
The wnter suggesls that there are reasons why knowledge of genetics cannol be relled on to predict

Summarise the reasons provided trom paragraphs 4 - 6 ol Passage'l in no more than 120 words. nol counUng the opening words given below. Use your awn words asfar as possible. t8l
The

tu,

teason why genes should nol be used to predict heallh is that.

. ihe manitestatons of inheited conditions can vary consiclerably and unpredictably

Different desrees

associaied wdh differenl murations

oi

nisabrl'ty

signs/symptoms thai differ/fluctuate loVnoticeably/signincantly and randomly/ erratically/ capriciously Secondly. varying levels of affiction/ ailmeni j are linkecu related lo various chanqes/ l

inheiled

diseases

63

same pe6on al dit{erenttimes.

irom one person to another in

the

Vaiations that

BD!h raLlats ate lurther oppendant on


occasions within the same individual.

\hF

behryeen

individuals/people and between instances/

.degree of disability is difterent for different people as also for the same lhis is so because many factors, both within and outside ourselves atfect lhe
ways we clevelop and function
OR

internal and external influences/ determinants/contributors lhai have an


eftecl on how we lrve our lrves/rnilLence ho*

..due to lhe numerous/ large number of

relevanl genes and proterns engage ln multiple interactions wilh othe. things going on withjn the organism as well as
Even cond tions .which follow predictable These varialbns occur evn for known patterns of inheritance, can exhibit a inherited diseases/ illnesses lhat follow a set wide range of symptoms that difler in of highly ascertainable/ foreseeable severity n dirierent people or indeed. in the same person aldrfferenl times When ii comes to the more common and prevaleni he;nh conditions or diseases.

genes do nol predict lhe fact or time of

foresee/foreteluanticipate
OR

The siudy of genes ls nol abte


inception of widespread/prevailing itlnesses

the

to

onseu

.cannot foretell when or why an ilness may

rity with

any degree of

better prediclore than their oenes are

person

s life circumstances are

serious the condition may be wilh '9!9!4!!!racJ4l9!!!$!. ln tact, a mo accurate indicator ofour heatth
G

ihe level of riskdanger/how

But the relationships between our DNA ...knowing that the way our genes are aQd our proteins are neither s'mple nor associared wilh/linked to our biotogicat indirecv not

the condition of life itself

.e!d l|qyj!?ltgg 9I9IIUI9


",'un,opn

'An) S our al tO pan s dbare

srraishtrgrward/mqlli-E!9!9q-co,1pl9l and tlrgtabte/do noistaykemaiionsrmt

t
(line 53)
rn rne pl-rase "make

4
-he

pretense
.o"1o e\

lde,rilv Ine w teic dtl lLde a-d e{Darn how,t rs convFvFd

a mockery of the
12)

le"

mdhe arnoct<oy.

ot te The wiler

made use of words connotations/degradation like

Paraphrased/lnfered

. ihat we can use the worl.lwi.lF lhe genes of human populations unclerstanding of our history

.to suggest ihe possible repercussiom/negative consequences re. rh'n9 from gprerr derelmrr.sm
t%)

6ry

Questions from Passage


Paragraphs
1

f ',""rr.

"r^,"*". ,
2

and 2:

5.

Explain lhe wrilers rntenlion in using lhe Pop Quiz

t2l

The tall person and lhe short person


signiticantly more different
in iheir geneUcs

are

The pop quiz suggesls thai there are more considerable/ noteworthy genetic differences

oetween

pFop

rulh differenLes
(%)

in

f earures/appearances

...1han the black and lhe white man of a similar heighl

people trom different racial

than there are genetic diflerences between groups (%)

' Answets lhat atlenpl b ad.Jress the concept of having nore/less genetic .Jiflerences wlhoul bringing n lhe conplele conparison between appearance an.J nce wil be given %n; answers with no attempt lo nake any forms of conpatison
Race is somelhing we do to each other; il has nothinq to do with what our DNA does to us
The walers' intenlion is to debunU discreclit the atlachmenl of genes to lhe concepl of race
OR

The wrilerc want to rejnforce their pornt that race hacl nothang to clo with DNA/genes
OR The wrilers want to engage/ altract the attention

Why are Lalinos good al soccer? A golf qene for whites?' (lines 24 inlention in the use of rhetoricalquesiions.
Liftecl Why arc Latinos sood ai soccertD;we ever iark

27) Explain the wriiers


I2l+1

about lheir'soccei genes? lS lhere a "ma'1ial arls" gene lor Asians? A 'golr gene ior wiites? What about Tlger lryoods? Did he get hrs abilities
from his black father or his Asian moiher?

The wnlers are making a poinu reinforcingl


rhetorical queslion

emphasizins/ pe6uadinq readers (1) "1n bonus lat answerc lhal explah lhe function of

as

slylistic lool

ta

pose a

question wnh an obvious answet

And thats exactly lhe problem with lrying.to denne people by racei ihere are no ctear sctentafac categories that truly separale humans by the cololr ofiheir skln

. .lhat abilities/skills/talents have nothing to do

ldenlify afd explain the ngurative language in;his pa.agraph

l3l+1

Paraphrasedllnterrecl
ldenlifv simile 'Race !s like th dols on the screen (1)

'1m borl6 far

answets idenffytng similes an.l analoqv here.

the use of

ts'

The dots mean nothing uniil we create the meaning, as weve done over human history
n science race means nothing too

Jusl //(e dots on the screen makes no sense


uness we interpreuinfer what is happenrng.

Just like dols have no meaning unless lhey are


joined to torm a piclure. (1)

so race remains a meaningless concep, baseless hypothesis uniil we inject meaning


into it
OR . renders itselfto possible socialconskucts OR rd' e has no -eanrng as rt,i a (on(Fpl we l-aJF

Why does I he author feel that il is difi cull to create relationsh ips with one andihe r of diffed ng race ? Use you a$/n words as far as possib/e I1l

Lifted
People have a whole pile of history

Paraphrased/lnferred

ll is be;use peopie have a lot of/


many (%)

(holocausts of ethnic, ncial and re gious externination perpetualed by peoples of widely differenl cu ural and polttical affilalions and beliefs

lhat makes 1l difncult lo create these kinds of ...negative/ unpleasant predispositions/

preconceived notions/ prior negative


experinces towardspeople of anothe. race
1%)

' The idea of past experiences being many and negatNe musl be ponrayed fot a lull nark
FromPassageslA2.

6.
a

Vocabulary: 0m

I5l

,o/ocarsts ot elhnic, racial and religlous extermination


(N)

Eradication Eliminalion Geti'ng rid of Ending of Killing Slaughler

'To get .l af

sanething

Complete/Total destruclion Getting rid of complelely

the quesllon ihai

has

Baffled

Spoilt Cursed

bedevilled Europeans for a question

very ong lime? (u

lhat haurrs r's

in

'To be extrenely puzzled Bugged

liyins to map the /ocatjon and functton oi every gene


(v)

ldeniiiy
Denne

Explain

Show

Charl
in

Aff?nge Drait

'fo arcnge and detine

ldentify/show the position of


Draw

Match Link

d. No such lhing as race? Has


sc

ence suddenly gore ,rad? CGzy thai is tolally absurd (Adj) Obviously/Ulierly senseless/
illogical/ untrue/ incongruoLrs Contrary to allreasons

Laughable

Slrange

'Obvlous/y serse/ess

Laughably/ Silly Siupid

llad

So... what is ;f? Essentially (Adv) race is like the dols on

Basically Chiefly

'A
n d ispe
n

necessary
sa

Overall Largely l$ostly

and

lMainly

ble crit e r ia n Crucially Obviously


Especi?1ry

7.

Application Queslion:

Ruth Hut'barcl argues that it is unreliable to depend on genes to predict our illnesses. Moore and Wilson cliscuss how our race is not dte.mined by senes.

tn your opinion, is there a value for the study of genes? Using relevant arguments from the passases as well as your own experiences, justify your view. I8l
Sludenls need lo argue either in favour for the study ofgenes or against it by evalualing or discussing lhe benefits lhat could be derived irom il, or the deleteious effecls that could possibly stem from it Ihey need lo reier lo sonre ot the argumenls made by both aulhors and make clear their oM view and describe some of the experiences or evidence that they could draw on to suppodlhem The ldeas from the passages that studenis could discuss include:
There are those who leelthai we should delve inlalhe sllrdy oi genes in order
1

from person io pe.son due lo varying faclors or cond(ions lt cannot be escerl.ined d!e io oenes alone
5

This plan has cerlain drawbacks Firslly. the symptoms of disease can deviale

Geneiic siudies are prob ematic and unreliable because genes do nol p.edicl the facl or time oi thei. occurrence or severity of dlseases with any degree of

cenes do pray aj?4 in

9!jrq!q!!9!e!!r999!99!r n addition. there cannot be a constanl pattern of disabiliiy as.ribed lo any one
rson or time as lhe condiUons tend io fluctuale beyond any measure ol

0l

These complexit'es and complications thus show thal we cannoi study ofgenes lo comprehend his]ojj- or to shape social pollc

The sludy ofqenes re.ders race differences


1

1o

be irrelevanl or minuscule

Race is a socialconsiruci miherthan a biologicaldifference


2

Genetic sludy actually shows us thal we are more alike biologically than we
3

think. ll is difficull to eradicale social noms on how racl difierences are.ooted in our oenetic make uD

Posgible Value of Gene

Benefits)

lVedicalBenefits Gene study woLrld reap ce'lain bnefils in the medical sector for instance, in terms of genelic tesiing ior disease prcvention Genes ideniiJied as those responsible for cerlain diseases such as hearl drsease or breast cancer can be located within the DNA sirands ot individuals cluring genetic lesling Thls helps lo wam people of the irnpending health risks and allows lhem to take concrele measures lo avoid or delay developing the diseases

The study of genetics prornises lo create new markeis and jobs especElly
perlaining to the research andjevelopnrent irduslry. ln Singapore, the biomedical sector has been emphasised so as to make Singapore a biomedical hub This in turn would generate g.ealer rcvenue for Singapore as medicaltourism would be

Economic Prooress

Slimulates Discussion on Ethics Given lhe conhversjal nalure ol qene sludy, lhis would stirnulaie and create a piattorm for invigoting debales or discussions on the pros and cons of uUlising gene siudy to determine,or ireat cedain nredical conditions. ln Singapore lhe Bidelhics Advisory Commitlee was sel up lo ponder over such elhical issues This also acts as a sprlngboard tor fudher discussion amongst lhe citizenry

Eradicates Racism Gene study would clarit misconceptions aboui racaal difierences as shows us lhat we are biologically akin to one another. This is especially important 'l in a

country like Singapore. amidst the modern socieiy


diffrences have led to 9olatile situalions in the world.

we live in where

raciaJ

Prevenlion of PsvcholoqicalDisorders Genelic tesling has also promised 1o help identfy cetuin genes which purpodedly cause psychological behaviours such as addiction lt would hlp to idenlify ii children have cerlain genelic predispositions to abuse drugs and alcohol

6t

,l

Fosters Sense of Belonaino People are cu.renily using DNA tesling lo irace their ancestry lhus enabling them to develop a sens of identit and fosler deeper lies wilh their racial groups. This also calses socal perceptrons of cial difterences lo abate as results often indicate lhai each person has an ancestry coniaining ot various races rather lhan

Harmful Effects of Gene Study Economic Conslrarnts

With genetic iesting becoming more prevalenl, health insurers may genelic tesling lo bar cena n individuals who have been idenlited to geneswh (h may caLse dspdse from pur'h"5i g insu anre ln addltion, geneiic lesting is expensive and may nol be accurate
merely based on prediciions

Breeds Preludice

Although racism may be eradicated due to gene study, il may erstwhile resull in a new form ot prejudicei genetic discrimination in which people are discriminaled against due to the genes they have. fhis wouid be again, a new form of a danqerous social construct which may prove io be haza.dous io social stab ity.

ln addilion gene siucly may link susceptibildes lo certain dlseases or undesnable genelic dispositions wilh certain races, which may result in even fudher

PsVcholoqical Paranoia

Gene Sludy may resull rn psvcholoqical pamnoia as people may develop fala islic or defeatist mindsels after rcceiving negalive results in genetic lesting. This may
lead io an ovecdependency on genetic iests.

Ethical Conside.ations There may be abuse of geneiiciesling which would lead eihicalconcems such as if parenls were to choose designer babEs based on genelic folmulalions o. compositions Ihis may also iuelracialdilierences as parenlswould also be able lo choose the skin colour of lheir offspring or genetically determine oiher cha.acterislics closeJy associaled to race

6f

Ml Mid-Yeor 07 Poper I

1 2 3 4 5 6 7 I I
10
'11

"Punishment always the right solution to stop crime." To what extent do you agree?
Can the education system ever meet the needs of the young?

"Science is for the mind, literature is for lhe soul." How valid is this?

How necessary is adversity in nation building?

"The basic things are

lie

best th'ngs in life." How far do you agree?

We have more information but less knowledge. How true is this of the Singaporean society?

Discriminalion starts trom the privileged ;n our

"*i"ty.

Do you agree?

Old isgold How true is this? The worst kind of crime is crimes against the environment Comment.

There is heavier responsibility on the media to censor than ever before. Discuss.

Maierialism is the new reliqion. Comment.

12

Do the arts today represent reality?

)a

Ml Mid-Year 07 Paper 2

ln todays world, mulli-elhnic slates are the norm The tradilional nalion stale, where a dlslinct national group corresponds to a lerritorial unit, has become an endangered species Globalisation and the increasing movement of people across borders lhreafen lo krll ofi the nation-slate once and for all However, some ml,'ths resist reality, and majorily or dominant clritures in countries around ihe world still seek to impose their identities on olher groups with whom lhey share a lerr ory
Altempts to impose uniculturalism in muiti ethnic environments often come at the expense oJ minorily righls. To avoid marginalisation, minorilies oflen inlensitr/ their efforts to preserve and protecl lheir idenlilies The hardening of opposing forces assimilalion on the one hand and preseruation of minority dentity on the olher can cause increased intolerance and, in the worsl case, armed eihnic conflict. ln such cases and in order lo prevent escalalion, lhe proteclion and promolion of minorty rights is essential
Even though the evenis of the twentieth cenlury have lalrght us to think ofthe term ethnic contlict as one word, lhe two concepls do not have to go hand in hand Thal is, elhnic

1A

conflict is not inevitable in mulli elhnic slates Jusi like a mosaic arl piece; where a plethora of individual tiles ot contrasting textures and colours are inskumental 1o ihe
overall composition.

15

Good governance plays a vital role in involving minorities in socielies and protecting their rights and interests. Through recognilion, dialogue, and participation, alllhe cilizens ol a diverse society can form a greater understanding of one anothefs concerns. The media 20 and educalion have imporlanl roles lo play in this regard, as do political representalives and communiiy leaders.

Although no counlry has a perfect record on minority righls, a couniry like Finland for example, has worked hard 10 iinplemeni legislaUon in order lo promote good ethnrc relalions among its populaiion The Swedish'speaking Finns are lhe largesl minorily in 25 Finland al 5 71 per cent of the population. The status of the Swedish speaking Finns is exceptional compared to thai of other naiional minoriiies, due lo the fact thai Swedish is in addiiion lo Finnish, an offcial language of Finland. ln recent years, the government hds redoubled its efforls lo sellle the queslion of land ownership by the Sami, lhe indigenous people of Finland. Finnish, Swedish or lhe Sami language is laught as the mother tongue 30 of lhe student, and under lhe new legislation, children who reside in Finl?nd permanenlly, lhus including immigrant children, have bolh the duly and the right to go to comprehensive -, hool Other posttive actions taken by states include: legislalive measures that inlroduce higher maximum penalties for Fcially motivated crimes; lhe use of ethnic monitoring to ascertain 35 the number of persons of particular ethnic and nalional origin in various kinds ol employmenl and lhe setling of targels to increase the employment ol persons ot minorily origins in fields where they were under-represented; the eslablishmenl of new advisory bodies on maiters relevanl to combating racism and lntoleralce, including the launching and implemenlalion public awareness campaigns intended to prevent racial discriminalion 40 and increase tolerance, and the establishment of human rights institutions and ombudspersons lor ethnic and racial eqlrality. State aulhorilies need 1o ensure that minorities enjoy the lundamental righl io equality, bolh in wriflen legislalion and in society at large. The roles of local government, civic organisaiions and NGOS are imporlant in this respect Police, prosecutors and iudges 45 need to be of what conslitutes racial discriminalion and racially motivated cr rnes and in some cases, changing the composition of police forces to better reflecl lhe multFethnic communities lhey serve may be appropriale lt is also incurnbent upon

7/

3.
minonlies to rnlegrale themselves inlo lherr communilies. Other recommendations inclllde 50

mon(orrnq hale sp-ei h. promo|ng ampowermenl rhrough eclurdlion. and ensur,ng


adequate housing and access 10 heallh care.

to

During the Warsaw meeting, experts emphasised thal regional cooperation remained key

combaling discrimination

of

minorities. Special attenlion was paid

to the

rapid

proliferalion of hate speech, hale crime, and hate sites on lhe lnlemet. Experts agreed that many counlries in the region ofien did not acknowledge thal racial discriminalion existed and seemed to consider il only a problem in lhe United States and South Africa. The World Conference, experts said, should help ensure lhal internaiional and regional minorily rights instrumenls were publicised and understood by disseminating information ;n relevanl languages and conducting public educalion campaigns

55

ll is when staies lack the foundation for protecling minorjly rights or governments aclively 60 encourage intolerance for minority groups thal conflict-ridden environmenls ensue As tensions involving nalional minority issues iniensify, disenchantment with one's governmenl can evolve inlo conflict siiuations ln the pasl ten years alone, ethnic conflicts have plagued a handful of countries such as Rwanda and Burundi, the former Republic of Yugoslavia and more recenlly, lndonesia, East Timor and Fti. lt is tragedies like these lhat 65 compel the inlernational community to encourage a dialogue between minorities and qove.nmenls within all societies.

t0

Almosi three years afler former President Suharlo's dictalorship collapsed, lndonesia's problems with its minority communiiies are growing despite lhe benefits brought by democracy The new governmenl faces separaiisi activities in the provinces of Aceh and lrian Jaya, East Timor has yei to recover from its vote for independence, and ethnic violence has recently erupted in ihe lndonesian section of Borneo. Human rights groups
eslimate that between 3,000 and 4,000 people died in separalists and ethnic violence lasi year in lndonesia and thal more than one million people are now homeless because of these conflicis Although there have been new effods 10 devolve power 1o lndonesia's reqions, ihe Governmeni has yel 1o implement pro'minority policies, which means there may be more trouble ahead

70

75

1',t

The proteclion of minorily ights and the prevention of elhnic conflict was also diseussed at the Oclober 2000 Regional Seminar of Experts for Africa, held in Addis Ababa, Elhiopia. The Seminar agreed thal greater atlention must be paid to the economic 80 problems that give rise lo outbursts of ethnic conflicts in Africa. lt turther recognised lhat lhe realisalion of economic, social and cultural rights and the right to development is of crucial relevance to the preven$on of ethnic conflicts in Africa or any other region. lt therefore encouraged "full parlicipalion in political life ior all, non-discriminalory treatment 85 of all regions and ethnic groups within a country, and respect for the rights of minorities."
Ultimately, implementing policies ior minorfties should be done 10 iosler long-term stability and not just to appease the international community. Through dialogue, all parlies can share their concerns and work toward iinding a common ground As the High Commissioner on National lvlinorilies for The Organizalion for Securily and Co-operation

12"

as political correclness or pandering to special inleresls groups Nor should it be diminished lhrough tokenism or shori term concessions lnstead there should be a
genuine commitment to protecl the idenlily of national minorilies and create condilions for

in Europe {OSCE) has said, "Accommodating minorily interesls should not be interpreted

90

the promolion of that identity." The rnain objective is to prevenl conflicts in multi ethnrc siales before lhey happen. As the Secrelary-General lo the World Conference has
repeatedly said, "At both lhe human and lhe financial level, a cullure of prevention is more communiiy can beneficial than a cullure oi reaclion". All slates as well as the 'nternalional an inclusive approach to work together 10 increase dialogue among parlies and creale

)r

Ml lvlid-Year 07 Paper 2 '1 Whal are the faciors thal put'the traditional nalion-siate' (line 1) at risk? Use your own words as far as possible. [1]

2. 3.

Explain why the aulhor believes'the protection and promolion of minority rrghts is essential' (line i2) Use your own words as far as possible l2l

ldentify the metaphor used to describe multi ethnic societies What is the writefs intenijon in using it? Use your own words as far as possible. [3]
From paragraph 4, what ingredienls are necessary in'a diverse society' in forming a grealer understanding of one another's concerns' (line 19 20)? Use your own words as

4.
5. 6. 7.
B.

far as possible.

[11

How has democracy an lndonesia failed to resolve the'problems with ils minority communities (line 69)? Use your own words as far as possible. [3] Whal does lhe word 'outbursts' (line 81) suggesl aboLrt the ethnic conflicts in Africa? I1l
Explain why "a cullure of prevention is more beneficialthan a cultuae of reaclion" (line 96-97). Use your own words as far as possible. [2]

Give the meaning of the following words as they are used in the passage. You may wflte the answer jn a word or a shorl phrase. ascertain (line 35) fundamenlal (line 43) ensue (line 61) devolvA (line 75) pandering (line 91) [5] Summarise the measures taken by the states to protect minoriiy rights. Using material from paragraphs 6 8. write your summary in no more than 120 words, not qounting the opening words which are given below. Use your words as far as possible.
Thg stdte\ protect nlinorit!" tights h),...

l8l

10 ln

hrs conclLrsion, the wnter stales thal ,l rs for staies to make a comrnitment to equal treatment of all persons regardless of 'mporlant their racial or ethnic origin . Using arguments from the passage, what are some of the challenges governments today face in observing such a commitmenl? How can lhese chalienges be overcome?

/3

Ml Mid-Year 07 Paper 2 Answer Scheme From Paragraph 'l Question 1 What are the factors that put'the tradjtional nation-state, (line 1) at risk? Use your own words as far as possible. I1l From the Text
- Globalisation and

Possible ParaDhrase

The opening up of geographical/national boundaries and markets worldwide (1/2)


and

-the increasing movement of people the growing numbers/ migration of across borders threaten to kill off the people/individuals travelling to other nation state once and for all. countries ('1l2) have caused the t.aditional
nation state to be at risk of becoming extinct

From Paragraph 2 Question 2 Explain why the author believes.the protectron and promotlon of mtnonty rights is essential' (line 12). Use your own words as far as possible.
minorities often intensify their efforts to preserve and protect their identity The hardening of opposing forces assjmilation on the one band and preservation of minority ideniily on the other can cause increased intolerance and, in the worst case, armed ethnic conflict. ln such

I2l

-to avoid marginalisation,

From the Text

Possible Paraphrase

in order to prevent escalation. lhe prolectton and promol,on


cases and
of minofity nqhts becomes essenttal to avoid marginalisatioi
.

They are esseniial because:

+il ensures minorities are treated fair,y or favourably (1/2) OR not torcibly set" aside/ be isolated from
society 11/2)

Note: "discriminated,
"increased intolerance,,

''sligmatised" not acceplable

againsl

,'ostracised

+it

behaviour
]

ensures greater

differences (1/2)

and willjngness to

accommodating accept

-"armed ethnic conflict,'

+it deters the use of weapons during racial

c]2007 Mllnnra nsritulc PU2 GP MY E:AMiN"ON

1+

disputes/clashes (1/2)

{o prevent escalation...

+it puls a stop to growing tension (1/2)

Note: Answers should be phrased


sitive manner From Paragraph 3 Question 3 ldentify the meiaphor used to describe multi ethnic societies. What is the writer's intention in using it? Use your own words as far as possible. I3l From the Text Take for example a mosaic art prece Possible Paraphrase -The metaphor is the mosaic art piece {1/2) -just like lhe mosaic art piece is made up of many different components (1/2)

-where a plethora of individual tiles of so a.e the different races integral to the contrasting textures and colours are nat'on (1/2) because they come together to instrumental to the overall composition. form a cohesive society (1/2)
OR /can co-exist harmoniously each indivjdual or racia, group is integral {l/2) to the existing social makeup/fabric (1/2) of the nation OR

A vibrant sociely is made up of


racial groups and ind,viduals cohesively integrated (1/2) Flhnrc conflict t5 elhnic states

different

(l/2) who are

not

aevttdble tn

;altt-

writeis intention is to OeUunt/retute {11 the general perception that there wi,l always be conflicts in multi-racial/cultural states
-The

02007 Miennia isrillr Pll2 cP [lY ExamnatDo

From Paragraph 4 Question 4 From Paragraph 4, what ingredients are necessary in ,a diverse society, in forming ,a greater understanding of one anotheis concerns (line 1 9 20)? Use your own words as far as possible. ljl

7d

Note: Any of 2 of the 3 ints. From the Text


(From Line 19

Possible Para hrase The ingredients arel


and

20):

Through recognition, dialogue,


of one anotheis concerns.

participation, all the citizens of a diverse society can form a greater understanding

-acknowledgement of rights

(1/2)

OR

open discussion/communication (1/2) OR

-involvement processes (1/2)

politica l/social/civil

From Paragraph 10 Question 5 How has democracy in lndonesia faiJed to resolve the 'problems with its minority communities' (line 69)? Use your own words as far as possible. I3l
From the Text

...the new government faces separatist the recently estabrished democrattc activities in the provinces of Aceh and state/state authority contjnues to lrian Jaya... encounter problems/troubles with groups
wanting autonomy/self-rule independence..
(.1

Possible ParaDhrase

..has yet to recover from its voie for -continues

plagued by strife/struggles/confJicts despite a majority agreement for self rule (1)

to be

ethnic violence has rec@nt'y erupted in -fresh racial fights and .iots have broken the lndonesian section of Borneo out in Borneo (1) Note: 1/2 mark awarded lo response wlth mention on 'additional ethnic confticts' From Paragraph 1,1 Question 6 What d-ogs the word oLrtbursts (line 8 l ) suggest about the ethnic confljcts in Afflca? Nole: Bonus mark to be given (it applicable).
From the Text
Pa

I11

hrase

-suggests in unplanned/ sudden/unexpected to reactive/spontaneous (1/2) AND many /numeaous /widespread/prevaJent '112) multi racial clashes
to

From Paragraph 12 Question 7


02007 NirllenDi. nslituro
Ptr2 GP MY Fr3nin:iion

76

Exp,ain why a culture of prevention is more beneficial than a culture of reaction" (line 96 97) Use your own words as far as possible. t2l
From lhe Text
Line 96
971

Possible Paraphrase

-A prevenlive culture is more humane


respecting the rights of individuals. (1)

.. at both the human and the financial level, a culture of prevention is more
beneficial than a culture of reaction

..

sense

as well as making more

cost-savings when bloodshed and damage are minimised (1) Question


8

in terms of

economtc

Give the meaning of the following words as they are used in the passage. You may write your answer in one word or a short phrase
t5t

Word/phrase
ascertain (line 35)

fundamental (line 43)

ensue (line 61)


devolve ([ne 75)

l mark -determine -establish basic elemental -foundational -follow -arise -happens imtr9qE!9ly_949l -decentralise -delegate kansfer
acceding to others' wishes for self gain or benefits

th matk -coniirm -make sure

0 mark ensure

-underlyinq

pandering (line 91)

hand to pass to -entrust -qive to -accommodating -specially cstering -acceding -qivinq in

FromParagraph6-8
Question

Summarise the measures taken by the states to protect minority rights


O2007 Millennia rnsllule PU2 GP MY Fxaminal on

Using material from paragraphs 6 - B, write your summary in no more lhan 120 words, not counting the opening words which a're given below. Use your words as far as

possible.

IBI

The stales prctect mino ty rights by ... From the Text


motivated crimes (line 34 35)

Possible Paraohrase
(1t2\

l.legislative measures that introduce -by passing harsher laws/punishment (1/2) higher maximum penalties for racially on offences relating to or against ethnicity

2.-the use

ascertain the number of persons of ratio/makeup of different races in numerous pari.cJlar ethnic and naIonal origin in jobs (1)
various kinds of employment (line 35-37)

of

ethnic monitoring to -to delermine in order to keep track the

3.-the setting of targets to increase the to establish bigger quotas (1/2) in different employment of persons of minority professions where certain racial groups are origins in fields where they were under lagging/iacking in numbers ('112)
represented (line 37-38) advisory {he setting up of organisations (1/2) to bodies on matters relevant to combating deal/resolve with issues relating to ethnic racism and jntolerance (line 38 39) discrimination and prejudice/bigotry (1/2)

4.{he establishment of new

-the establishment of human rights -and oflicial organisations/committees to institutions and ombudspersons for handle issues relating to civil liberties (1/2) ethnic and racial equaltly (line 41-42) and to appoint advocates to ensure fair
5

treatment for all (1/2) 6. the roles of locaf government, civic organ'zations and NGOs a.e important in this respect: (line 44-45) +more aware ofwhat constitutes racial discrimination (line 46) +changing the composition of police forces to better reflect the multFethnic communities (line 47)

{he functions of state authorities, non-profit


bodies are signjficant in:

+gaining heightened knowledge of the factors leading to ethnjc stigmatisation/


segregation +altering the makeup of the police forces to mirror/improve the representation of the multi-racial groups in society

+monitoing hate speech (line 50)


+promoling empowerment through education (line 50)
O2007 Millennla

+supervisinq incidences
(1t2)

of

racist remarks

+encouraging people io fight for their rights through formal learning (1/2)

risirule

18

+ensurinq adequate housinq and to health care (line 50 51) +regional cooperation (line 52)

sure there sufficient accommodation and availability of medical services (1/2)


+makinq
'

is

working logelher

wilh

ne,gl-'bour'rg

countries (1/2) +ensure that international and regional minority rights instruments were publicised (line 57,58)

+rnaking sure that global measures for the preservation of rights of people who do not belong to the majority were promoted widely (1t2) +presented in different languages so that they can be comprehended easily (1/2)
+carrying out planned activities (1/2) to instil awareness of minority rights amongst the masses (1/2)

+understood by disseminating information in relevant languages (line s8-59)


I

conducting public education campdrqns

(line 59)

L
Question 10 Application Question

Note: Any
qqc-gplgd

-6 pornts ltsted

under 6) wrll be

ln his conclusion, the writer states that 'it is important for states to make a commitment to equai treatment of all persons regardlpss of their racial or ethnic origjn'. Using argurnents from the passage, what are some of the cha,lenges governments today face in observing such a commjtment? How can these qhallenges be overcome?

them

ln your answer, give your own views and the experiences which have helpecl you to form

igl
Possible Response Challenges governments face:

From Paragraph I

'Globalisation and lhe increasing Globalisation has made il very difficult to instil movemenl of people across borders Joyalty in people. I\Iovement across borders means lhreaien to kill off the nation state once lhat people who make up ihe human capital
and for all.'

is a problem for many developing nations and those lhat face constraints such as limhed
i

essenlial for a country's growth, may not stay. Th;s


resources like tn Singapore. How to overcome these challenges:

t
PL12

11 is importanl 1o make a country a warm and inviting place io live in ln this way. not only do we relain our precious human capital, we also attract

e.nq
CP MY

-q3! lSlglglf

tatenls who

blng in

much

Eraoinatoi

)9

needed experlise. The Singapore governmenl achieves lhis by:

Offering atlractive remuneration packages which come with benetits for spouses and

children as well Oflering benefils such as workfare bonus 10 look afler the welfare of its people Acting as an example in promoting a more

From Paragraph 2:

gracious and compassionate sociely thal does nol discriminate againsl any race, religion, language or gender E g The Singapore governmeni has even recenlly allowed for ihe hiring of homosexuals in the civil service Challenges governmenls tqce:

"The hardenang of opposing forces - ln couniries like Australla and UK, all potential assimilaiion on the one hand and immjqranls musl lake a lesl lo a\cerlain lheI preservalion of minorily identity on the proficiency in English. ln some countries, poteniial
oiher
and in the worsi case, armed conflict.

can cause increased intolerance ,mmigranls are even lesled on lhetr htstory.

This is to ensure lhal foreigners are nol excluded from mains{ream society and are successfully assimilaied

However. torcigners slill

speaking lo each other in their nalive tongues even lhough they are proficient in English or French

foreigners are seen by the roadsides or train sialions, selling pjrated wares, or at the park,

isolaled lrom ma,nsteam socrely tn some pla(es such as London and Paris, migrant communities or

do end up

somewhat

Howlo overcome these challenges:

"..the protection and promolion


become essenlial.'

minority rights

The governments have put in pjace a system of affirmalive action to assimilate or reduce lhe

marg;na|salion
foreigaers.
E

ol migrant

communrttes or

g. ln Canada, miqrants are provided with fiee

English classes. Hiring of foreigners is made easy as long as there is proper documenlalion.

ln Singapore, each major race is protected in the .onslilulion Speeches tn parl'dment can bp g,ven
in any of the four malbr tanguages according to the speakers' preference. Every beljever is allowed to actise his/her relioion
PU2 GP MY EXAMINAIiON

jD

g. Some roads are closed,for

Thaipusam and

Chinoav Drocessions

paragraph
no counlry has a perfecl record on minorily rights, a counlry like Finland has worked hard .. to promote good ethnic relalions amonq its population.

Challenges governments face:

.Although

As the aulhor has rightly pointed out that

the

corrmlnenl ro equal kealmenl of all pe,sons is an arduous one as reflecied in the phrase 'no
country". Such a view is a reasonable one and it is indeed one ol lhe grealesl challenges governments ln the example about Finland provided, it is evident

thal governmenls need to do more than merely


a1lempl to inlegrale different races and ethniciiies logelher. There is a lol more that has to be done simultaneously such as the proteclion of lhe rights of each ethnic group in order 1o achieve equilable treatmenl for all.

E.g. The Ame.ican slory tells us that, despite efforts lo provide lor equal treatment for all, the cohscientious application and observation of such a commilment can be difficul{ to follow.
How to overcome these challenges:

As seen trom the passage that it is necessary for governments to make a qenirine effort to implemenl legislalion that provides recognition to the ethnic groups involved and nol just acknowledge the presence of majorily groups, just like what the Finns did lmportant markers of elhnic groups include language, access lo educalion and

From Paragraph 6
"including ihe launching and implemenlalion of public awareness campaigns jnlended to prevent racial discriminalion and increased

employmenl, privacy are some examples of faclors lo consider when allowing for equal trealmeot. Challenges governments lace:

. The

author, again righlly highlighied thal legislative measures are just part of the 'package' ihat would pave the way for grealer respF(l lo be shown to all elhnrc Aroups rn a
socieiy.

The challenges governmenls loday face are the manifold programmes that have to work hard ir hand so ac 1o d, hteve d maximLrm level ol elhnic tolerance and equal lrealmenl tor ali, which includes both hard' measures such as passing of laws and soft measures slrch as education programmes ihat promole

9t9919r!!99rqe!q!94!Jslgleryc
G]2007
Nf

i r-cnnia rnsr,rule

P!2

CP MY Eian nal

oi

f/

Such polilical eflorl and

measures require much

How to overcome these challenges:


Governments need lo ensure or be more aware of

the elfectiveness of ihe measures that they have proposed. This can be achieved by making sure lhat lhere is reasonable represenlation of lhe
differenl elhnic groups and conducl regular checks and reviews of existing laws This would aid in making sure lhal measures are relevant and

From Paragraph 7
Siates aulhorities need io ensure that minorities erljoy the fundamenlal right lo equaliiy, both ,n wrillen leg'slation and in society at large.

9[ec!v9

as far as possible.

Challenges governments
Hale crimes are hard lo eliminate, especially if

the police force is made up of the majorily. Law enforcement could be difficull because
racism mighl exisl wilhin the police force ilselt.

New governmenl may face difliculties promoting empowermenl through education and also ensuring adequate healih care. New governments may iace financial slrains and may nol have enough processes in place to carry oul their plans.
How to overcome these challenges: Educatinq the public lhrouoi) public awareness programmes and campaiqns From Paragraph Paragraph
minorily rights

Chailenges gov;fiments facei

EnsLrring disseminalion of information of

This maybe difflcult as developing countries and coufllries currenlly in slrife may nol have the faciliiies to disseminate lhis informalion. Facilities such as inlernet and also dio maybe under the conirol of ruling parlies lhal may not want such
information

How to overcome these challenges: Use the media effecl ir{ely

O2o07 Minnn a luslitule P(.r2 GP [iY Examrnation

&1

Challenges Faced by Governments Today From Paragraph 10 ".. lndonesia s problems with its minority Despile lhe fact thal democracy is praclised in communities are growing despite the some nations, the issues of thic violence amongst the minorilies are still teelhing problems wailing to benefits brought by democracy... be resolved. These riols and ouibursts of violence" '' between 3,000 and 4,000 people died resulled rn innolenl lo\s ol lves and ddmagp ir in separatisls and elhnic violence last property year in lndonesia....more lhan one million people are now homeless... The Governmenls have lo dealwiih interesl groups and aclivisls who have a differenl polilical agenda "..lhe Government has yet 10 implement that go against the stale authorilies pro-minority policies, which means ihere may be more trouble ahead...' These challenges can be overcome by: Elecling a strong Governmenl thal is keen lo implement pro-minority policies and other legislative measures lo prevent further racial
conflicts.

the Syrian occupalion forces were forced lo wilhdraw. The new governmenl proclaimed ils commitment io help mainlain peace and provide s{rong representation for all elhic and religious
factions.

ln the mid-1980s, children were caughl in lhe elhno-national and religious civil war in Beirui, the capitalot Lebanon, whose population is aboul two ihirds Muslim and one fourth Chrislian; wilh each faction fotther divided into sects In 2005. in view of global p.essure, lensions receded in Lebanon and

Example:

Challenges Faced by Government Today

As mentioned in the passage, there will be more


unforeseen consequences it minority riqhts and fair treatmenl are not addressed

A sluqqish economy and hioh rales of ''...economic, social and cultural rights unemployment among lhe minority would resirll in and the righl lo development is o{ crucial sudden, uncontrolled and fatal racial riots where rdevance 1o lhe prevention of elhnic lhe authorilies cannol avert conflicls in Africa or any other reqion . " Other underlying faclors such as marginalisation ''...encouraged full participation in and abuse of righls againsl the minoriiies will political life for all, non-discriminalory further. hasien lhe siale of racial discord in the lreatmenl of all regions and elhic groups counlry within a counlry, and respecl for the righls of minoriiies These challenges can be overcome by: Prolecting ihe economic, social and cullural interesls of lhese minoriiies by working closely wilh the non r!t19 rt!91!I99rEe1!Sl!l!q9!
PU2 GP MY Eraminalioi

From Paragraph 11 "...greater atlenlion musi be paid to the economic problems that give rise to outbursts of ethnic conflicls in Africa. "

lnfesled wilh elhnic disharmony and tensions, il will be hard tor ihe Governmenls to sleer ils economy

10

dJ

The aclive involvement of the M3Os can help lo prcmole their shared interesls and ideals in order lo influence the policies of state governmenls.
Example:
NGOS such as Amnesty lnternational, Save the Children and Global Youlh Connect work with the

govemmeni to regulate certain issues sUCh as human rights; and lhis is evidenl in autocralic governments where NGOS work hand in hand with the authorities 10 reduce the repression of minority racial in ihe communilies.

ll is questionable if any one counlry can ".. encouraged'full participation in political lite for all, non discriminatory lrealment of all regions and elhic groups within a counlry, and respect for the rights of minorilies..." it is indeed a challenqe for the governmenis today lo exercise equalily and meritocracy in its policies for all ils people regardless of race and .eligion.
Example:
However, in my opinion, Singapore has done it with pariicipaiory state policy where every Singaporean is given a chance to voice out his/her

concerns. There

communication for differenl ethnic groups to contribute their opinions.

are many channels

o{

For inslance, minisler visils, meel the resident' gatherings and dialogue sessions are great means lo get 'up close and personal' wilh the government officials
Singapore's nalional pledge encapsulates ihe
essence of respect and sovereignty of rights given

10 every Singaporean so that future generations are given the liberty lo actualise their dreams.
The close monitoring and heavy involvemenl of the Government in the economy, social sectors, media and education "ensure lhe inlerests of Singaporeans

Chatlenges Faced by covernment Today From Paragraph 12, we know it is hard for the governments oi loday io find a common ground and commitment to ensure the minority interesis are protected as there might be a clash of international agenda wilh lhe global communily

t
02007 Mr reniia nsllule t,u2 cP MY Eramination

It coud qe an easy way out io focus on short term


11

t4

gains than to solve the key issues of concern to prolecl the culiural uniqueness of these minorities, allowino them to flourish rn lhe fulure.

These political blind spots will eventually result in


elhnic violence on a massive scale.

From Paragraph 12 implemeniing policies should be done 10 foster long ierm stabilily and not just to appease the internalional community... "

''

Example:

On Seplember 11, 2001 the Al Qaeda terrorisl network aliacked ihe United Siates to venl anger o{
its exlremist lslamic members against the West

This lerrorist allack and global calaslrophe is a siern warning thal ethnicity, racial conflicts and ground...' aggressron are isslres Ihal are nol drsaop-arinq and becoming less imporlant. ln fact, the "accommodaling minority interests should governmenls in the world such as the G8 nalions not be interpreled as polilical correctness' should promole more dialoque and policies to or pandering to special inleresi groups. resolve lhese issues. Nor should it be diminished throuqh These challenges can be overcome by: lokenism genuine be a Promote an inclusive culture a culture of concessions...there should commitmenl io protect the identily oi prevention lhan culture of reaclion where ce.tain national minorities and creale conditions measures dre laken rnlo prio'consideratio' for the promotion of thal idenlity " New policies and mandate should be established lo ".. a culture of prevention is more bring a revived notion of ethnicity sensilivily in lhe beneficial lhan a culture ol reaction..all form of racial campaigns and public education siales as well as the international where schools, media and the family funclion as communily can work logether to increase positive p,atformslo dissemidaie information. dialogue among parlies and creaie an Nevertheless, not all countries can apply the same inclusive approach 10 national identity .' old slralegles for racial harmony; certain policies have to be reviewed and localised for ils
effecliveness to lake place.

'' all parties can share lheir concerns and work loward finding a common

Closure Wqshall always learn frdm history and reflecl upon our roles as an individual member of the society we
live in

ll is disturbing lo know how elhnic conflicls ,mpact hve5 (eq Cenocide in Rwandd :n tg94 drove more than 1.7M refugees from their homeland and lhe roving lslamic radicals
slaughtered 2M in Sudan).

Though these chailenges ale not easily overcome, lhere is a need lo minimfse lhese elhnic sirile io bring back world peace and order in the future.

12
PLr2 CP lL,lY Examnalon

t3'

Grade

Descriptors

79

O Q fl E I O O D O O B

Systemalic returenc 10 the requirements of lhe question with evidence oia balanced lrealment. Evaluation is very sensible, well-supported and developed io its logical conclusion. Explanation includes elaboralion and refercnce to personal insights and interpretaiion. High degrce ofcoherence & organisation. Covers basic requirements ofthe question but not necessatly a balanced lreatment. Evaluation is not always convincing and tends to be superficial wiih limlled development of ideas. Explanation is not as through as lhat in an A

4-6

Coherence & organisation are not as sharp or


systemalic as thal in an

script.

Fails to address the given requirements of the

Explanation demonstrates limiled relevance and


developmenl ot ideas.

1-3

O O

Evaluaton is non-xistent. Ihe answer is d mere summary or restatement ot lhe text ratherthan an evaluation ot it. lnconsislency in the argument is evident. Coherence is in question.
Higher incidence of misinlerpretation of the te)d. There is evidence of inaccuracies.

c)2007 [4illennia rnstilute PU2 GP rVlY Examinalion

13

8b

NJC l\titl-Year 07 Paper

1 2 3 4 5 6 7 8 9 10 1l '!2

"Discrimination in any form will always persis{, regardless of measures taken to counter Discuss the slatement with reference to one form of discrimination
More needs to be done in Singapore to promote inlerest in ihe Arts Comment Discuss the impoiance of marriage in today's sociely.
How important is il for young people to be aware ol current affairs?

ii"

Teachers will never be replaced bylechnology Doyou agree? The lnternet has proven to be more a curse lhan a t,lesslng. Do you agree? The developmenl of a counlry depends on its leaders, not ils citizens. ls this a fair comment?
How far do you agree thal lhe mass media needs lo be conlrolled?

"ln our counlry, lhe role of the domeslic helper has extended beyond mere housekeeping."
Analyse the view Presented.

All competition is detrimentallo genuine education. Discuss.


Good sporlsmanship is no longer a realily Discuss. Which is more wodh saving: a foresi or a starving child?

O Nar o..1 Junror Colleg 2007

8J

NJC Mid-Year 07 Paper 2 Passaqe A When we talk about evil in lhe comfortable herd, lhe ethical core of the whole issue is whal psychologisls would call passivity in groups o. the bystander etfecl.

The byslander effecl is watching some evil lake place, bul since we are watching with others who are watching, and no one seems !o be doing an],'thing aboul lhe evil,

we go on walching and doing nothing about il lnstead of consulting our own feelings aboul what we do, we iake our cue from lhe olher byslanders. They are not doing anyihing. Therefore, we also do nothing lf something needed lo be done, somebody would have done il
No wonder we lake so naturally to waitinq around tor the word. Our childhoods were spent in groups. Theae we stood, smaller lhan the o1he6, to see what was coming.
10

We have 10 be bystanders while oLlr parents figure out how 10 load the car' Sometimes the leaders ol the group were involved in evil, not elsewhere bul againsl us, but lhe family leaders were what car ed lhe day. Whaiever simple orders of papal bulls, whatever bealings or blessings, come down from our parents are seen by us children not as just or unjusl but as reality, the way it is, the way it's done, the way il always willbe.
Should one judge one's parents? I feel disloyal because akeady by the age of ien I have lhese two moral values: staying sober and keeping promises. My parents have failed in carrying ou1 these lwo vir{ues But I myself am really living at Slage llloyalty lo my particular tribal group, to my parenis. So I leave ofl any furlher mention of the picnic. I will delav commilling myself lo judgment. I will hang around some more and see how lhings develop. I slop planning whal to pul on rny wriling pad I had ihoughl lwould write something beautiful, but something else goi in the way of il so I stop PeoplF who have bpen possve gFnerally havenl gol a qood sens; or tme lhey don't distinguish behteen what is happening to them now, at this very minuie, what has always been happening lo them, and probably will happen again in the fulure Since they are out of the loop of planning lheir own lives, their minds don't categorize time segmenls in the way minds must il they want to feel and be aciive. "l will do this now, that later": lhat is the conversation of an active personalily, not someone who doesn't expect lo lake control. As children, most of us spend years hovedng, happily or not. at the circumference of people in authority over us. Pulling oneself inlo wakefulness when others look dazed does not come natllrally For one thing, passiviiy is in the shorl ietm a thousand limes more practrcal lhan slepping forward to takeaction. One attracls attention from lhe enemy ifone moves: if one freezes, lhe enemles in your line of sighl conlinue killing whalever or whomever they have begun to kill and do not suddenly swing their atienlion to you A lesson oflhe most ancieni part of our brain is to slay slill and nothing will get you One needs learning from another part of the brain td overcome natural passivily. One needs a molivalion more passionale than self-interested Practicaliiy. When a woman is being knifed to dealh the praclicalthing ior those watching is todo nolhing. One of the most difficull lessons children learn is how to make their minds shout at lhe pasc've psyLmlogral mu<cles inlide rhem

15

20

25

30

40

Thal is how children have to live. lf lhey don't develop oul of the bystanding habit
ihey become adult byslanders
A.tapledlrcn Evil n the Conlatlable Her.i bv CaratBlv

45

B&

Passaqe B l\,4oral drifl is a particular and fascinaling kild ot bystander effect, a plain fear of being disliked if we stand apart tlom group flow Let us say lhal eight people are discussing slavery One pelson is sirongly againsl it FoLlr are strongly in lavour. Three suppose it is OK. or maybe not: they are backya.disls; ihey have no feel for the subjecl because il hasn t come up in their own lives, and they have poor imaginations, being self-cenlered like N,4acbeth. The three who suppose that slavery is OK make no menlal image of enslaving someone, or of being enslaved themselves.

Thal's the ambience of the room. The one anlislavery person makes a sirong objection 10 slavery. People defend slavery. Others shrug. The anlaslavery person speaks again The others keep giving one another more and more eye contact and less and less eye conlacl lo lhis inilaling speaker Nothing makes a strongly opinionaled speaker yield so well as being denied eye contaci. Afler len minutes or an hour or a week or two weeks, the sometime sirong dissenter says, "Well, I
suppose there are some lnstances wherc slavery mighl be all righl, of coutse"

10

lvloral drift is not just bovine behaviour at a public meeiing bul a lhreat to iustice N4oral drifl has laken place when a strong person of universal morals lloals over to a position oI cultural relativism or lo a less pronounced objection 1o the evil in question lncidenlally, much of the "mellowing out" that forly and f'fly-year'olds smilingly observe in one another is simply moral drifl The conscienlious d;ssenter has gotien t'red and lefl the huslings

15

20

Moral drift is a dynamic of people lalking to olher people on issues requiring conviction. For years lthought of gtoups as being venues in which one got all fired up aboul some project ln fact, groups tend toward more gentleness lhan firing They damp things down io a mossy agreeableness. That is why very great poems, like very great short stories, are nol wriiien in groups.
As with any human proclivity, some people are rnore vulnerable 10 herding in groups than others. A marvellous and readable expert in this field is Dr Alice Miller, who has given us scholarly and poignant podraits of Adolf Hitler and his father of how psychological abuse of children gives them a poor self image thal they attempl lo repair or replace with love of a charismaiic leader. Anothet wise counsellor about herd menlality, especially as il operales in our commercial world today, is Tome Kitwood. He slates lhat because modern people spend such a significant share of

25

30

their lives working in large organizations, they scarcely can recover {rom the psychological effects in lhe time left ihem afler retirement. His is a grim view
Accepling a low-level moral code during the day is bound 10 infillrale one's off lime judgmenl as well

35

The gloomiest social insight is the facl that although the ways in which we spoil our personalities by herding slarP us in the face, we ignore lhem. These obsewations are corroboraled by the social work theory ihat some undesiaable behaviour, such as acting aulhorilarian arorlnd one's workplace and one's home, comes nol of present bad feelings bui oI yeaGin, year oul habit Such behaviour will not respond, iheretore lo ordinary psychotherapies: one needs 10 exert 'character" lhat is, willpower lo drop the bullying habil. Such characler or willpower, however, is a quality typically urldeveloped or attophied by life in the "low levetjustice struclure" oflhe herd

40

How, precisely, does the herd make moral m'stakes? l-Jow can we spot the group dynamic as il siaris up? For il we can spol a bad dynamic, we can stop it by showing
A.iapledfron Evlnt
Ih,"

45

Canlodable Herd by CanlBly

87

NJC Mid-Year 07 Paper 2 Questions on Passage A

1.

Using materials from paragraphs 5 to 7, summarise in about 100 words (excluding the words provided),: the characteristics of passivity how and why people fall into passivity how passivity can be overcome be given lot good Use your own words as far as possible. Crcdit

wi

organisation. One ol lhe "haracleflsliLs tel

ol

passrvrly

rs

Questions on Passage B paragraph 2 From lnfer how the example on slavery illustrates lhat "moral drift is a 'lhreat to 2 justice .
t3l

from paragraph

3a

What does 'smilingly" imply about the attitude of the forty- and fifty-year olds? t1l Explain such an attitude. t1l

3b. 4

Wilh reference to lines 16- 17, how does "float' illustraie the image of "moral dtill"2
12)

From paragraph 4 Explain the f'guralive language in the sentence: They damp things down to a 5 mossy aqreeableness (line 23-24\
"t21

From paragraph 5 Why are some people more vulnerable 1o herding? Use your own words as far as possible. t2l From Paragraph 7 7. . Based on lhe writer's closing remarks, whal can you infer about his attitude to lhe issLre of the herd mentality? Use your own words as far as possible

t:l

Questions on Passage A & Passage B


Give the meaning of the following words and phrases as they are used in Passage 1 and Passage 2 You may write youa answer in one word or a shori phrase

fo

core (A, line 1) naturally (A, line 9)

proc[vrly (8. line 26)


qloomrest {8. line 37)

atrophied(8,tine144).............................
9.

...

..

..t51

Passage A and Passage B explore the 'bystander effect' from the psychological and sociological perspectives respectively. With reference to both passages, discuss the extent to which the 'bystander effect' is observed in today's world? Suggest how yolrr society can possibly dealwith this concern. Support your answer with examples from your personal experience.
tel

q/

NJC Mid-Year 07 Paper 2 Answer

Scheme

Using malerials from paragraphs 5 to 7, summarise in about 100 words (excluding the words Provided),: ihe characteristics of Passivily - how and why people fall into passivity - how passivrty can be overcome Use your own words as lar as possible. Credit wrll be given for good organisatron. [9 m + 1 bonus]
.

One of the characieristics of "passivlty" is

g!9Eq9rE!i.ci 9I

P9 "l
ottime/
They lack temporalreasonrng skills / Their lemporal orientation is lacking/ poor
A lift of the word tine is acceptable, providecl the cancept is elaboratecl on a lillle more e g. ''does nal have a strang canceploftime, orthe

(a) haven't got a good sense

their minds don t categorise time segments in the way rninds must if Ihey want 10 feel and be
active. (1)

sequence of events ''poortinle managemen!

,.

(b) lhey don't distinguish between what is happening to them now, at thisvery minute,
(c)what has always been happening lo them, \1t2i

They are Lrnable to differentiate lhe occurrences taking place al that instance From habitual occurrences

(d) and probably will happen again in

the

Occurrences that might take place sometime


They are aimless/They lack/ are devoid of a umose and aooalin life

lutute l1l2 (e) lthey do not] plan their own lives / don'l

" and how it can be oveacome: ltow & 1ry!yj99p!9 tun into " They haie been, a long time, being overly years the hoverang at {f)spend circumference of people in authority over us relianv too dependenl on those in power/ leaderc/ those who are rn positions of (1)
(s) Pulling oneself into waketrlness when dazed does not come naturally oth

l) (h) Passivity is in the short term a thousand


times more praclicalthan stepping forward to

leadershlp in lheir society. It is not one s instinct to be aler, observanU sharp/ want to be dissocrate themselves from

9]!9I!

rafearor ()
(1)

useful/ pragnaticfat lhe byslander etfeci to We have been physiologrally wired/ proqrammed io be unresponsive / remain 1on' hdrarV -,l nvolved/ drsengeged esoeLidllr' when danger is in sight (infe_rr9!) To be able to get beyond the stage/state of being nonchalanU indifferent requjres

5 nc.u,al, tarate/ e{lramely easy/,e./1. i;(

:!!tql]).i,4rp

1i)A lesson ofthe most ancient pari ol our brain is to stay slill and nothing willget you One needs learning from anolher parl of the brain to overcome natural passivity (1)

l)

QI

(k)One needs a motivation more passDnate lhan self interesied practicalrty (1)
Passage B

One also has lo be rnspired/ stimulaied/ spufied/ stirred by a more noble desire than merely ihat of fulfrlling one s personal inclinations and desires

Fro.n paragraph 2 -mo lnier howihe example on slavery ilLslrales lndr Fiom the
T) one antislavery person makes a slrong

dldtttl

rs

l"real to juqlr(

[^ml

objection lo slavery
OR antis avery person

Someone may feel exiremely passionately against an apparent social iniusiice While ihe rest ot the group is supportive oithe action / ihat apparent injusuce /Even as that lone dissenter allempts io argue his case againsl the iniustice conceot: one aoainst ma Fe e' pelences be ng gnored by l_e Iesrll is because ol this experience ot belng turned a deaf ear to/ neglected Conceot: Beino iqnoredl Not qiven attention that he willeventually surrender and shed/ lose his moral convictions.

People deiend slavery Olhers shrug /The spks again.(1)

,21 I

ne olner\ k.ep

qrv,nq l'ss and lFss eye

contact io lhis irritaling speake(1)

3) Nothing makes a strongly opinionated speakeryield sowellas being denied eye

contaci (1/2)
4) Well, I suppose ihere are some instances where slavery mghl be all right of

Concept: S!trendeling
He will a so eventually be brainwashed lnto being able lojusiify why that apparenl social injustice might al trmes be all right

course"l(1/2)

concept: Justify tnq

From paragraph 3 abolri !!931!4q99!!!ltq forty and,f i11v-vear olds? 1m Suggested answer disenchanted, resigned disengaged, Cynical, that forty-and out of the mellowing Much fifty year-olds smilingly observe rn one another Note: Given lhal a smile can convey a mnge af, rs srmply moraldrifi. aftitudes/ emobons, please allow far as many sersib/e arswe/s as lhere arc. The 1 nark

Exolain such an attilude Jlml

Fe4!qP.a!!

Suggesled ar!9!9I Perhaps lhey rcognrse/are a\rvare that lhey are shirking therr responsibilily (1/2)

lo speak oul against what they know is


righvshould be done (1/2)...
'

ne mi/\ wtt be g^ pr onty il n" c o'onal'en ' p.soniot, sLppois tne o'ir
Wiih reference io llnes 16 17,howdoes tloat ilustrate lhe image

'/ "

'd

of

moral dtII"? 12m)

l-Fromthqle$qsg f

,suqsesle! answer

f3

Just as somethrnq liqhunot anchored to the seabed floats to the sudace


of the waler (1 m) So, a sirong person of universal morals is expected to be anchored/ groundecJ/ rooied in firm moral values otherwise hd will drift /As when moral drifl sets in. he loses his moralanchor. and hence loses his moral

From paragraph
Explain the iigurative language in the sentence "They damp lhings down to a rnossy

agreeableness'[2m]
Suoaested answer They damp things down to a mossy

agreeableness

mots| soft condtlians (1/2), in an uncontrallable OR shape/ess


Moss grows in damp/

manner (12)
Being in a graup is likened to a damp condition as il promoles an exlremely conducive envjronment for hreedng passive padicipants, by smalhering ones personality

and opinions. (1/2)


The bystander effect takes place and becomes widespread when a person s conviclion is nol E\/en suppo4 and lhq petson gt-dudlly latet
his canvicttcn, aclopling lhe mindset
ar

Nate: The emphasis is not an damp . . dowF' as a phrasalverb Whal is being emphasised is the assaciatian belween "damp and "moss", gjven lhal noss grcws in damp areas

ofthe

majority, regardless of whether it is morally acceptable


ath e r,u

ise OR

ln addition, just like nosswhich has no definite lorm or shape, people wha agree are
nol abaolutely ceftain aboul what they are agreeing with.

o/2)
OR any other sensjble answe"r.

From paragraph 5

6 _

Why are some people more vulnerable to herding? Use your own words as far as

po\sible.

[2m]

Frol! the passagepsyclro ogicalabuse oi children grves ihem a poor seli image that they atiempi to repair or replace with love of a charismaiic leader.

Suggested answer _, The first reason might be because oftraumailc childhood expeiences where they were mentalLy toriured/ shaken which resulled rn them losingi confidence in ihemselves. that they now try to re-assert themselves by hoping to win the affeclrons/ approval of someone they

(Because modern people spend such a signifcant share oflheir lives working in large

organizaiions,) they scarcely can recover from the psychological effects in the time left them after retirement.

The second reason might be"that people nowadays, as a resull of staying loo long in a particular job. find il difflcult, when lhey stop work, to pul away the mentalstless/pressure exerted on them dLrrlng their career

From Paragraph 7

7.

Based on the wriiels closing remarks, what can you infer aboul his attitude to the issue of the herd mentahty? Use your own wods as fal as posslble. [1m]

From the passl

Suqqesled answer (any sensible answerwill be accepled) To encouraqe the readers io avoid being passive, and to lake action when necessary To convey a moral message (didactic) - Stop ihe bad practices by exposlng ihe bad
-

- Ends on a hopeful noie that the reader will be inspired to stand up for whal is righl

PassageA&PassageB

8
Core

Give the meaning of the following words and phrases as they are used in Passage 1 and Passaqe 2. You may wrile your answer in one word or a shorl phrase [5m]

%m,
Naturally Proclivity

0m

lnstinctrvely Tendency, rnclnation pessimistic

Gloomiest

(Passage B, 144

Passage A and Passage B explore the'bystander effect from the psychologicaland socioloqical perspectives respeciively With relerence to bolh passages, drscuss the exieni to whrch the bystander effect is observed in ioday s world? Suggest how your society can poss bly deal wrth thrs concern Support your answer with examples from your persona experience 19 ml

1 2. 3

Candidate should discuss one idea from Passage A. Candidate should discuss one idea from Passage B. Explain how their society can address ihe bystander effeci, using personal experience as

supporl

a.

Good answers would discuss new ways of addressing this concern

Nole: Given the phrasing of the question, it would be undesirable for the student to propose a soluiion as part of their d iscussion for R 1 & R2.

7/

PJC Mid-Year 07

PAPER I

l. 23. 4. " 5. 6. 7. 8. 9.

'Today's technology is d;viding us as much

as

uniting us-' Comment

Are harsh punishments the besl answer to rising crirne?

To what extent are the values of teenagers in Singapore loday shaped by tbe
mass media?

'ln .ealify, the United Nations

has limited

value.' Discuss.

'Work is more fun lhan fun.' (Noel Coward). Do you agree?

'Educalion is the only effective instrumenl society.' Is this true?

in bringing about changes

in

'The envircnment must be sacriliced for economic developmnl ' Do you agree?

'Enjoyable, but ultimately of littte practical use Consider the value performing arts in Singapore today in the light ofthis commenl.

'

of

the

How concemed should we be about world poverty?

lO.'There

is nothing good in war except its ending.' Do you agree?

ll.
12.

Has competition resulted in a less compassionale and caring society? Discuss this with reference 10 your coun1ry.

'Obsessivc behaviour is very much a part ofmodm

life.' Discuss

?l

PJC Mid Year 07 Paper 2

PASSAGIi A
Anne Nloir and David lessel lvrilc... 'l'here is now solid and consislenl cvidence liom scientisls all over the \a()rld that thc bra;ns of girls and bo)s in the womb are struclurall) dialcrent and thal thesc dillcrences come about as thc resuk olchemical subslances known as hormones Mnlc

lll

hormoncs chielly lestosleronc {rgan;se the dcvcloping brain into a male paftern \\'hich leads l(J malc behaviour. Abscncc of male hormones pcimhs the brain lo pcrsisl ils ltmal pallcm, rcsuliing in lemalc bchaviour. The wide diffcrences in the amounts of tesioslerone fiooding the growing brain account lbr lhe .ercal lariet) of sexuxl dilferences humans display. l'his variel' is turlher;ncreased by thc fact thal hormones conlinue lo be prod ced by thc body througboul lifc, and exen their influcnce on our behaviour. nolably al pubeny. In the extremes. young men \!ill be pushed lo*ards l0 liolcnce, and loung \lomcn to$'ards irrational and disruplive swings of mood and behaviolrr- More commonlv- nrcn become more confidcni. sho\v grealer conccnlratioD and channel their aggrcssion into molivalion and ambition. whilc women are slimulalcd to dcsire to form and maintain closer and richcr relationships. ln old age, as the honnonal springs begin to dry. lhc brain difierences bcgin 1o lose their sharp focus: t5 \lomen tend to bc more assertive and aggidsiive as lhe female hormones losc their po\rer lo neutralizc the teslosterone prcscnl in all women. Mcn, in lurn. become lcss aggressiee as their leslosterone loses its power to neutralize their orvn nalllmll) occLrr.ing female hormoDes: as they conlcmplale the garden lhcy lvonder why lhei' 20 \raslcd so mllcb time climbing up the laddeFof stlccess .
Dcspite this incontrovefliblc.esearch. there is slill the fear thal lhe hard won ballle tor \\omen s ighls rlill be lost iIthc concept ol'innale scxual dii]erences is conceded.

I2l

lhis l'ar leads some 1o deny thc \,alidily of all lhc thousands of clinical and sociological sludies as bcing all biased. thc lainted work- in tacl. ol 'conditioned scicnlisls. Others advocalc lhe slippression altogcther of such rescltrch- for fear ot lhe 25 \, 'n\.qucnce lhr r{.men , \lalu.
\\rde\pread anempls to destroy lhe sexual slercot] pe through pro!re,si\e ellu(dtron. childrin rre rc.luilcd lo r.od ahout prin.e tet.lalrng dragons lhcy are asked to rvrile stories beginning 'Nadinc put on the boxing gloves and slcppcd into the ring...'and 10 look al piclure books displaying female fire-fighlcrs- Al work. posilive discrimination is employed, dcliberatel) givingjobs to pre-determincd quolas ot women. \rho sometimes may be lcss well qualilied or proficient, in ordcr 10 compcnsate tor the apparenl unlairness of a male_dominated slstcm. and to give examplcs ofsuccessfirl role-modcls which will encourage olher llolnen !o compete and -.n;-e ro fl.i s rlh in il

lllThcrc Jre still

t0

i5

[,{] All this wcll intentioned aclivity is based upon thc false prenise that men and wonren rre the same. Once their essontial differences arc understood and accepted _ bearing in nind the substanlial number of men and \lomen who are 'excePlions lhai prove thc rulc' and clearly do not confbrn lo thcse generalised descriptions of se\ual dilltrencc \!c can begin to clear the air of tho ahosphere ol sexual ridicule and recrimination. Wc'n1en need no longcr hold men responsible for their supposcd 'lailure' and men need no1 fcar or deride the cffons of women to bcal them at thcir own game.

:10

/r

I-iberated try honeslr- ralher than imprisoDod by selldcception. men and \\'omen Nill ha\'e the coniidencc 1() slrike their own balance betwcen love and ambilion, lcndcrness and striving, and lcarn 10 cnioy and give proper'valuc lo their nalural selves.

45

PASSAGE B
Salzman

wriles...

llowevermuchNc strive as individuals and as a society to be gcndcr neutral. il is vinually ;mpossible lo be gender blind. Male and fcmale are tundamonlal calegories for human beings in dealing with one anolher. ln cvcr.,'place and cullurc. people immcdialcly identily olhers as male or female on firsl coDtacl Some sociologisls would sa) wc havs lo know a person s scx in order 10 know how lo inleracl wilh him or her. Any parent who has broughl a baby in gcnder neulral clolhing lo a playground knows tha{ anong the questions hc or she will hear is. "ls lhat a boy or a girl?' Only once that has bcn eslablished does lhc slranget begin lo inlcracl t\'ith the baby. At leasl one sludy has sho\lD that parenls and olhers tend to smile morc at female babies ll may

[]l

rvcll bc that $,e cannol cvcn seille on a facial cxpression


turks bcncath the diaper.

lil,

we have ascenained what

t0

Dormal characteristics of "maleness" and "fcmaleness." and how lhose affccl individuals. their relationships and society at large. Over the Iast quarlcr cenlury, therc have been seisnic social-psychological shilts takiDg place behveen the sexes. These shifts are b a largc extent drivcn by the d)namic inlcraction of sciencc. lechnolog). and economics, and are maenified-by the increasingly omnipresenl nledia ln one Nay or anolher. mosl major socielies today have to come to grips rvith big changes in the bcha!iour ofmen and women

[2] Whar does diflcr from place to place

and from one era to anolher are the perccived

l5

[]l In thc Wes1. there havc been lierce debales, still raginS. aboul which gcnder

)o

behaliours are ;nnate and which are socially dclennined. Whal s nature ard rvhal s nudurc ir the sexes iislill open lo discussion and research, bul t!'hat is cenain is tbal r\e e. Jnom) doe\ nol mulh carc. it rerr:rrd. pcoplc uho , rn prudu(c rnd , nn\umc and it care\ n^rhrno aboul $hellrer'h() bullon rheir j x kcl' on the n"hr,'r lhc lell a,n lhc\
play? Can they pay? 25

dependenl agra.ian economies through faclory-dcpcndent manufacturing econonics to inrclligence-ilependcnr service economies. The more women becoinc iully qualilied economic pafiicipants in their own right. ralher lhan mere depcndanls ol mcn. the rnore

[4]

The growing economic porver

ol

womcn owes much to the shifl from land

we all have lo face fuDdamcnlal questions about marriage. procrcalion. the role of famil) and thc changing fbrce of 'rhc tribe. ' rrsing in mo{ ul the de!el'neil n.r orr.. rnd thal r.o. h.. ir' imlr(t un rn"rridge i"J froc'calion Fe pl( drc no long.r orer thc hill at fofly. past it at filiy- old al s;xt) and gonc at seventv wc now have a \'asllv grcaler range of oplions in mosl arcas of lite and a longer lile in which to e\periencc tben1. so lhere is less nrsh 1(l gel nanied early and is more challcngitg 10 Lalc parenthood. sccond families and cvcn third families are stay marricd for a 'ifelime. couplcs sepa.ate and form ncw pairings And more olien lhcse da)'s far fioln unusual as

.10

Ii] ln our new \\,orld, lil'e expeclanc]'is srerdill

F7

lhan in the past, women are the ones with the wandering eyes. [6] Shifts in g"ender behaviour are rarely greeted with unanimous approval. Okay make 40 thal "nver." The people with more financial,.social and political power (mostly men) tend to resist the shift of power by those with less (mostly women). The vehemence of male resistance varies from culture lo culture, from the extrmes ofAfghanistan under the Taliban to the rearguard actions of male-solidarity movements in the Uniled Stats. In lhe short term, it is clear that cultures that resist the rise in femate power are los;ng 45 oi.r1 io those cultures thal accept it because the cultures that accept it are progressing health, economy, security and technology, lo name just a few. faster on most fronts Only history will tell what the longer trm consequences may be.

From paragraph

State in your own words. one negative and one positive example that hormonal eflbcts have

t21

2 Whal is the writer's intention in

commenting thal,"-..as they contemplale the Sarden fhey wonder why they wasted so much time climbing up lhe ladder of success." (lines 19 - 20) Explain your answer fully.

trl
From paragraph 2 3 Why do wonen want to protecl their riShts through "...lhe suppression altogether of such
research''? (line 25)

t21

4 What

From paragraph 3 does the word, "deliberatety" (line 3l), tell us about the attitude ofwomen's rights advocales? Explain the reason for this.

12)

5
trl

In your own words, xplain why in progressive education the 3 examples are often cited?

From Passage B From Paragraph 4 6 Why is lhe phrase. "the tribe" (line 3l) in quotation marks? '

tll

I to 5, in about I50 words, accounl for the cbanges in the probleins arising from these changes. Use your own words as status ofwomen and state lhe
lAing material from
tar as possible. paragraphs

IIUI
Questions from both Passages Give the meaning oflh following words as they are used in the passages. You may write your answer in one word or a short phrase.

' -

From Passag A

...............................t11
(b) premise (line 36)...

From Passage B
(c) ascertained (line

I0)...............

......111

trl
(e) unanimous (line 40).....

Ill

Passage

A acknowledges that men and women are naturally difterent but cautions thal these

differences should not be the reason for gender discrimination. Passage B $ates lhat society has to bear the consequences arising from i1s reluclance to accord women equal status \4ilh

DrawinS information and ideas from the passages, assss the place of womn in Singapore today. You should rely on your own knowledge and experience tojustify your answer.

Iql

/at

PJC

['rid-Yerr

07

Paper 2 ANSWER SCIIEME


Qucstions frorn Passagc A

Fron Parrgraph I L Stalc in )our own words. one negalive


elGcls on adolescents.

and one positive example


(2 marks)

of homonal

Notei Ans}r'ers need not be gender specific as long as


exanrples are shown.

negative nnd

I posiiivc

Negative

a) In the cxtrmes, youog men lvill be pushed to ards violencc.(lines 10- I l) OR b) young rlomen l0wards irralional and disruptive swings ol mood and behaviour.(lincs I
1

Beyond normal limits- young mcn will becomc aggrcssive/resorl to the use of force OR
Young women will become illogical and lcmperamenlal/cmotional.

(l-)

12)

Positiie c) More commonly, men bccome more confidenti show greater concentralion and channel lheir aggreosion into motivntioo and aInbil;on-(lines l2-l 3) OR

Men bccome rnore sure of


themselves/self-assu[ed, display strongcr

powers of attenlion and direct their energy from bad/violnl behaviour 1()
malerialist

(Notei Award /, mrrk for general stalements like achieving thcir goals/ achieling success 'ilhout spccific reference 1o material success/pursuil)
OR
On the othcr hand, womcn are inspired forge slronger bonds with onc anolher.

ic

pursu ils/ach ieve menl s-

while rvomen arc stimulatcd to dcsirc to fbrm and maintain closer and richer rclafionships.(lines ll-11)

d)

1()

(ln)

What is lhe wrilcr's inlention in commenling that, " ....as they confemplale thc garden lhey \ronder why they waslcd so mnch timc climbing up the ladder of succoss'-(line l9-20)? I-xplain your answcr tulll. ( 3 marks)

(a)

(inlcnl;on)'-inferred

To show lhat lhe diffcrences in


changes.(l/2In)

male

behaviour a1 lhe trvo different stagcs of their lives (I/2n) are a tesuh ol hormonal

(b). .lhe),lvasled so much lime clinbin

wh"n

!!.f I9S ]S!!C.M d !9!!!9

/o3

the ladder ol'success'.

their time scnsibly(l/2m) and instead wcnl all ou1./spen{ all their tine chasing

aflcr/lrl'ing lo
Mcn in turn. aggressivc...female r9)
bccome

acbieve material goals/tD/ing to rcitch for higher posilions in lheir careerc. (l/2m)

ln their old

age. they mellow

and

horm,,nc\.(line

I7

become lcss ambitious.(l m)

From Prragrapb 2

h) do t{om(n $anl lo prolecr lhcrr IrLhl\ lhrough ...lhc suppre'.ron dllu8elher (2 marks) ofsuch research "l (line 25)
(Nole: The correcl anslvers are in (;) and (ii).llowever if thcy only make a general slalcment as in (iii).only Z mark to be awardd).
Total marks
i1

,. \

{i)-{ii) and lrri} crvcn:

L(t

(i)...lhey still lear thal thc hard-won lhey have ioughl so hard lo
battlc...(linc 2l ) (ii)fbr women s rights r}ill 22\
be lost . (line

__

achieve

gcnder equalily/ women's libemtion.

(ln) (ln)

And are afraid that these enlitlemcnts


be laken a\a,ay

will

lrom thcm. '

Gi

women

--ro,

ra. .rt rh" .o*.q,,.tt*s r* s slalus {lines 25 26)

Afraid tbat it will


womcn in

allcl society.

thc position oI

(l/2\

trom I'aragrat'h J

4.

"dlocales? 'deliherrrel-\"

Whal does the \i'ord, "deliberately" (line 31), lell us aboul lhe attitudc ofrvornen's (2 marks) Explain the reason for lhis ighls

ovcrly eager/ anxious/ biased/ forceful/


p.e_

Al lvork. posilive...giving jobs to


delemined quotas

of \\'omen, who order to somelimes ...in compensate...unfairness of a maldominaled system...(lines 30' i3)

detcrmined/ adamant in addressing the issuc ofqender ineoualitv (lm Reason for havins such an attitude so as 10 ensure \\,omen have an equal footing with/arc on par wilh men (l/2m) in the .iob/labour market/ workforce / employmenr.(l/2m) Nole: Reference 1o iob market must bc

5.

In your o\\n words. c\plain \\,h)' ;n progress;ve education the

eramples are oftcn

ciied?

I mark
end

1l is to Pul an

lo

neralization/fi xed idea or ;m

the fixated

/of

...princesses slaying dragons...Nadine pur on $e boxing gloles.-.picture books displaying female fire-figh1crs.(lines 28 -

nolion lhal women/ females are (lrn) inferior/weaker scx.

thc

Questions fronl Passage B

From Paragraph

{ ll)
in quolalion marks
?

6.

why is the phrasc, "the 1ribe" (line

(lmark+lmarkbonu9
11

inferred

an

portayed. (lm)
(Bonns) Explain charrcterislics of "the tribe"

is bccause womeo are described in unusual . fashion/negatively

As membcrc of a tribe, they belong to a social group of people with the same belieflt/customs (l/2m) and adhere lo rules & regulations lhal arc
rigidly structured for then.(1/2m)

7.

Using material fron paragraphs to 5. in about 150 'words, account lbr the changes in the slalus oflroDlen and the problems arising froln thesc changes. Use (10 Inarks) your own words as far as possiblc.
1

Ltfi
Reasons

fot the chonges in fie

status

of

a)...*,e strive as individuals and as a ln today's social conlexl, as pcrsons and societ) to be gender-rentral...(paral. as a group. we are making tremcndous eflo /rllempl to accept bolh sxes line l)
wilhoul Drciudice/discrimination.. How men and women are vieNed is dependent upon the place and lime in question-/ Over time and in different places or locations. lhe stalus and/gr thc perception ofwomen has changedc)...lhere have bcen seismic social- Radical changes in society and lhe psychological shifts...(para 2, lines 14- mindsels ofpcople are apparent. I5) d) ...driven by thc dynanric intcracljon of These changes have been influenced by science, Iechnolog). and cconomics. and science. lechnology and econom;cs and are magnified by thc increasingly made more imporlanl lhan the), really are 9!!r!]!!!!!'n441Isq?.!!!!]!:L7I bv the Dowerlul mcdia. b)What dos differ from place to place and liom one era lo another arc fhe pcrccived normil rharaclerislics of "mrleness" and "temaleness"...(para 2, lines l2-l3l

/at-

e) ...the economy does nol much care; it rcu'ards...Can thc) pa)'? (para I, lincs 21,25) OR

Economic consideralion transcends lhe supposed dillerence between men and women. / Nowadays men and women are treated in lhe same wa) in our consumerorienled socicly.
(same

-lhe gro\\ing
26 )t3)
Prcble ms ari

economic porver of
l!!!ese chdnees:

pojnt

economic consideralion)

\!omen,. servicc cconomies.(pam 4- lines

qualificrl crononic prrticipants...face the basic issues of marriagc, childbinh fundamental queslioDs about marriage, and the function of tamil), will become procreation,the rolc of family and the concrns that need addressing. cbanging force of thc tribe. (para 4, lines 28 3l) g)ln our nelv world. life crpectancy is Due to the bnger lifespan ioday, \\'omen are marrying laler, thus aflecting steadily increasing...impacl
marriagc and procreation. (para 5, Iincs conceplion and childbinh. h)...and is morc challenging to slay married for a lifetimc. (pa.a 5. lines 16

f)The morc women become frlly A. t"".* b4"." c"ittlir'iy

si!8:lM!

employed.

ll

is more

difficuh for womcn to

reDrarn

married lhroughout lhcir lite.

i7)

i) j)

familics...lar from
38)

Lite

parenthood.

second

unllsual

They are hcsitaot in starling rheir famil'es earll /do not wish 10 start their fanilics

couples . new pairings.(para 5, lines 17

early/divorces/ multiple maniages arc

And more often ...women are the Nowadays. \\,omen arc more Prone to onrs ]rilh the *andering c)es. (para 5. infi delily/rnfaithlilncss. lines l8-39)
Questions from bolh passages 8. Cive the meaning oflhe following words as thcy are used in the passages. You may write your answer ilr one lvord or a slort phrase (s rnarks) From Prssag( A

asserlive(line 16)

selFassurcd,firm,bold,lorcelul. strong confiden1, possess;ng

confidently
aggressivc. decisive concept,idca-

p'.-;iitlin.
r6)

assumplion.hypothesis.postulation

idcology,rolion,claim.

lirom Passage B

/rt

cstablished. verified, madc clear. bc surc- made/make sure. lo bc sure/verv sure

irgrained. inborn. inhercn{. natural. inslincti!e born wilh agree, no objeclion, Lrnanimous (line in complete agrccment, 1o lolal agreement by all in a consensus- uniled unopposcd. 10)
innate (linc

2l)

group,

unequivocal,

in opinion

indispulable.

doubtless.
doubt-all in lavour

llithoul

9.

Passagc A acknowledgcs thal men and woincn are naturally differnt bul cautions thal these differences should nol be the reasons for gender discrimination Passage B stales that society has lo bcar thc consequences arising from its reluctance to

accord women equal slatus wilh men,

Drawing infbrmalion and ideas from the passages. assess thc place of llonen in
Singapore loday.

You should rcly on )our orvn knorvledge and experience lo jusliry ). our answor. (9 marks)
Candidates a.e to suppon lheir evalration wilh reasons and iustifications Weaker Onswers ma) shorv evidence of over generalizalion instead ofconcrete support and
ease

otunderslanding

Band A (7-9

mark)

Band R (,t-gJnarks
Examincs points lrom bolh

,e,?!4!

11-3

l!r{k

Points are taken from bolh passages;lhere are slslematic rcferences lo thc lcxts,

Examines points from only


one passage.

Passages..

"
Bolh aspects of lhc queslion are covered. Atlempl is made to presenl ideas systcmaticallyEvaluales the kcy arguments thoroughlyAns!vers the queslion.

Answers the qnestion


\\.ilhoLrl referring 1o idcas in the texts. Answers onlY one pafl otthe question.

Lvaluales relevanl 'orgunrcnls to so*. e,*tcnt bul discussion is

Mcrely summarises the


ideas

ofthe wriler wilhout

adding their own

supgrficial. Brings in ;nteresling


elaborating on these. Able to jusl ify some (economic, lirical. educational clc.)

contributions.
General discussion !vi1hou1

Explanaljon is oflen limiled or not l;nked 1() thc maiD arguments well.

relelring lo their socicty.

/al

achievemenls otwomen in their society clearly.

Sho$,s high dcgree

of

Coherence and

organizalion.

orgarizalion are nol as shary or sl,slemalic as the lop band.

Inconsistency in the argulnent. Coherence is in


question.

Possiblc Points from Passaee A t ftt"r" i" nJ solid and consistnt eridcnce from scientisti...structurally diffcrert.,-(p:rra l). {For example. wonen are cxempled from NS and caning llo*ever, {his must not be the basis 1br society to givc unfair lrcatmenl lo rfomen'

t,"tge' hold Incn re\ponsible lur ihrir "o supposed'failure'..(para 4). (They are unable lo succccd in their career because I
\4

omen *".t

of.sexLral discrimination and unequal distribution

Lib(raled b] ttun."tl-*it" p.op". talue lo lhcir nalural selve\' {para J)' ( In order ibr local women to cnjoy equal righls with men. there must be an honesl atlempt by all members of society b remove all misconceplions aboul women
being the inferior gender and to embrace/celebrat the unique slrcnglhs/qual;ties hoth men and somen )

..

of

domestic responsibiliries)

of

Possilrle Points

r. ...1ft"." ttut. U""n scismic sfl(ial-p\]chologi.al shilh laking plac( botscen lhe sexcs. (para 2) ( Since indcpetdencc, Singapore has $'ilnessed major changes

women significanrly. It will bc difticull for SingapoJe lo progress if womcn were rlenied thJ opponunities to fully parlicipate in lhe economic devclopmenl of the counlry. In a globalised economy- it is crucial to have a level playing field tbr

marriage.procreation, the role

of &q4X.4ql\"-j!?'Citlg lqrce of ':t!9

/0p

tribc"(para 1). { I he traditional roics ot women as child bearers and homemakers have changed. Social expectations srch as eariy malriages. staying married
rhroughout lile- stigma ofdivorces and rcnrarriages etc have changed dlamalicaily no longcr expected to bc srbserlienl as they are economically

Some Doints to consider aborlt ihe Droeress nrade bY local rr omen:

Pol!l!s4.!44qp49d

Grcalcr involvement ol women in polilics. For example, in the GE 2006 many women candidales lronr both lhe ruling and opposition polilical parties, stood 1br election logether rvilh thcir male counlcryans. Some womcn like Sylvia l-im holds important posilion like Secrelary Ceneral of the party (Workcrs Pany) $at shc bclongs Women are appointed to high political posilions such as Minisler of State, Parliamentary Sccretary and as aDbassador aparl from be;ng Mcmbers of Parliamenl. Examples include Ms.Lim II\\'ee Hrra (MOS, Min ofl'inancc and T.anspon). Pro{l Chan )'leng Chee (Singapore's Ambassador lo the Uniled Slalcs). Mrs. Yu Foo Yee Shoon ( MOS, N{ in.ol'Comn un ity Developmcnt. Youlh and Sporls) woncn MPs like Madam llalimah Yacob. Indranec Raiah and Dr. Alny Khor are making their voices heard lhrough their political pa(icipalion and are well respeclcd in the socielY.

E.

&ry!!a!ry",!rraq!
Successl'u' womcn entreprcncurs are a common feature in Singapore loday success of Singapore arc by no means lrivial. Some er(anrpl;s include olivia Lum (Hyllux). Jannie Tay (Hour Glass). l-ina I an-l-eo (The Link Croup) elc.

l heir contribulions 10 tbe economic

Thcrc are women who are spearhcading big ecoDomic corporalions. for e\drnplc Ho t lring r lcmasck Iloldindsr. c5\o,rIl( Plolcrsor lv) \r rKK lluspirdlr ( lairc ( h;dng (Ban\rn lree HoldrnS.r (lc l\e1 pl;,1 moior role' rn rhc c.ono'nr, L'c\clofmenl uf lhc.uunln. Woncn coostitule a high percentage ofthe rvorklorce and are no longer seen
as passive contribulors to the econom). ln the pas1. local women lvcrc mainly performing household chores and werc no1 regarded wilh imponance ;n terms

oftheir cconomic coDtributions to the counln'. "

l:.

.rq!ry

"'a r,rr""ri."l

Women are no longer associalcd \\'ilh jobs ihat are trad;tionally "female" dominated like nursing and leaching. They can be lbund in all areas of rvork like. enginecring. law. archilcciure, neurosurgcrv etc thal *rc once male enclaves. They have also made lhcir mark rs commcrcial airlinc pilots (fbr example in Jetstar). fighler pilots c1c Edlrcationally, woncn are exccliing in all liclds of sludy and lhcre is a high number o1 f!rnalc siudents in all the teniarv insl;tulions in Singapore Th

number of wonen :rchieving Poslgraduale qualificalions is also increasing More women arc better educaicd today as compared to their predecessors

Some impedimenls that arc hinderinE local women from makins Droercss:

il.

Culture and Tradilion


Some Singaporeans ot Mala). Chinese and lndian origins arc still clinging to thcir age-old cultural and lmdilional beliels and practices Girls are seen

"

as less imponanl than bols (lbr cg. they are considered lo bc fl1 tor domeslic chores only) and as such are given lcss priorilics and opportunilies gencrally. Prolcssional women have also bccn known to resign from their jobs alier marriagc to look aflcr the childrcn and lheir husbands as expecled by their cultures and tradjtions Unless these beliefs. valoes and praclices are changcd, women today ma) slill have to sufler the fale thal lhei predccssors vrenl through

,.

Minrtsq!
Dcspite 1he lacl thal Singaporc is economically affluent and progrcssing. somc people slill hold on to thc mindset that \lomen are nol men's equal and can ncver be.(Sadl). some e(fucalcd women. may also subscribe 1() lhis

think;ng). Singaporeans ofall generalions- regardless ofrace. language and rcligion need lo lotalll ch"angc their mindscls to accepling womeD as equal . nnlcrntordrie\ u' nrcn llllr pcr.(pri^r car cnhance $, rrrcn Tdnr. pali^n ' in all domains ol lifc. Curentl). no iroman is appoinled as lirll lime Minister in lhe Cabincl.
Religion can also be a barrier lo womcn achieving thc same rights
as men

Ftrltr!rion of nropres\ midc hv local $,,m(n:


L Although thcre ,s equalil) for bolh scxes to excl in all domains. in realily nary women are denied lhc opporlurities 10 showcase their talents and abilitics. For exaDple in lhe political arena. no women have been appoinled
as Cabinel Minister. 1el alone Prime Minisler or President Employing womcn as pilols is mrc and f-ar in bcl$'een. Rt'fardle\\ oJ hcr cdy.arr. nalcareer r.lirc!c-nenl. r\onran. un n nrarriag( is slill stereotyped lo be the homemaker. Singapore can slill be considered a

parriarchal socicly $here the lraditional perccpl;on aboul the role of lvomen

as homcmakers prc!ails.Ho\\evcr- the undcrcurrenls of change are palpable

loda]. Fi.\ed mindsels/prejudices of emplol(rrs regardiig the contributions lhai $omen can make due to lheir dual roles as emplolces ard molhers posc a hindrance to lheir progres(lhc ironl'is thc country needs them to procreatc for long lcrm survival bul the mindsel ofenploycrs is exlrenrely resislanl to
changc).

,'/b

RJC Mid-Yeat 07

eaper't

Answer one question from this Paper. Answers should be between 500 and 800 words in length.

1. 2. 3.

Are high salaries for government officials ever justifiable? "sinqapore needs more scientists than artists." Do you agree?

"The lnternei is the best thing that has happened to democracy in recent
years-" Discuss.

4. "Future wars will be fought over the control of natural resources-" Comment 5. "Beauty pageants are degrading to women." How far do you agree? 6. ls marriage still relevant in today's society? 7. ls it everjustifiable to infringe intellectual property rights? 8. What can we learn from the study of Mathematics? L ls capital punishment compatible with a truly enlightened society? .10. Do you agree that genetic moditlcation brings about more problems than
solutions?

11. 12.

"There is much more we can do to protect the rights of the migranl workers in Singapore." How far is this true?

"The winners in globalisation benefit solely at the expense of the losers


Discuss.

"

End of Paper

///

Passage

Jim Hatt wrdes abaut the rise of soft paternalism in America

'1

When lhe government tells you that you can't smoke martuana or that you must wear a helmet when you ride your motorcycle even if you happen to like the feeling of the wind in yoLlr hair, it is being paternalistic lt is largely treating you the way a parent taeats a child, restricting your liberty for what it deems to 6" your o*n good. Paternalistic laws aren't very popLllar in this counlry' We 5 hew to the principle that, children and the mentally ill apart' an individual is a better iudqe of what's good for him than the state is and that people should be free to do what they wish as long as their aclions don't harm others
But what if it could be shown that even highly competenl, well-informed people fail to make choices in their best inlerest? And what if the governmenl could 10 somehow step in and nudge them in the right direction withoui interfering with their liberty, or al leasl not very mLlch? Welcome to the new world of 'soft paternalism." The old "hard" palernalism says, We know what's best for you, and we'll force yoLl to do it ln contrast, sofl paternalism says' You know what s best for you, and we'll help you to do

it,

15

ln some states with casino gambling, like Michigan' compulsive gamblers have the option of putting their names on a blacklist or "self exclusion" list, thal bars them from casinos. Once on the list, lhey are banned for life lf they violale the ban, they risk being arrested and having their winnings confiscated.

4 .

The voluntary gambling blacklist is an example of what's called a self binding 20 scheme. lt is a way of restrucluring the external wodd so lhat when fulture temptations arise, you will have no choice but to do what you ve jLldged to be besi for yoLr. The classic case is that of Ulysses, who ordered his men to iie him to the mast of his ship sb that he could hear the song of the Sirens ihout being lured to his destruclion. As a freely chosen hedge againsl weakness of 25 the;ill, self-binding would seem to enlarge individual liberty, not reduce il So what is there lo object to"in a programme like Michiqan's? Plenty, say libertarian critics To begin with, they don't like soJt palernalrsm when it involves the state's coercive power' they are much happier with private self-binding schemes, like alcoholism clinics and Weight Watchers clubs What 30 bothers th;m is the way soft paternalism relies for its justification on the notion that each of us conlains multiple selves - and thal one of those selves is worth more than the olhers. The short-run self eares only about the present lt is perfeclly happy to indulge loday and offload the costa onto fulure selves. For example' recent shjdies 35 show thai teenage smokers are aware of lhe aisk of getting lung cancer as adLrlts, but they simply don t mind making the future seli suffer for the pleasure of the moment. The prudenl resolutions oi the long'run self are continually
rgnored

Bui why, sorne sceptics ask, should the government side with your pnrdent 40 long-run self against yoLrr hedonistic short run self? lf the goal is to promote

/'Z

freedom, there is an interesiing argument favouring the former. A distinclive quality of humans is that we do not simply have desires, we also have feelings aboul our desires. Take the unhappy heaoin addict: he gives himself an injection because he desires the drug, but he also has a desire to be rid of this 45 desire.
So why can l soft paterndlsm be lerl to thF prtvale seclor. as sone hbenarlans prefer? The problem is that private self binding schemes are easjly subverted when someone can make a quick buck off your weakness of will. One Michigan man who signed up for a casino s private self-blacklisting programme found the owners all loo accommodaling when he had a change of heart .Within half an hour, I was back in, he said.

50

Besides sofl paternalism, there are certainly rrore exalted ways to achieve mastery over unwelcome impulses. Exislenlialist thinker Jean-PaulSarlre used to insrst thal each of us is free Io redefine his characler lhrough an act of radical choice. For the religiously incJined, an access to divine grace might be what is needed lo stiffen the will
10

slratagems, like self bindang? The general problem you face is lhis: For a given uphill goai and a given strength of will, does there exist a paih, however circuiloLrs, that will get you to the top of the hill? By adding a new path here and there, state soft palernalism makes it more likely that the answer will be 'yes'.

Bui what if you are one of those people who rely on more mundane

so|]rce New Yark Times.

3 Dec 2006

//3

Passage 2
Daniel

Klein

es aboutthe morcl consequences of paternalism

1 2

Picture a man gobbling a second helping ot chocolale cake, or chain-smoking a pack of ciqarettes, or injecting heroin into his veins ls aggression or violence belng done to an innocent person?

ln a sense, yes. A fleeting, short{erm self ihat enjoys chocolate, nicotine, or


heroin is working his will on an enduring self that pays the cost. ll makes sense to describe ourselves as a bundle of multiple selves that overlap, intermingle and sometimes conflict Atthough you may not think of yourseF in the plural, the idea of multiple selves is really familiar Some people recognize a dark self - a Mr. Hyde - that lurks within, and act stralegically lo defeat him. lf such personal tactics are insufficient in subduing the Mr. Hydes that lurk, perhaps the government can lend a helping hand. Afler all, subduing bad guys is what the government is all about This reasoning underlies numerous palernalistic laws worldwide

t0

Laws help us not lo overuse a huge variety of substances, kom heroin 1(l penicillin Other laws protect us from buying on impulse, from not saving for our old age, and lrom murdering ourselves. Allthese laws are offered as a seTvice to the erstwhile partaker. Yes, people sometimes do things they regret You'd be ridiculous to say that yoLl never make mastakes, that you never do things to
excess. Bul does that mean the government should step in and protect us from ourselves?

15

20

One good reason to rgect paternallsm is that pubhc oflicrals, do not rn fact' know better whether an activity is detrimental to our enduring self Passing blanket restrictions on behaviour rides aoughshod on individuality Some people drink too much, or gamble too much, but many others do not Another reason to reject paternalism is that it sels a nefarious precedent. Up to what point does the government get to play nanny? Where does it end? Sometimes il is the government that seems to suffer addiction to power and it is the one thal needs to show reslraint Furthermore, the justification of "il's for their own good is bound to be abused. lt will be Llsed to justify all manner of special interest plLrnder, such as excessive licensing laws. But the chief reasons for rejecting palernalism do not deny irrationality in the private individual, nor even wisdom and benevolence in government slewards The chief reasons to reject palernalism are its moral and spiritual consequences..
Paternalism is demeaning to the Indlvidu;l because it demeans his exlstence. It makes existence a happening rather than a wilful aciion; it makes the story arbitrary and alien, rather than purposive and personalised The autonomous individual admits his inconsistencies, his conkadiciions, his bedevilling impulses, but insists nonetheless: Grant me the dlgtlfy of choosing which behaviours deflne my being, and charge me with the responsibility for the Mr Hydes that lurk. lf you suspect that some Mr. Hyde seeks lo undo me, then I

25

30

6" 7

40

say go ahead and let him try. A tirst mora! consequence of liberty, then, is dignity, the romantic sense of being the captain of one's soul and relishing the drama of one's existence There is a second moral argument against paternalism. To give our existence 45 beauliful meaning, io make ourselves becoming, we must learn how to manage our troublesome impulses But how do people learn self-command? The best teacher is liberty herself. The second moral argument is that liberty
breeds personal responsibility
ln the intimate contest of self-command, hubris often prevails. Beforehand we say we won't gamble at all. we won t get angry, we won't watch TV, we won t we won't! We forget that the mood and vision in which plans are laid may vary greatly from the mood and scenes experienced as the road is travelled So hubris produces unhappy experiences and a feeling of regret. From experience

we qain awareness of the need for better self-command and a will to defeat our entrenched impulses.
10

55

Rather than leaving the individual free to learn from experience and example, paternalism deprives us of moral opportunilies of choice. By pre-empting choice. it weakens the moral faculty of choosing one's own course And by presuming thal the individLlal is incapable of choosing competenlly, a paternalistic government may aclually make him incapable Wean a person in a world of decrees and prescriptions, and he may fear personal independence and responsibilily.

11

Mr Hyde sometimes makes me eat too much chocolale cake or smoke too mani cigarettes. that is my problem, and everyday I musi practise the art of
lf a

65

subdulng and negolialing with him

So$ce The Freeman: ldeas on Libefty, May 1994

//:'

Answer all the quesiions


you your own yoLl slill use passage your musl for answer, select the appropriate material from the words to express it. Liltle credit can be given to answers which only copy words and phrases from the passage.

Note: Give your answers lN YOUR OWN WORDS AS FAR AS POSSIBLE Even when

Questions on Passage

I
121

1.

According 1o the author in paragraph 1, why are paternalistic laws not very popular in Arneaica? lJse yout own words as far as possible.

What are the differences betlveen the old hard paternalism' and the new "soft paternalism (lines 12 - 13)? Use your own words as far as possible

t21

3a

According 1o the author, how do self-binding schemes help the individual manage "fLrture temptations (lines 21 - 221? lJse your own words as lar as possrb/e
ln the analogy of Ulysses (lines 23

t1l

3b.

25), what represents future lemptations"?

tll

ln paraqraph B, what does the aLlthor say about the nature of companies offering
private self binding schemes?
11l

5.

lJsing your own tvoads as far as poss/b/e, explain ihe allernatives to sofi paternalism sugq;ated by the author in paragraph

I2l

Questions on Passage 2
"After all, subduing bad guys is whai tlle governmeni is all about' (lines 12 13) What do these lines imply about how the government perceives the world? Why does the author put the word "my" (line 65) in italics?

tll
I2l

Using material from paragraphs 5 and 71o 10, summarise the auihois argLlments agai;si paternalism Write your summary in no more than 110 words not counti'g the openinq words belo\N. lJse your awn words as tar as poss/b/e
One of the reasons why paternalism should be rejecled

is

tgl

,/l

Questions on Passages

and 2

Explain lhe meaning of the following words as they are used in the passage. You may write your answer in one word or a short phrase.

t51

a) nudge (Passage '1, line 'l1) b) hedge (Passage 1, line 25) c) hedonistic (Passage 1,line 4'1) d) distinctive (Passage 1, line 42) e) fleeling (Passage 2. |ne 4)

l0

Jim Holt presents some arguments in support of soff paternalism in Passage l, while Daniel Klein is not in favour of it. Which w.iter's views on patemal,stic governments are the majority of young people in your country inclined to support? Explain the reasons for your choice. Your answer must reler to ideas raised in BOTH passages, as wells your own ideas and knowledge.

Iel

End

of Paper

4v

RJC Mid-Year 07 Papet 2 Answer Scheme


Questions on Passage
1

Accordlng lo the author in paragraph 1, why are palernalistic laws not very popular in America? Answer in your own waKls as far as possible

t21

a) We hew lo the principle lhal,children The reason is that Americans believe that a anc! the mentally ill apart, an individual person rs more capable ol makrng super oJ is a pg49Li!9!99 of v/hal's sood for decisions about what is beneficial
/advantageous for hamself (1/2 m)

"belletdecisions" (0m)
b) thaD the state is (hnes 5-7)
than the governmenl/authorities
(

1/2 m)

c) and that people should be free to do People should have the autonomy io act as they want to (1/2 m) whal they wish
cl) as long as their aclions don't harm

9!l?rf ..(i!9

!)

provided that they do nol others. (1/2 m)

hud to

Examiners' comments on Q1: Most students found this a manageable, skaightforward question

2.

What are" the differences between the "old hard paternalism" and the new soft
paternalism" (lines 12

13)? Use your own lvords as far as

posslble.

[2]

|roryi!9

p9:tegq

a)The alcl "hard" patemalism says, We know y!3!Sj9S!_b! y9g


b) AEd well force you to da it (lines 13
14

..-

The old hard paternalism claims to of whal is most beneficial for the

and wall coerce lhem to do it (1/2m) make sut e (0m) c) ln cantrast, soft paternalism says, while the new soft paternalism thinks that the cilizens themselves are aware what rs You know what's best for vou. most beneficialfor them (1/2 m)

d) and we' help you lo do it. 15)

nes 14- and will assist them to do so (1/2 m).

Examiners comments on Q2: Points [a) and (c) were often left out of students' answers. The focus for points (a) and (c) is on Wbg is assumed to be aware of whal is most beneficial to the indivjdual in each kind of paternalsrn

3a

According to the author, how do self binding schemes help lhe individual manage "future

/,/,

temptaiions" (lines 21-22)? Answer in your own watds as far as passible. Fram the passaqe a) lt is a way af restructuring the external wotld (line 21) b)so thal when future temptations arise, you will have no choice
It is a method ot reshaping/changing the surroundinqs/the environment/ circumstances (1/2 m so that when lhese future lemptations arise, you cannot help (1/2 m) "chosen" (0m but do what you have determined lo benefivadvantaqe you the most (1/2 m)

I1l

c) but to da what you ve judged ta be best fot vou (lines 21-23)

Students have to get any 2 of 3 points to get the Iull 'l mark. Lxaminets commenls on O3a For poinl (c), the parl about what is "best for you" was oflen leJi oui of students' answers Students should try to be as accuraie as possible in their paraphrasing

3b

ln the analoqy of Ulysses (lines 23 25), what represents "fulu.e temptations"?

l1l

From the passaqe IJlysses, who ordercd his men lo tie The song of the Sirens/the Sirens. him ta the masl so thal he could hear the song of the Slrers wtthoul being OR lured to his destruclion (lines 23-25) The inclination / desire to hear the song of se4! 1!!!I9fq!np19!19!!1 the Sirens re Lxamrners commenls on O3b

There were some very peculiar answers to this one - eg "Future temptations" are represented by the desire to sleep;the desire to cause massive destruction: the desire to go to war: the desire to commit suicide, etc. Needless to say, all such answers failed to
score any mark for this queslion.

ln paragraph 8, what does the author say aboul the nziture of companies ofiering private
self binding

schemes?

Ill

The author implies that such companies are self -servrng/concerneo only aboLll lurlher ing their own interest as they seek to make a quick profrt even at ihe expense of their clients'welfare (1m) % m for 'prdfildriven" (no hint of unethical

0mfor
FYaminers' comments on Q4:

4.J!g!!gpu9t9a

,/t

lo score a full mark for this question. Those who lost half a mark mostly gave answers that failed to convFy the idea that the companies had a lack of
Ivlosl sludents managed

iniegrity.

5.

Explain in your own wotcls as far as possib/e, the alternatives to soft paternalism suggested by the author in paragraph L

I2l

a) Existentialist thinker Jean-Paul The author suggests one alternative is the own Sa,'1re used io /rs/sl lhat each of us is individual re-shaping his exerting his free lo redefine his charactet characler/personalily by
through an act
(lines 54-56) OR

of radical

choice

willpower to resisl lemptations. (1m)

lhe aulonomy an rndNidual possesses in decisaon-makinq allows him 1o shape his unique personality. (1m)
The key idea here is thal the individual has the power Io make changes of his own

b) For the
57

religiously inclined, an lndividuals can seek supernatural assistance access to divine grace might be what 1o gain conkol over unhealthy desires (1m) is needed to stiffen the will. (lines 56
idea of the answer is lifted.

Examiners' comments on Q5: '- Point (b) is !q! an example of point (a) They are tlvo separate and distinct alternatives to'soft paternalism - A number of students did nol understand the word tadical" in point (a) and paraphrased it as "rational"l " - For point {b), the idea of "supernatural help from a higher powef' has to be made clear. Hence, answers like "being religious" and "having a strong faith" don't get the fLl ll mark

Questions on Passage 2

6.

"After all, subduing bad guys is whal the government is all aboul " (lines lines 12 - '13 implt about how the government perceives the

world?

12

13)

What do
I11

Tne governrnent sees the worlcl in a very simplistic/unsophisticated manner (1t2 m) of evil/

b) ;nd that iis duty is simply to restrain evil OR the world is full
temptationsldangqrs. (1/2 m) . Examiners' comments on Q6: Most students dld noi get the half mark for point (a)

Why does the author put lhe word "my" ln italics in line 64? lnferred Answer a) The author wants to emphasise/ highlighU stress (1m) his view

tzl

b) that the government should not interyene or influence people's decision_ making even if they may nol always make prudent choices OR the individual should
have the freedom to make his own decisions (1m)

Accept any logical inferred answer.


Examiners' comments on Q7: - For point (a), students should give an answer lhat conveys some degree of intensily {"to emphasise" is far betlea than to show", for instance) - lt is 'to emphasise' rather than "to emphasis" (something)

Using material from paragraphs 5, 7 10, summarise the authois arguments against paternalism. Write your summary in no more lhan 110 words not counting the opening words below Use your own wards as taras possib/e.
One of the reasons

aternalism should be The oovernment is not wiser about whether certain actions are really harmful/benef icial for the individual. Regulating actions uniformly suppresses the sense of self.
Key point herc is the idea of blanket

Iel

public officiais, do not, in fact, know better whether an activity is detrimental 10 our endurinq self

\lines?1-22L. passinq

btanket restrictions on behaviour rides roughshod on individuality (lines 22 23)

Up to what point does the government get to plal nanny? Wriere.loes rl end? llines 25 26 Sometimes it is the government that seems to sufier addiction 10 power (lines 26-27)

It is potentially dangerous as authorities may not know where to stop as they may be obsessed with the influence they have

lf degree af intensity of preoccupation is not prcsent,award r! justification "it of q for their own ALrthorities are certain to enrich the qood" is bound to be abused. ll themselves under the guise of knowing will be used lo juslify all manner of what is good for the individual special-interest plundeJ. such as excessive licensing laws (lines 28 OR
30)
l\,4isuse power to benefit themselves Accept eilher idea af "abuse" or 'specjal interesl plundea Paternalism degrades self-worlh

it de-means his

existence (line

35)

..

ft/

Eilher s)
OR
h)

Grani me the digrify of choosing which behaviours define my being (lines 39-40) To give our existence beautitul meaninq, (line 45) It makes existence a happening rather than a wilful action; (line
36)

It makes the acl of being a random event rather than one of choice

it makes the story arbitrary and alien, rather than purposive and Dersonalised. (line 36-37) to make ourselves becoming, we must learn rowto manage our troublesome impulses But how do people learn self-command?
(lines 45-47)

It makes life random and remote rather

than individualised and goal oriented.


Paternalism disallows us from acquiring ways to conkol/deal with our instincts / human nature.

The second moral argumenl is that liberty breeds personal responsibility. (lines 48 49) . the need for better seltcommand and a will to defeat our entrenched impulses (lines 5556)

i)

The best teacher is liberty herself. (lines 48)

as it deprives us of ihe freedom needed to do so.

k)

From expeaience we awareness (lines Y-56)

Lifl'libeftv=0ii
gain

Paternalism prevents us from learning from the past.

Lift'experience'=0m
D

it weakens the moral taculty of choosing one's own course iline

Kev tdea is the idea of exqerience ll deteriorates our elhical senses.


It makes a person unable to make good choices by assuming and acting as if he cannot

m)

And by presuming that the individual is incapable of choosing competently, a


(lines 58-60)

5B)

paternalistic government may aclually make him incapable Both


Wean

idea of presumption and consequence must be mentionecl to get

n)

person in a world of Paternalism renders one afraid of selt decrees and prescriptions, and he reliance and accouniabilaty by diclating may lear personal indePendence his very actions. and responsibility- (lines 60-62) (Each rdea = 1m)

Sample Answer

/ L3'

There are several good reasons lo reject palernalism The governmenl is not wiser about whether cedain aclions are really harmful for the individual. Regulating actions uniformly suppresses lhe sense of self ll is potentially dangerous as aulhorities may not know where to stdp as they may be obsessed with the influence they have Aulhorities are cedain to enrich themselves undea lhe guise of knowing Paternalism degrades self worth and makes the act of being a random what is good for lhe 'ndividual. evenl rather lhan one of choice lt disallows us from acquiring ways to control oLlr instincts as il deprives us of tbe freedom needed to do so. ( 100 words)

Examiners commenls on Q8: Some students still failed 1o indicate the word count or to give the right word counl for their summary, despite repeated reminders Half a mark was deducted for such scripts. - Some sludents continued to lift key words/phrases and were heavily penalised as a result Most made a fair attempt at paraphrasing the key ideas. - An annoyingly cominon language error is the failure to use "the" before "government'- eg. ''Government may become obsessed wilh the influence it has. "

/2J

Questions on Passages

I and 2
I5l

Explain the meaning ol the following words as they are used in the passage. You may write youa answer in one word oa a short phrase

l
a) nudge, verb (And what if the governmenl could somehow steP in and nuc,ge lhem in the right direction Passaqe 1 line 11) b) hedge, rout (As a freely chosen healge against weakness of the will...
Passaqe 1. line 25)

mark

Y,

mark

ioax/persuade/prod/ urqe/push

0 mark shove/poke/ guide/ direcl/move/ place/ encourage/ steer/ propel/ prompt

reshiction/defence/
/saf eg uard/protection

/shield/restraint

barrier/boundary/ obstacle/ obstruclion

fence/border/ enclosure/ prevention/ tool/ preventive measure/ precaution pursuing of happiness/ sensual/carnat/ evil/ decadenV enjoyable/ cares only about the present

c) hedonistic,

adjective (should the government side with your prudent long-rLrn self against your hedorlsfic shortrun self?
Passage 1, line 41)

pleasure-seeking /devoted to sensual enjoymenvlhe pursuit of carnal enjoymenl

self indulging

d) distinclive adjective

quality of humans is that we do not simply have " desires


Passage 1. line 42)

unique/distinguishing /defining/ oLrtstanding/ differentialing/ striking

characterisiic/ conspicuoirs/ noticeable/ obvious/ special/ distinguishable

distinct/ differeni idiosyncraiic/ recognisable/ significanV importanV crucial/ typical

e) fleeting,

transitory/
impermanenV

adjeclNe (A f/eel/rg, short term self that enjoys chocolate...


Passage 2, line 4)

kansienvmomentary/
quicklY

flittingfading/ quick/short lived/ short term (found in line 4)

swiftly/ passing/temporal/ temporary/ brier/ very briefl eDhemeral

'

passing/Bassing

Examiners' comments on Q9: - Sorne students gave a list of allernatives as theia answer. When a list is produced, only the first alternative rn the list is marked, even if lhere were an allernative that is correct after the first one Many students did not pay atlent'on to lhe word class of their vocab answer. Eg.

For the verb 'nudge , many gave the answer 'a gentle push" rather lhan "to push gently . Such answers wil be penalized in the language mark Jim Holt presents some arguments in suppod ot soft paternal'sm in Passage A while Daniel Klein is not in favour of it Which writeis views on paternalistic governmenls are the majority of young people in your country inclined to support? Explain the reasons for your choice Your answer musl refer lo ideas raised in BOTH passages, as well as your igl own ideas and knowledge
Re

10

uirements:

on paternalislic people are ljkely to suppori' in tfreir country governments the majority of young making reference lo ideas raised by both writers. They must demonstrate a good understanding of young people in their counlry' and the influences young people may be subjecl to which would shape their thinking on ihe kind of government lhey find to be desirable. There must be adequate support for candidales' answers based on ideas from both passages, as well as their own relevant knowledge and experience
Candidales must clearly idenlify which wriler's views
Explanation
and-

Evaluation:

Candidates should show understanding choices and related issues are:

of the issLles they choose

Likely

Passage 1: 1 Palernalislic govenments resttict an individual's libefty (para 1)/ Ctitics clislike paternalistic governmenls as it involves lhe state s coercive poweL (para 5) Younq people may disapprove of paternalistic governments because lhey are al the stage where they want to assert their independence, instead of having their parents, and by exlension the state, breathing down their necks ? An individual is a better judge of what's good far him; people should be'free to do what they wish as long as their actions don't harm others. (parc'1) Young people may think that they know what's best for them, rather lhan the state Even highly competent, wellinformed people fail Io make chaices in their besl interesl, so the govenment shoultl slep in and nudge them in lhe right direction (para.2) Young people may lack the maturity and experience lo make prudenl decisions, so they may feel that the state should guide them to make the right
choices.
4.

6.

The shoft-run self carcs only about lhe present; lhe prudenl resolutions af the Iong-run self are continua y ignared. The government should side wtth the long run self to prcmote freedom. (para.6-7) - Young people are generally fun loving and may hence lack moderation in their pursuil of pleasure. The state should step in to help them make choices that would promote their liberty Fxlstertia/lsf lhinket... used to insist thal each of us is free to rcdefine his character lhrough an act of radical choice. (para 9) -Yottng people may loathe palernalistic qovernance as ii prevents them from delermining their identity and becoming who lhey lruly are An access af divine grace might be what is needed Io stiffen the wi (para g) Young people may prefer to lurn to religion lo help them overcome unwholesome impulses or desires Pary!nsl4q39!9!l!!E!E tglpt a!9L9 !9!93t-greater chance af reaghing orte's

/tJ-

gaal. (para 70) - Yolrng people may not mind governmenl inlervention if they feelthat their aspirations/goals are more likely lo be realised as a result Passage 2: 1 The government can lend a helping hand to help one subdue his dafu self (para 1) / Laws help us not lo ovearse sl/bstances (para 4) - Young people may feel thai they need the s{ate s assistance to prevent ihem from going astray 2 Other laws protect us from buying on impulse. fram not saving for our old age, ancl lrom murdering ourcelves (pata.4) Young people may accept paternalistic laws ii they believe that such legislation safeguards their inieresVwelfare 3 Government officials do not know befter whether an activity is detimentalto our enduring self. (para 5) Young people may think that they know what's best for them, ralher than the state {similar lo idea from Passage 1, pt 2) 4 Passing htanket rcstriclions on behaviout tides roughshod on individualily (para.5) / Paternalism de-means the individual's existence (para 7) and robs a person of his dignily (para.B) - Young people may loathe paternalistic governance as it prevents them from determining their identity and becoming who they truly are. (similar to idea from Passage 1, pt.5) 5 The government may abuse jts power, using palenalistic laws and policies lo fufther its own ends (para.s) - Young people may rejecl paternalistic governments because they believe Such authority is likely to be abused. 6 Libefty breeds personal rcsponsibility (para 8) / From expeience we gain awareress of the need for better self command and a will to defeat our entrenched impulses (parc.g) Young people can learn to take responsibility for their own actions when they arqgiven freedom of choice. They wanl to have the room to make m'stakes and learn from their experience, hence they prefer non inlerference from the governmenl. I Patenalistic govenments weaken lhe moral faculty of choosing an individuals own course. By presuming lhat lhe individual is incapable of chooshg competently, a paternalistic govenment may aclually make him incapahle (para.11) Young people may loathe the idea that the government assumes ii knows better than lhe individual. When they are nol given freedom of choice, they become incompetent ;n making decisions. 8 Wean a person in a world of decrees and ptosctiptions, and he may fear personal independence and responsibility. (para 10) - Young people may feel that they need to have the liberty to make decisions, so thai they can learn to take responsibility for their own actions.

q"!J9!99L.

Good answers must fulfil the requirements outlined above and demonstrate logical flow of ideas with good organisation "and the use of relevant conneclors/signposting words,

Examiners' commenls on Q10: A significant number of studenis failed 10 address the question adequately lhe queslion asks about what most young people in one's country feel regarding paternalism. not about one's personalopinion regarding paternalism - Many answers lacked support / evidence for their arguments - Some even contradicted thdmselves: eg. Some claimed that young people supporied paternalism at the stad of their response, and that they opposed palernalism by the end of the response! Some students seemed to know the writers of the passages personally and

rl,4

addressed them by their first names! Students should use last names "Holt" and "Klein" instead of first names. Common language errors inclLrde the failure to use determiners "a" or "the" before "rnajority"; and the tendency to use colloquial expressions like 'kids" or "youngsters" in reference to young people.

l0

/Ll

SAJC Mid-Year 07 Paper

Franklin D. Roosevelt said that the world should be founded upon 4 freedoms. Freedom of speech, Freedom from want, Freedom to worship and Freedom from fear. Which, in your opinion, is the most important freedom? 'The beauty of the arts lies in it not needing a reason to exist ' Do you agree with the statement?
To what extent is discrimination present in Singapore today?

2. 3. 4. 5. 6. 7. B 9. '10 11 12.

'sports is about money.'To what extent is this an accurate description of sports todaY?
'The mass media is no longer reliable.' Do you agree?
"Ask not what your country can do for you but what you can do for your country." (J.F Kennedy)Are the youths in your country able to

take up this challenge? Humility has no value in today's world What is your view?
tn an increasingly borderless world, national loyalties have become unimportant. Do You agree?

ls it true that Man needs religion now more than ever? 'A good is equally concerned over the state of the world and ' Comment 'eader his country Youth is easily deceived because it is quick to hope " (Aristotle) Do you dgree? 'Medical science should do what is always good' even though it might not be always right ' How true is this stalement?

.LI

SAJC Mid-Year 07
A C Grayling says thal ihere are many reasons why Man goes lo war

ln 1932, Albert Einstein wrole lo S'gmund Freud to ask, Why war? He had come lo see ihal science cannol explain lhe fact ihal human beings, uniquely among animals' kill their own kind in highly organised ways, channelling vasl resources into doing so He fell thal the only defence against fulure war is world government What, he asked did Freud think?
Freud's answer was pessimistic Violence and inequality, he replied, are nalural to mankind. History sees weak people banding togelher to oppose strong individuals, their collective strength evenlually conslitliing a legalorder. Such an order might one day bring aboul the eagerly desired reign of "everlasling" peace. This peace can be achieved only by the paradoxical means of the colleclive's power to wage war on war mongers However, a utopian slale of peace is only theoretically conceivable because in praclice inequality, aggression and slrife are endemic io human existence
ln this answer there is neither real diagnosis nor real cure. lt assumes lhe naturalness of man's violence towards man, and sees collective aggression as the sum of individual aggression. Neither assumpiion, however, is convincingly explored Other animals are aggressive, but in specific and self'limiling ways: males compete for males, females protect iheir young, all are wonl 1o squabble over food However' fighls invariably end when one combalant flees or submns. Apart from man, animals do not prey on their own kind. A troop of baboons mighl drive away territorial invaders bul will not pursue them to enslave or kill them. Similarly, social animals often seek new territory in search of food, bul they do not seek to enlarge exisling lerrilory by conquest. I\Ian alone does such things. Whalever explains war, th.erefore, has to be souqhl elsewhere, in lhe complexity of man's economic, political and psychological conslitulion. Here suggestlons are legion.

'10

15

20

On one view, wars are atlempls to solve political and economic problems during

?5

periods of international inslability. Wars involve massive redireclion of produclive and social forces, lhereby easing, if only lemporarily, the pressures ihal cause them Moreover they quicken the pace of technological innovalion, and by selting massive logistical problems ior governmenls they sometimes creaiively forge new polilical and adminlstralive possibilities War is also seen as a lool by which countries struggle 10 gain advaniage over each other States do not slumble accidenlally into war' bLrl choose it as a deliberale means of securing influence 5nd control of resources These analyses say thal wa are instrumenls in the hands of political and commercial interesls; they are whal leaders gel us into. Forlunately for such leaders, these analysts point oLrt, public control through the simplistic. themes of nationalism and patriolism can be counied on to rouse enough young men for slaughlerlo follow

30

35

The lwentieth cenlury's maior wars might not exactly flt these lheories, bul their joinl effecl was indeed to shift power and redirect wealthi they weakened Europe and eslablished the Uniled States as supreme in lhe economic and mililary spheres The only other combatants lo beneflt rn lhe longer term we those who were so havily "defeated lhal lhey could reslruclure irom scralch, unhampered by illusions of pasl glory and the burdens of victory Like many wars, they were revolutionary in impact: Lenin s Soviei Union was crealed by the First World War, Mao's China by lhe Second One of the iess optimistic obserualions on whrch analysls oi all slnpes agree rs that war is no longer so conlainable a process Vast leaps in military technology have made ii tolal and polenlially final. The power of modern weaponry shows that we are

40

45

/r./

cleverer, but not wiser, than our ancestors, ior we have merely subsliluled the intercontinental ballisiic missile for the spear, but we slill behave like cavemen motivaled by greed and fear and unable 10 resist lights
lislen lo views aboul war expressed before lechnology reached its horror story proportions. There are sane voicas who argued lhal it is betler 10 achieve one's aims by negotiation than by bloodied swords, and those who indefaligably opposed war on the grounds thal it harms commerce and hinders progress These voices, alas, were lhe few unequivocal adversaries ol war Much misplaced optimism was expressed regarding the subiecl, exlolling u/ar's benefiis in promoiing virility purifying lhe race, and enhancing crealivity Olhers argued that war gives nations tbelr place in the world, while simullaneously purging their peoples of effeminacy and degeneracy. WaI was also regarded as integral lo human idenlity and ungovernable onae starled, and unsloppable until it exhausls itself. Nevertheless many lhought it has a place in the critical sludy of history, as the way capilalism willdeslroy ilself

ll is instructive to

50

55

60

Romanticisers of war describe war as a physical necessily for Man, arguing that it purifies and redeems ilself by bringing grealer evils than itself to an end ln this latler respect they are right, the war againsi Nazism was necessary in just this way The romanlicisers of war also decried the mechanisation of war, which meanl the passing ol coudly conflict, the end of chivakous jousts beh/veen knights under flutlering banners. They saw no romance in arlillery and mud They had an aesthelic view oi wart once, it was a sublime enterprise, in which every man put on a crown, when lhe band of flute-players gave lhe signal for attack; all the shields of the line gliitered wilh therr high polish, and mingled their splendour with the dark red of the purple mantles Men were sanctified by combal, {rom which rose all that is highesl in culture There is no greal arl possible to a nalion," said one romanticiser of war, 'but lhai which is bdsad on batlle
10

65

/o

Such misconception is, however, harmless in comparison 10 eugenic theories. Darwinism led some to treat war as a mechanism of natural seiection lts exlreme proponenls applauded wafs exlerminalion of inferior individuals and nations Oihers argued that war is compassionale because it rescues the weak and feeble from lhAir mGery This viel / did not go unchallenged as c tics argued ihat war is actually dysgenic, because the titiest and besl march off lo be slaughlered leaving ihe unfil at
home
Lo

/5

falher childten
ao

11

The only certainty of war is thal there are always losers in war' Sometimes lhe grealest losers are the victors. lt mighl be necessary io defend one s fleedoms or to ;ombat such evils as Nazism; so there are iusiifiable wars, and we have 1o be prepared to fight lhem. However, in its inception and characler lhere ls a prolound madness in war. To organise murder on lhe large scale - to plan ll lo conduct scienlific research into it. to build ils instruments in factories, to lrain men for il to applaud them when they do it; or, on lhe passive side of the equation io sit sileni when bombed houses, spread eagled bodies, weeping refugees, waste and deslruction appear on our ielevision screens suggesis thai we have embraced as normal something vaslly ugly and disaased. Perhaps Einslein's question should not
have been, "Why war?" but "How can we possibly tolerate ils existence?
Adaeiedtton tUhv War,lhe Heart oi ThLnos Applvrna Phrbsoph!
1o

90

the 21"r Centurv

Why does the author use quotation marks when he refers


peace in line 9?

to

everlasling"

Ill

Explain whal the aulhor means when he says thal peace can be achieved only by the 'paradoxical means of the colleclive s power to wage war on warmonoers" (line10 11). Use yaut own words as faras poss/b/e. I2l
l)sing yaur awn words as far as possib/e, slaie how, in paragraph 3, I\lan s aggression is different lrom that of animals. l2l

Explain what is meant by "we are cleverer, bul nol wiser, than our anceslors . in liae 47 Use your own words as far as possible I2l

(i) The aulhor does noi fully supporl lhe romanlicisers views on war

in

paragraph 9. SIate in your awn words as far as poss/b/e in what way lheir views are similar and how lhey are I31

differenl

(i!) What does the term, tomanticers of war" (line 61) tell you aboul the

authols atlitude towards those who supporl

wat?

I1l

From pa.agraph 10, explain why the authorthinks that juslificalio n for war based on eugenic theories is harmful? I1l Why does the author claim that "the grealest iosers are lhe victors" in paragraph 1'1?

t1l

The aulhor says that lhere is "profound madness' in war (lines 83-84). LJsng yaur own words as /ar as possib/e, explain what he means by this
Give ihe meaning of the following words as they are used in lhe passage You may write your answers in one word or a shorl phrase (a) endemic (line 12) (b) legion (line 24) (c) unhampered (line 41) (d) instructive (line 50) (e) extolling (line 55)

121

15t

10

Using material from paragraphs 4-8 (lines 25-60) summarise the authofs
reasons for why people wage war on each other and ihe outcomes of war. Write your summary in no more than 150 words not counting the opening words which are prinled below Use yoor own words as far as possib/e [7]
PPaptP $dqP

\ a, bP

dLse

11

ln the passage, Graylinq presenis difierenl views on war. Which of these views do you agree with? Are these views still reflected in the
Juslify your answer by referring both io what you have read in the passage and your ideas and knowledge t8l

/l/

You might also like